◆ わからない問題はここに書いてね 13 ◆

このエントリーをはてなブックマークに追加
1132人目のさくらたん

    , ― ノ)
 γ∞γ~  \   / ̄ ̄ ̄ ̄ ̄ ̄ ̄ ̄ ̄ ̄ ̄ ̄ ̄ ̄ ̄ ̄ ̄
 人w/ 从从) )  わからない問題はここに書いてね ♥
  ヽ | | l  l |〃 過去スレ,業務連絡・その他,関連スレ,
  `wハ~ ーノ)   数学記号の書き方例は >>2-10 の中にあるよ♪
   / \`「   \_________________

【前スレ】
◆ わからない問題はここに書いてね 12 ◆
http://cheese.2ch.net/test/read.cgi?bbs=math&key=999689496
2132人目の素数さん:01/09/24 23:46
うんてぃ
3132人目の素数さん:01/09/24 23:47
はぁ?
4132人目のさくらたん:01/09/24 23:47
【過去スレ】
◆ わからない問題はここに書いてね ◆
http://cheese.2ch.net/test/read.cgi?bbs=math&key=967755172
◆ わからない問題はここに書いてね 2 ◆
http://cheese.2ch.net/test/read.cgi?bbs=math&key=970795775
◆ わからない問題はここに書いてね 3 ◆
http://cheese.2ch.net/test/read.cgi?bbs=math&key=974911042
◆ わからない問題はここに書いてね 4 ◆
http://cheese.2ch.net/test/read.cgi?bbs=math&key=978209589
◆ わからない問題はここに書いてね 5 ◆
http://cheese.2ch.net/test/read.cgi?bbs=math&key=981372834
◆ わからない問題はここに書いてね 6 ◆
http://cheese.2ch.net/test/read.cgi?bbs=math&key=985594205
◆ わからない問題はここに書いてね 7 ◆
http://cheese.2ch.net/test/read.cgi?bbs=math&key=988952592
◆ わからない問題はここに書いてね 8 ◆
http://cheese.2ch.net/test/read.cgi?bbs=math&key=991223596
◆ わからない問題はここに書いてね 9 ◆
http://cheese.2ch.net/test/read.cgi?bbs=math&key=993571403
◆ わからない問題はここに書いてね 10 ◆
http://cheese.2ch.net/test/read.cgi?bbs=math&key=995448453
◆ わからない問題はここに書いてね 11 ◆
http://cheese.2ch.net/test/read.cgi?bbs=math&key=997329928
◆ わからない問題はここに書いてね 12 ◆
http://cheese.2ch.net/test/read.cgi?bbs=math&key=999689496
5132人目の素数さん:01/09/24 23:48
    ┌――――――――┐
    |.B1@ABCDE. |
    |FGHIJKLM|
    └――――――――┘
     ┌―――┬―――┐
     |      |      |
     |      |      |
     |      |      |
  age   ∧_∧  / ̄ ̄ ̄ ̄ ̄ ̄
  ▲C ( ´∀`)< あげとくか。
  ▽ \    )  \______
 sage |O  |_|_____|
     (__(_)
6132人目の素数さん:01/09/24 23:48
■□軍師助言■□
-------------------------------
          諸葛亮曰く
   ¶ヾ:ヾ:ヽ  / ̄ ̄ ̄ ̄ ̄ ̄ ̄ ̄
   ∧| | | |  |アヒャヒャヒャ
  (  ゚∀゚) < ア-ヒャヒャヒャヒャヒャヒャヒャヒャ
  /ヾΘΘヘ |ケケケヒャーーーヒャヒャヒャヒャッ
 / |:::|つ (つ \________
          HIT ANY KEY
-------------------------------
7132人目のさくらたん:01/09/24 23:48
【業務連絡・その他】
■900を超えたら新スレに移行準備.
■旧スレ側 → 終了宣言,新スレへの誘導.
■新スレ側 → 開始宣言と目次,旧スレのリンク,掲示板での数学記号の書き方例,業務連絡・その他,旧スレ側の残り問題の移動.
■数学板の要望スレで数学板の注意書き(リンク先)の変更依頼.

■単独の質問スレは,このスレか「くだらんスレ」に誘導して下さい.
■誤って過去スレに書き込まれた質問は,最新スレに誘導して下さい.


【数学板削除依頼スレ】
http://teri.2ch.net/test/read.cgi?bbs=saku&key=986384122 (レス削除)
http://teri.2ch.net/test/read.cgi?bbs=saku&key=987829968 (スレッド削除)

【リンク先更新総合スレッド】
http://teri.2ch.net/test/read.cgi?bbs=accuse&key=992178408
8あぼーん:あぼーん
あぼーん
9132人目の素数さん:01/09/24 23:49
  ┏━━━┓┏┓┏━┓┏┓┏┓                   ┏━━━━━┓       ┏┓
 ̄┗━━┓┃┗┛┗━┛┗┛┗┛. ̄|..┏┓| ̄ ̄| ̄ ̄| ̄ ̄| ̄ ̄|┗━━━━┓┃ ̄| ̄ ̄┃┃
 ̄| ̄ ̄| .┃┃┏┓┏━┓ ̄ ̄| ̄ ̄| ┏┛┗━━┓ ̄| ̄ ̄| ̄ ̄| ̄ ̄| ̄.┏━┛┃ ̄ ̄| ̄┃┃
 ̄ ̄| ̄ |.┃┃┗┛┗━┛. ̄| ̄┏┓..┗┓┏━┓┃┏━━━━┓ ̄| ̄┏┛  ┏┛ ̄| ̄ ̄┃┃
 ̄| ̄ ̄| .┃┃ ̄ ̄| ̄ ̄| ̄┏━┛┃ ̄ ┃┃| ┗┛┗━━━━┛| ̄┏┛┏┓┗┓ ̄ ̄| ̄┗┛
 ̄┏━━┛┃ ̄| ̄.┏━━┛┏━┛| ̄.┃┃ ̄| ̄ ̄| ̄ ̄| ̄ ̄| ̄┏┛┏┛┗┓┗┓| ̄ ̄┏┓
 ̄┗━━━┛ ̄ ̄ ┗━━━┛ ̄| ̄ ̄|┗┛| ̄ ̄| ̄ ̄| ̄ ̄| ̄ ̄┗━┛ ̄ ̄┗━┛ ̄| ̄┗┛
10あぼーん:あぼーん
あぼーん
11あぼーん:あぼーん
あぼーん
12132人目のさくらたん:01/09/24 23:49
【掲示板での数学記号の書き方例】
■数の表記
 ●スカラー:a,b,c,...,z, A,B,C,...,Z, α,β,γ,...,ω, Α,Β,Γ,...,Ω, ... (← ギリシャ文字はその読み方で変換可.)
 ●ベクトル:V=[V[1],V[2],...], |V>, V↑, vector(V) (← 混同しない場合はスカラーと同じ記号でいい.通常は縦ベクトルとして扱う.)
 ●テンソル(上下付き1成分表示):T^[i,j,k...]_[p,q,r,...], T[i,j,k,...;p,q,r,...]
 ●行列(1成分表示):M[i,j], I[i,j]=δ_[i,j]
 ●行列(全成分表示):M=[[M[1,1],M[2,1],...],[M[1,2],M[2,2],...],...], I=[[1,0,0,...],[0,1,0,...],...] (← 列(または行ごと)に表示する.)

■演算・符号の表記
 ●足し算:a+b
 ●引き算:a-b
 ●掛け算:a*b, ab (← 通常は"*"を使い,"x"は使わない.)
 ●割り算・分数:a/b, a/(b+c), a/(bc) (← 通常は"/"を使い,"÷"は使わない.)
 ●複号:a±b=a士b, a干b (← "±"は「きごう」で変換可.他に漢字の"士""干"なども利用できる.)
 ●内積・外積・3重積:a・b=(a,b), axb=a∧b=[a,b], a・(bxc)=(axb)・c=det([a,b,c]), ax(bxc)

■関数・数列の表記
 ●関数:f(x), f[x]
 ●数列:a(n), a[n], a_n
 ●平方根:√(a+b)=(a+b)^(1/2) (← "√"は「るーと」で変換可.)
 ●指数・指数関数:a^b, x^(n+1), exp(x+y)=e^(x+y) (← "^"を使う."exp"はeの指数.)
 ●対数・対数関数:log_{a}(b), log(x/2)=log_{10}(x/2), ln(x/2)=log_{e}(x/2) (← 底を省略する場合,"log"は常用対数,"ln"は自然対数.)
 ●三角比・三角関数:sin(a), cos(x+y), tan(x/2)
 ●行列式・トレース:|A|=det(A), tr(A)
 ●絶対値:|x|
 ●ガウス記号:[x] (← 関数の変数表示などと混同しないように注意.)
 ●共役複素数:z~
 ●転置行列・随伴行列:M† (← "†"は「きごう」で変換可.)
 ●階乗:n!=n*(n-1)*(n-2)*...*2*1, n!!=n*(n-2)*(n-4)*...
 ●順列・組合せ:P[n,k]=nPk, C[n.k]=nCk, Π[n,k]=nΠk, H[n,k]=nHk (← "Π"は「ぱい」で変換可.)

■微積分・極限の表記
 ●微分・偏微分:dy/dx=y', ∂y/∂x=y,x, D^(n)f(x) (← "∂"は「きごう」で変換可.)
 ●ベクトル微分:∇f=grad(f), ∇・A=div(A),∇xA=rot(A), (∇^2)f=Δf (← "∇"は「きごう」,"Δ"は「でるた」で変換可.)
 ●積分:∫[0,1]f(x)dx=F(x)|_[x=0,1], ∫[y=0,x]f(x,y)dy, ∬_[D]f(x,y)dxdy, 点[C]f(r)dl (← "∫"は「いんてぐらる」,"∬"は「きごう」で変換可.)
 ●数列和・数列積:Σ_[k=1,n]a(k), Π_[k=1,n]a(k) (← "Σ"は「しぐま」,"Π"は「ぱい」で変換可.)
 ●極限:lim_[x→∞]f(x) (← "∞"は「むげんだい」で変換可.)

■その他
 ●図形:"△"は「さんかく」,"∠"は「かく」,"⊥"は「すいちょく」,"≡"は「ごうどう」,"∽"は「きごう」で変換可.
 ●論理・集合:"⇔⇒∀∃∧∨¬∈∋⊆⊇⊂⊃∪∩"は「きごう」で変換可.
 ●等号・不等号:"≠≒≦≧≪≫"は「きごう」で変換可.

※ ここで挙げた表記法は1例であり,標準的な表記法からそうでないものまで含まれているので,後者の場合使う時にあらかじめことわっておいたほうがいい.
※ 関数等の変数表示や式の括弧は,括弧()だけでなく[]{}を適当に組み合わせると見やすい場合がある.
※ 上記のほとんどの数学記号や上記以外の数学記号は大体「きごう」で順次変換できる.
13132人目のさくらたん:01/09/24 23:51

★__________________________.
|.            │
│ はにゃ〜ん.   │
| γ∞γ~  \    │
│人w/ 从从) )   │
│ ヽ | |┬ イ |〃  │
│ `wハ~ . ノ)    │
│  / \`「 .     │
| 数学板さくらスレ. │
|_________________________│





(● ´ ー ` ●)ノ さくらスレ旗掲揚
14132人目の素数さん:01/09/24 23:52
まぁ、マターリと待ちましょう・・・( ´ー`)y-~~~~~
15132人目の素数さん:01/09/24 23:53
>>15は海綿状脳症に罹っている様です。
皆さん>>15が肉骨粉にされたら摂取しないよう気を付けましょう。
逆に遅くなりました。リソース食い過ぎです。
1,Bが定数の時、次のベクトルポテンシャル
A=(-By/2, Bx/2, 0)・・・@
  からなる確率密度B=(0, 0, b)であることを示せ。

2,ベクトルポテンシャル
  A=(0, Bx, 0)
に、適当なゲージ変換を実行すると上記の@が得られることを示せ。
なんだ、われずスレか。
906 名前:132人目の素数さん 投稿日:01/09/24 23:58
>1 名前:132人目のさくらたん 投稿日:01/09/24 23:45
>4 名前:132人目のさくらたん 投稿日:01/09/24 23:47
>7 名前:132人目のさくらたん 投稿日:01/09/24 23:48
>12 名前:132人目のさくらたん 投稿日:01/09/24 23:49
>13 名前:132人目のさくらたん 投稿日:01/09/24 23:51

あーあ
準備して6分なのか?
はう〜
半分以上荒らされちゃったよー
21あぼーん:あぼーん
あぼーん
なんか書き込み時間がとても遅いです
もっと早くできるのに・・・
例えば>>11>>12は投稿時間が逆転しています
23あぼーん:あぼーん
あぼーん
24あぼーん:あぼーん
あぼーん
25132人目のさくらたん:01/09/25 00:24
なんとか移転が完了しました.
ここ最近のさくらスレは私(ただし,「132人目のさくらたん」
というHNを使っているのは私だけではない!)が立てましたが,
次回以降は900を超えたらどなたかおねがいします.

***************************************************

 さくらスレ第13部開始(とっくにはじまってるけど)

***************************************************
>>17
確率密度?磁束密度じゃなくて?Bそれともb?問題はちゃんと書こう。
27132人目の素数さん:01/09/25 01:41
行列とはそもそもなんなのですか?
私にはただの数字の集まりとしか見えません。
どうして掛けたときにあんな変なかけ方をするのでしょうか?
根本的なことから教えてください。
28あぼーん:あぼーん
あぼーん
29132人目の素数さん:01/09/25 01:58
>>27
ベクトル空間というものを考えるとき、基底をとると、ベクトルは数の列で表されます。
ベクトル空間の間の線形写像というものは、「ベクトル空間の算法を保つ」写像です。
基底を入れて考えると、任意の線形写像はある行列で一意的に表せることがわかります。
そして、行列の積は線形写像の合成に対応して定義されています。
f、gがM、M‘で表されるとき、g・fはM‘・Mで表される。
すなわち、行列は線形写像を具体的に計算するための道具です。
非常に多くの応用ができ、それはいろいろな本に書いてあるYO!
1,Bが定数の時、次のベクトルポテンシャル
A=(-By/2, Bx/2, 0)・・・@
  からなる磁束密度B=(0, 0, B)であることを示せ。

2,ベクトルポテンシャル
  A=(0, Bx, 0)
に、適当なゲージ変換を実行すると上記の@が得られることを示せ
31あぼーん:あぼーん
あぼーん
32あぼーん:あぼーん
あぼーん
33あぼーん:あぼーん
あぼーん
34あぼーん:あぼーん
あぼーん
35132人目の素数さん:01/09/25 02:10
1,Bが定数の時、次のベクトルポテンシャル
A=(-By/2, Bx/2, 0)・・・@
  からなる磁束密度B=(0, 0, B)であることを示せ。

2,ベクトルポテンシャル
  A=(0, Bx, 0)
に、適当なゲージ変換を実行すると上記の@が得られることを示せ。
36132人目の素数さん:01/09/25 02:31
>29
>行列の積は線形写像の合成に対応して定義されています。
どういう意味ですか?もっと詳しく
37132人目の素数さん:01/09/25 02:41
>>36
一応下に式書いといたんだけど・・・
Vはベクトル空間を表すことにして、基底を定めておく。
f:V1→V2、g:V2→V3という線形写像がそれぞれある基底をとったときに
行列M1、M2で表されたとする。(具体的にどうやるかは、基底の像を考えるとわかる。)
このときに、合成写像g・f:V1→V3もやっぱり線形写像で、
これを表す行列が、行列の積M2・M1で表される(ただし、列ベクトル表示のとき)
となるわけ。これが行列の積の意味。
これで、よろしいでしょうか?具体例で確かめてみるとよくわかるYO!
381:01/09/25 02:44
bingo,bingo,山城bingo
40132人目の素数さん:01/09/25 04:28
>>8
感動した!!
41前スレの695:01/09/25 07:33
亀レスですが
問題の解説まで詳しくしてくださった
はなうさんと798の方
どうも有難うございました。
最近荒らしばっかでまともな質問無くて暇してるだろ
もったいないお化けからの挑戦だ解いてみやがれ。
特にこいつ→(´-`).。oO
w+x+y+z=100 (ただしw≧-1,x≧0,y≧1,z≧2)をみたす整数w,x,y,zの組(w,x,y,z)は何通りあるか?
43132人目の素数さん:01/09/25 09:40
さくら12号から移ってきました.
皆さんありがとうございます。たいへん参考になります。
そうなると、私なりの考えでは
@「nが奇数のとき、その素因数分解にあらわれる素数の種類をmとすれ
ば、Z/nZの指数2の部分群は2^m-1個ある」となったのですがどうでしょうか?

Aついでに、
>Hom(Z/6Z×Z/4Z,Z/2Z)=Hom(Z/6Z,Z/2Z)×Hom(Z/4Z,Z/2Z)
>から4つあることもわかるね
のところで、「位数が偶数の巡回群→Z/2Zへの準同型写像」は
0写像以外に1つしかないですよね?(一応確認のため)

B“Gの指数2の部分群”と“Hom(G,Z/2Z)”が一対一対応している
これは群論の本にあるのですか?

?ばかりの長い文ですいません。。
44132人目の素数さん:01/09/25 10:40
>>43
1と2はあってるとおもわれ。
3はそんなむづかしくないよ。
{準同型φ}→{Kerφ}
という対応で全単射になる。
前スレでZ/qZの乗法群を完全に決定してるレスがあったけど
それをみとめると“位数が偶数の巡回群→Z/2Zへの準同型写像」は
0写像以外に1つしかない”などももっとすっきりわかる。
ただしそれは結構むづいとおもう。たしか有限体上のGalois理論に
もちこむんだったとおもう。2べきとかに話をかぎれば
かんたんだけど。
>>25

いつもごくろうさんです
最近は変わり目を狙われているらしくて初期化荒しが
横行しているようです

気が早いですが 次スレ立てるかた
厨房時間帯を避ける時間差防衛や
dat状態での作成の効果検討など可能な方策をお願いします
46ん?:01/09/25 11:10
>>43
指数2の部分群は全体を半分にわけてるね。位数が奇数のとき
何で半分にわけられるの?
47132人目の素数さん:01/09/25 11:19
>>44
円分体のGalois理論だった。
>>46
Z/qZの単数群の位数はq(1-1/p)。たとえばZ/125Zの位数は
125(1-1/5)=100。つまりつねに偶数だよ。
48なし:01/09/25 12:40
もったいないお化けからの挑戦:
>w+x+y+z=100 (ただしw≧-1, x≧0,y≧1,z≧2)
>をみたす整数w,x,y,zの組(w,x,y,z)は何通りあるか?

例えば整数nに対して x+y=n; x>=0, y>=0 を満たす整数 x,y の個数は、
グラフから明らかなように n+1 個です。
w=W-1; y=Y+1; z=Z+2 とおくと、解くべきものは

>x+Y+Z+W=100 をみたすゼロ以上の整数W,x,Y,Zの組
>(W,x,Y,Z)は何通りあるか?

となる。x+W を固定し Y+Z=100-(x+W) と書けば 101-(x+W) 通り。
明らかに 0≦x+W≦100 が十分条件で x+W = k とおこう。
ここで x+W = k となるのは k+1 通り。よって、求める個数は

Σ(101-k) (k+1) [個]
k

である。kの範囲は 0≦k≦100。お化けは退治された。
49なし:01/09/25 12:44
>>48
君にも間違いはあるものさ。x+Y+Z+W=102 となるので
Σ(103-k) (k+1) [個]
k
であり、kの範囲は 0≦k≦102。
50132人目の素数さん:01/09/25 13:17
>>47
またまちがいだった。有限体上の多項式をあつかうんだった。
奇素数pと3以上の自然数eにたいし
Z/p^eZ~が巡回群⇒Z/p^(e+1)Z~が巡回群 (R~は単数群)
は簡単。そこでe=3のときが本質だけどそれは多項式
T^{p(p-1)}-1がT^(p-1)-1でわりきれてその商が
(T^(p-1)-1)f(T)+pなる形でf(T)が任意の整数でpの倍数になること
がいえるのでT^{p(p-1)}-1がp^3の倍数にならない
ものが存在することがいえる。これでOK。
51132人目の素数さん:01/09/25 17:09
>37
丁寧にありがとうございました
52132人目の素数さん:01/09/25 17:34
  1 a a^2
行列1 b b^2の行列式値が(b-a)(c-a)(c-b)を利用して
1 c c^2

  1 1 1 1 1
a b c d e
行列a^2 b^2 c^2 d^2 e^2 の行列式の値を求めよ
a^3 b^3 c^3 d^3 e^3
a^4 b^4 c^4 d^4 e^4


すみません。誰かわかる方いらっしゃったら教ええください。。
53132人目の素数さん:01/09/25 17:35
1 a a^2
1 b b^2の行列式値が(b-a)(c-a)(c-b)を利用して
1 c c^2

1 1 1 1 1
a b c d e
a^2 b^2 c^2 d^2 e^2 の行列式の値を求めよ
a^3 b^3 c^3 d^3 e^3
a^4 b^4 c^4 d^4 e^4


すみません。誰かわかる方いらっしゃったら教ええください。。
54132人目の素数さん:01/09/25 17:38
52の書き込みが変になったんで書き直しました。何回もすみません。
55もったいないお化け:01/09/25 17:50
>>48-49
くっ、くそなかなかやるな。
じ、じゃぁ
一般に
a+b+c+d+……=N
(ただし変数(a,b,c,d,……)はM個ですべて0以上の整数、NはO以上の適当な整数の定数)
をみたす組(a,b,c,d,……)は何通りあるか?
ただし答はΣ(シグマ)や!(階乗)などの記号を用いてあらわしてもよい。
これでどうだ!
すいませーん、日本語教えてくれませんか?

ring
σ-ring
algebra
σ-algebra
age
>>53
Vandermondeの行列式ね。
誘導どうりじゃないほうが楽。
もとめる行列式をP(a,b,c,d,e)とするとa=bで0になるので
因数定理から(a-b)でわりきれる。以下同様にして
P=Q(a,b,c,d,e)(a-b)(a-c)(a-d)(a-e)(b-c)(b-d)(b-e)(c-d)(c-e)(d-e)
次数をかんがえてQは定数でa^4b^3c^2^d^1の係数を比較してQ=1をえる。
誘導どうりにやれんことはないけど大変。10個の
(2次行列式)×(3次行列式)をたすんだろうけどしんでまうね。
59132人目の素数さん:01/09/25 19:49
>>55
M本のエンピツをN人に配る。エンピツを一本ももらえない人がいてもいい。
分配の仕方は何通り?

M個のエンピツにN-1個の「しきり」をいれて…で考えれば高校で習う
順列の問題(mHnとか書くのかな)だとわかる。
60もったいないお化け:01/09/25 21:04
↑☆大正解☆
やりますな、すばらしい。
ちなみに、Cを使って(N+M-1)CN=(N+M-1)C(M-1)ともかけます。
それから順列の問題じゃなくて組合せですけど。
まえの問題もそれで解いてほしかったのよ。
61132人目の素数さん:01/09/25 21:47
問1
岩沢理論の岩沢さんはどこの大学卒ですか?
62132人目の素数さん:01/09/25 22:05
東大卒で、母校で助教授くらいまではやってたんでしょう。
そのあとはMIT、プリンストンかな。
63132人目の素数さん:01/09/26 00:25
| ((log2nπ)/2)/logn | (nは十分大)
が有界であることをピシッと示したいのですが、どんな手法でやるとよいでしょうか?
些細な質問ですが、どうぞよろしくお願いします。

(これを用いてスターリングの公式を書き表すと、何かご利益があるようなのですが・・・)
64 ◆pvySbQO2 :01/09/26 00:30
>>63
2π<nならlog(2nπ)<2log(n)。
58の説明をもうすこしわかりやすく言ってもらえないでしょうか。
すみません。。
66132人目の素数さん:01/09/26 00:50
f(x)をxの多項式とする。
すべての自然数nについてf(x)=lognとなる
f(x)は存在しないことを示せ

この問題をお願い致します
>>66
・問題のf(x)は定数じゃない
・n->∞のとき、f(n)/n^k -> 0 (k = 1, 2 ..)なので、f(x)があれば定数
というわけで、なし
>>67の訂正
>・n->∞のとき、f(n)/n^k -> 0 (k = 1, 2 ..)なので
じゃなくって、
・n->∞のとき、log(n)/n^k -> 0 (k = 1, 2 ..)なので
69132人目の素数さん:01/09/26 01:46
53の問題を誘導どうりに解くにはどのようにしたらいいのでしょうか?
70132人目の素数さん:01/09/26 02:16
>>64
どうもです。安心しました。
ローレンツ条件を利用し、スカラーポテンシャル、ベクトルポテンシャルに対する
(△-1/c^2 ∂^2/∂t^2)φ=ーe/μ
(△-1/c^2 ∂^2/∂t^2)A=ーμj
を導出せよ

という問題です。
どうもこういった問題は苦手で・・・
72132人目の素数さん:01/09/26 03:01
0<p<1, 0<x<1に対して、Σ[n=1,∞]x^{(log n)^p}の収束・発散を判定してください。
見にくいですが、xの(log n のp乗)乗のnに関する無限和です。
73 ◆pvySbQO2 :01/09/26 05:38
>>72
a/n<x^((log(n))^p)
となる正の定数aがあるので発散する。
74132人目の素数さん:01/09/26 06:25
こんな話に聞いたことがあるんです。
中心が同じ位置にあり大小異なる円があるとします。
中心が直線に動いたとき(例えば左から右に)
大小異なる円は同じ円周分
移動することになると思うんですが、計算すると
あわないというものなんです。
自動車でも、電車でもいいんですがあの、タイヤ、
車輪のことをいってるんですが。
これって証明できないんですか、やっぱり?
75132人目の素数さん:01/09/26 09:10
コーシーの積分公式って何年で習うんですか?
76なし:01/09/26 09:16
>>74
>中心が同じ位置にあり大小異なる円があるとします。

用語では「同心円」と言う。

>中心が直線に動いたとき

「転がったとき」といいたいのかな。

>これって証明できないんですか、やっぱり?

検証可能です。
実験方法:500円玉と1円玉を1周だけ転がしてみる。

円周の長さは半径に比例しますから、半径が違うと
1周したときに転がる距離も異なります。
77なし:01/09/26 09:33
>>55
>一般に a+b+c+d+……=N (ただし変数(a,b,c,d,……)は
>M個ですべて0以上の整数、Nは0以上の適当な整数の定数)
>をみたす組(a,b,c,d,……)は何通りあるか?

(M,N)に対する2重帰納法を使えば明らか。
>>74
これを言いたいのかな?
76の500円玉と1円玉が同心円となって固定(接着)されている。
2つの円周は車輪のように、2つのレールの上に乗っている。
1回ころがると、2つの円周の長さは異なるのに、レール上は同じ長さだけ
移動する。何故か。

・・・これは、片方(または両方)が、「すべって」いるんです。
>>35
君、物理版でスレ立ててた人?
私は物理は余り詳しくないが、定義に従って考えれば良いんじゃない?
砂川重信「理論電磁気学」の42ページから45ページ辺りを見れば
すぐ分かると思うよ。

1.B=rotAよりclear
2.u=Bxy/2とすればよい。
80132人目の素数さん:01/09/26 10:59
>>75
2年の解析。
81132人目の素数さん:01/09/26 11:20
位相の問題です
誰か賢い方これ教えてください!わかりません。


Xを第二可算公理を満たす完備距離空間、f:X→Xを連続写像とするとき
次の(ア)と(イ)は同値である事を示せ。

(ア)U,VをともにZの空でない開集合とするとき、ある非負の整数n≧0があって、
f^n(U)∩V≠φとなる。 ただし、f^n(U)=ff…f(U) (n次合成写像)

(イ)あるx_0∈Xがあって、その軌道O(x_0)={f^n(x_0) | n≧0}はXで稠密
8281です。:01/09/26 11:21
すみません、訂正です。

Xを第二可算公理を満たす完備距離空間、f:X→Xを連続写像とするとき
次の(ア)と(イ)は同値である事を示せ。

(ア)U,VをともにXの空でない開集合とするとき、ある非負の整数n≧0があって、
f^n(U)∩V≠φとなる。 ただし、f^n(U)=ff…f(U) (n次合成写像)

(イ)あるx_0∈Xがあって、その軌道O(x_0)={f^n(x_0) | n≧0}はXで稠密
8381です。:01/09/26 11:22
あれ??φは空集合のつもりなんですけど変な変換になってしまいました。
84先生、単位下さい!:01/09/26 11:53
教えてください。

TS^3とS^3×R^3が微分同相である事を証明せよ。
8576.:01/09/26 12:52
(1/π)×∫[0、π]{sinnx/(cosa-cosx)}dx
ただしnは整数.
86132人目の素数さん:01/09/26 13:17
>>85
log(cosa-cosx)を微分してみてください。
87132人目の素数さん:01/09/26 14:22
>>82

イ⇒ア

nが任意でf^n(U)∩V=φなら軌道が立ち入らない区域ができて稠密に反するため自明

ア⇒イ

第二加算公理で加算個の基をとってあげて、稠密ではない場合を考えればこれまた立ち入らない開集合が
できるのでそれとの共通部分なんかを考えてあげちゃったりなんかすると自明
8874:01/09/26 14:55
>>78
ということは、滑っているで証明終わり
ということですか?
でも、固定されていて滑らないと
仮定されていたときはどうなるんですかね?
現に、電車などでは片方が滑っていては
安全上問題にならないんですか?
89132人目の素数さん:01/09/26 15:00
>>84
S^3をR^4の部分多様体と思う(原点の中心の単位球)
このとき、S^3上のベクトル束TS^3が自明であることを示すには、
TS^3の切断γ1、γ2、γ3で、S^3の各点pでγ1(p)、γ2(p)、γ3(p)
がファイバーの基底になっているもの(フレーム)を構成すればよい。
具体的には、R^4の直交座標でS^3の点をp=(x、y、z、w)と表したとき、
(−y、x、−w、z)(−z、w、x、−y)(−w、−z、y、x)
はpにおけるノルムが1の互いに直交する接ベクトルである。
よって、γK(K=1、2、3)を上のように定めると、
TS^3=S^3×R^3(接ベクトルの自明性)が導かれる。
ここで、上の三つの切断は、四元数の計算で見つけることができます。
90機械人間:01/09/26 15:26
中2で習う連立方程式の代入法のやり方
教えてください。
91132人目の素数さん:01/09/26 15:35
>>88
そのような条件を付けたら当然動けません。

電車の車輪がどのような形になっているのか、
またカーブなどではどのようにレールの上を走行しているか
ぜひ自分で調べてみてください。

自動車の場合、タイヤが空転している事はよくあります。
その辺の事情については免許を持っている人なら教習所でみんな学びます。
周りの大人に聞いてみましょう。
92132人目の素数さん:01/09/26 15:39
93132人目の素数さん:01/09/26 15:46
>>87
あまり自明自明と
いわれましてもよく分からんです・・・
頼んでおきながら何なんですが、
もうちょっと詳しくお願いします。
94132人目の素数さん:01/09/26 15:47
 次の命題の証明がわかりません。

 一様空間が全有界であるための必要十分条件は、任意の超フィルター
がコーシー・フィルターであることである。
95132人目の素数さん:01/09/26 16:37
>>93
どっちがわからないの?
96132人目の素数さん:01/09/26 16:51
90の人の揚げた
http://www.geisya.or.jp/~mwm48961/math/qu_2_1.htm
(中学生向けサイト?)
のほほんとしてて良いなぁ。
こんな調子で数論とかマターリ教えてくれるサイトって無いの?

無いか、やっぱ
9796:01/09/26 17:01
90 → 92 のまちがい

ま、サイトは無いとしても(あるかもしれないが)、
解りやすい書籍って無い? ※ 仮に分野は数論に限ってみます。
解りやすいっていうのは、オレにとっては
* 意味のある式(公式)の後に、例えばっていう数値(具体的数値による計算)が載っていること。
* 例題・練習問題には解法・解答が載っていること。
・・・・なんだけど。

もちろん、そんな書籍にしてもじーーっくり読まねば理解できないことは分かっているが・・・。
98132人目の素数さん:01/09/26 18:23
>>89
申し訳ありません、切断って何でしょうか?
厨房で済みません。
99132人目の素数さん:01/09/26 20:58
>>95
ア⇒イ
をもうちょっと詳しく教えてくれたらうれしいな♪
100132人目の素数さん:01/09/26 22:32
>>98
E→Mがベクトル束のとき、滑らかな写像γ:E→Mが切断であるとは、
π:E→Mが射影(pのファイバー上の点にたいしてpを対応させる写像)のとき、
π・γ:M→Mが恒等写像になることをいいます。
一応説明しておきますと、TS^3の点は(p、t1γ1+t2γ2+t3γ3)と表せますが
(ただし、tK∈R k=1,2,3)が、このとき、
φ:TS^3→S^3×R^3 (p、t1γ1+t2γ2+t3γ3)→(p、t1、t2、t3)
が微分同相を与える、γはこのとき基底の役割を果し、それは前レスで具体的に求めた
ものになるわけです。
101132人目の素数さん:01/09/26 22:34
>>100
訂正 一行目のγ:E→Mは、γ:M→Eです。
102132人目の素数さん:01/09/26 23:56
http://www.geocities.co.jp/CollegeLife-Cafe/7513/mon.jpg
この問題、誰かおしえてもらえないでしょうか?
一応自分でも解いてみたんですが、自信がないので
103132人目の素数さん:01/09/26 23:59
ちなみに、角度Xをもとめるのです
104数学すき:01/09/27 00:04
今日は数学の『または』について質問したいとおもいます

(1)AまたはB  (2)cまたはD という条件があるとき
(1)かつ(2)といったら

4種類の可能性を含みますよね つまり

(1)AかつC
(2)AかつD
(3)BかつC
(4)BかつD

でも最近参考書を解いていて おもったのですが
(1)または(2)というとき


かならずしもこの4種類が成立しない場合でも(1)または(2)が成立すると
いっているような時があるとおもうのですが どうなんでしょうか?


例 x^2-bx+c=0の解 α βが 1と0の間にあるとき どんな不等式が成立するか
という問題のとき

解答では   f(x)=x^2-bx+cとおくと
f(1)≧0 かつf(0)≧0 かつD≧0   かつ  0<軸のx座標<1
という式をたてますね
このときってf(1)=0かつf(0)=0じゃあ題意にあわないですよね
だけどもf(1)≧0 かつf(0)≧0という『式の可能性』をみたらf(1)=0かつf(0)=0という可能性ももちろん含みますよね

つまりf(1)=0かつf(0)=0を含まないときでも
f(1)≧0 かつf(0)≧0という言い方をしますよね?

どうでしょうか?
105132人目の素数さん:01/09/27 00:11
>>100
有難う御座います!
106http:// p2156-ipad02kyoto.kyoto.ocn.ne.jp.2ch.net/:01/09/27 00:12
>>102
答えは30°

∠A=20 ∠B=80 ∠C=80 ∠BCD=50 ∠CBE=60 としとく。
AB上に∠BCF=60となるようにFをとる。BFとBEの交点をGとする。
このときBC=BG=BD より∠BGD=80 ∴∠DGF=40
また∠DFG=40 より DG=DF また、EF=GEとなるから
DEは∠GEFの二等分線で、∠GEF=60より、求める∠DEG=30となる。
  
107132人目の素数さん:01/09/27 00:17
>>106
有り難うございます。
自分の答えともあっていたので安心しました
108名無し:01/09/27 00:21
>>104
まず,例としてあげた問題の件だけど,解が0と1の『間に』
って言うのは,おそらく 0<x<1 の範囲にってことでしょ?
となると,f(0)≧0かつf(1)≧0 って言うのは,必要条件であるけど,
十分条件ではない。実際,片方において等号が成り立つ時点で,
方程式がx=0またはx=1を解に持ってしまう。正確には,
f(1)>0 かつ f(0)>0 かつ・・・ となるべきじゃないの?
>>99
イの条件で稠密ではないと仮定すると
Xから軌道の閉包を除いた開集合は空ではないよね?
この開集合の中の点を一つとって、この点の近傍をとても小さく(距離空間)とれば
近傍ごと、この開集合に含まれるよね?こういうε-近傍を、開集合の基のなかから
一つ選んでおく。

アの条件から、x_0を含む基をとってfで写していくとかならず、このε-近傍と交わってしまうので
x_0を含む基はどんなに小さくとっても、最初に選んだ通過しない筈の開集合の中にどっぷりはまって
しまうんだ。
110104:01/09/27 01:25
>108さん
>f(1)>0 かつ f(0)>0 かつ・・・ となるべきじゃないの?
でもそうするとf(1)=0かつf(0)>0とか f(1)>0かつf(0)=0という可能性が
はぶかれてしまうよ
f(0)≧0かつf(1)≧0 って言うのは,必要条件であるけど,
十分条件ではない。っていうのはそのとうりですね
111俺は36にもなってるんだ:01/09/27 01:43
子供の夏休みの算数の宿題に出たもので、とうとうお手上げでした。
ずっと気になってモヤモヤしてます。どなたかご解答ください!

1匹のカタツムリが、まっすぐに6分間進んでいったとします。
そのカタツムリを何人かの子どもたちが観察していました。
子どもたちは相談して、いつも少なくとも一人は観察していました。
子どもたちはそれぞれ1分間ずつカタツムリを観察しました。

先生は子どもたちに「カタツムリは何センチ進みましたか。」と尋ねました。
するとどの子も、「30cm進みました。」と答えました。

【問題】

この6分間にカタツムリは最大何cm進めるかを答えなさい。
112名無し:01/09/27 01:47
>>104
>f(1)=0かつf(0)>0とか f(1)>0かつf(0)=0という可能性が
はぶかれてしまうよ
たとえば,f(1)=0かつf(0)>0のとき,f(x)=0はx=1を解に持つので,
『2解がともに 0<x<1の範囲にある』という条件に反すると思うのですが。
『2解がともに0と1の間にある』っていうのを,あなたはどう解釈したのですか?
『0<x<1の範囲に・・』なのか,『0≦x≦1』なのか・・・
113132人目の素数さん:01/09/27 02:09
>>111
180cmじゃないの?
もっと遠くに行けるってこと?
114132人目の素数さん:01/09/27 02:12
死ね
R×R−>Rの関数gが任意の実数a,b,cに対して
g(a,b)+g(a,c)=g(a,b+c)
g(g(a,b),c)=g(a,bc)
g(a,c)+g(b,c)=g(a+b,c)
g(a,1)=a
となるときgを全て求めよ。
これ分かる人いますか。
>115
前スレで解決済み
>>116
解決していません。
118132人目の素数さん:01/09/27 07:07
>>116
結局、出題者が回答を理解できないところで打ち切ったんじゃなかったっけ?
俺的にもその問題は終了してるんだが(w
>>118
理解できない回答ってどこにあるんですか。
120なし:01/09/27 09:33
記号論理の分かる人、間違いを探してください。
x,y は変数で、Pを述語とするとき、

?? ∀x:∃y:P
⇔∀x:¬∀y:¬P
⇔¬∃x:∀y:¬P
⇔¬∃x:∀y:¬P
⇒¬∀y:∃x:¬P
⇔∃y:¬∃x:¬P
⇔∃y:∀x:P. ??

?? ∴∀x:∃y:P ⇒ ∃y:∀x:P. ??
121無者:01/09/27 10:59
>>111
どの子どもも、1分間見てて、その間に30cm進んだのを見た、ということだと仮定して※、
見ていなかった時間はないので、
6分間の時間のあらゆる瞬間は、必ず誰かが見ていた1分のなかに含まれる。
つまり、6分のうち、どの1分間も30cmより進んでいることはない
=平均分速30cm以下として、
最大で進める道のりを出すのでは。

※子供たちが相談して、全部で「30cm」進んだと答えた、と受け取る子もあろうて。
 やな問題やなぁ。日本語の使えない人が作ったんだね。
>>120
⇔¬∃x:∀y:¬P
⇒¬∀y:∃x:¬P
がおかしい。
123132人目の素数さん:01/09/27 11:56
ロピタルの定理で質問。
@lim[x→1] lnx/x^3とAlim[x→∞] (lnx)^3/x^2が
解けません。@は自分でやってみたら1/3になるんですけど
グラフにすると0のはずなんです。Aの方はいろいろやってみた
んですが、ちんぷんかんぷん。よろしくお願いします。
124132人目の素数さん:01/09/27 12:34
なんでもかんでも微分すればいいというものではありません。
125名無し:01/09/27 13:24
>>123
@ ロピタルの定理が使える条件をもう一度確認すべし。
A これは,ロピタルの定理を3回繰り返して使えばできる。
126132人目の素数さん:01/09/27 13:30
(COS二乗)20゜+(COS二乗)40゜+(COS二乗)80゜
12772:01/09/27 13:47
>>73
そのaをもう少し具体的に与えていただけますか?
128132人目の素数さん:01/09/27 13:59
>>126
与式=(1+cos40)/2+(1+cos80)/2+(1+cos160)/2
=3/2+{cos80+cos(100-60)+cos(100+60)}/2
=3/2+(cos80+2cos100cos60)/2
=3/2+(cos80+cos100)/2
=3/2+{cos80+cos(180-80)}/2
=3/2
°(度)を補って読んでくれい
129132人目の素数さん:01/09/27 14:28
>119

前スレ>87によればg:R×Q→Rに制限すれば g(a,b)=abなので
bに関して連続な領域ならばこれに限られる。
これ以外の解があるとすればg(a,b)がbに関していたるところ
不連続な領域でのみ実現される
つまり初等関数の合成では構成できない

前スレ>717で
>この問題がきれいな答が出ない問題であるという理由が分かりません。

と書いているけど、初等関数で構成できないものを探すという時点で
きれいな答でないことは分かってもらえていると思うのだが?


>理解できない回答ってどこにあるんですか。

bに関していたるところ連続でない場合に関しては前スレの
>32
>410-411
>722-723
>727
で書かれている通りなので、これらが理解できない回答ではない?
130132人目の素数さん:01/09/27 14:53
この問題ってわかります?

宇宙人がパーティーをしています。
彼ら1人1人の片手には2本以上の指があります。
すべての宇宙人が左右あわせて、同数の指を持っていますが、
それぞれ片手の指の数は全員ばらばらです。
(*例えば全員が右手を出したとき、同じ数の指の宇宙人は
いないということです)
この部屋にいる宇宙人の指の数がわかれば、
必然的に何人の宇宙人がいるのかがわかりますが、
200〜300本の指があるとしたら宇宙人は何人いるでしょう?

ひとつの答えになるらしいんですが・・・
>>129
{(x,y,g(x,y)|x∈R,y∈R}がR^3で稠密でないなら
g(x,y)=xyとなることは質問する前から分かってますし
初等関数の合成で構成したいわけでもありません。
32,410,411,727に書かれていることは
条件を満たすgはたくさんあるということだけです。
>>131
>{(x,y,g(x,y)|x∈R,y∈R}がR^3で稠密でないなら
>g(x,y)=xyとなることは質問する前から分かってますし

おいおい・・・R^3で稠密って言葉の意味分かってる?
閉包とったらR^3を埋め尽くしちゃう場合について聞いてるのか?

>>131
>初等関数の合成で構成したいわけでもありません。

じゃぁ何で表現したいの?
どういうものをきれいな答だと思ってるの?

>32,410,411,727に書かれていることは
>条件を満たすgはたくさんあるということだけです。


前スレ>410-411には一例が載ってる筈
・・・・・・ちゃんと読んでる?
133132人目の素数さん:01/09/27 15:37
>>130
問題が不正確か説明不足ではないですか?。素直に解釈すると
結論が一つにならないので。
「どの二つの片腕も同数の指をもたない」と解釈しても、10人と11人の
場合が出てきますが???
134132人目の素数さん:01/09/27 15:41
不変測度って何よ?
確率密度とはちがうものなの?
>>133
>10人と11人の場合が出てきますが???
それに、2本指を持つやつが必ずいるとは限らないので(少なくとも3本以上,4本以上とか)、
12以上もありえるし・・・。
136ふひゅう:01/09/27 16:00
自然数(e=2.71・・)ってなんですか?
何でこの値は無理数なのですか?

分かるように説明してください(T_T)
137130の仲間です:01/09/27 16:08
>133
 その10人と11人というのを導き出すにしても、どういう風に考えればよろしいのですか?
 さっぱり見当がつきません・・・。(泣)
138132人目の素数さん:01/09/27 16:18
>>137
n人の宇宙人がいて、全員同数の指があるが、どの二つの指も本数がことなるとする。
このとき、腕に「指のつき方」が2n通りないといけないから、
一人の宇宙人は最低でも2n+3本の指を有する。
というのは、「2本、3本、・・・2n+1本の指がついている」腕がある
場合が最小本数になるが、この場合、宇宙人の指が同数になるのは、
一人をとったとき、右手k本、左手2n+3−k本でないといけないから。
よって、n人の宇宙人がいれば、指はn(2n+3)本である。
n(2n+3)<300を解くと、n<11となる。
もっと少ない人数でもできますが、(二人でもできるよ)12人以上では
できないです。
139132人目の素数さん:01/09/27 16:19
箱Aには赤玉15個と白玉5個、箱Bには赤玉4個と白玉16個が入っている。
サイコロを振って、5以上の目が出れば箱Aから、その他の時はBから
ランダムに2個の玉を取り出すものとする。
抽出結果が2個とも白玉の時、箱Aが選ばれた事後確率を求めよ。

が、わかりません。
P{A|WW} = P{WW∩A} / P{WW}
で、P{WW∩A} = P{WW} * P{A} = (5 / 20) * (4 / 19) * (2 / 6) = 1 / 57
P{WW} = (5 / 20) * (4 / 19) = 1 / 19
で、P{A|WW} = 1 / 3
じゃないの?答えは1 / 25なんだけど、どうしてもわかりません…。
140さる:01/09/27 16:18
ある掲示板で√2+√3+√5+√7が無理数である事の証明をたずねているところが
ありました。何かエレガントな証明法(できれば一般化しやすい証明法)を考
えてみたのですが,高校生にも説明できてかつ,エレガントな証明というのがどうも
思いつきません。どなたかわかる方いらっしゃいませんか。ちなみに,高校の範囲を
超えて良ければ,次のような証明を思いつきました。概略で述べます。

まず,±√2±√3±√5±√7なる数字を考えます。ここで±の記号は複号同順とい
うわけではありません。つまり,16個の数字を考えているわけです。(その16個の数は
どれも整数でないですよね)その16個の数すべてを集めて作った集合Aを考えます。集
合Aの任意の異なる2数x,yについて,その和あるいは差をつくると,無理数になる事が
簡単にわかるので,集合Aのなかに有理数があるとしても1つしかない事がすぐに分か
ります。つぎに,集合Aの要素すべてを解とする16次方程式f(x)=0を考えると
係数がすべて整数になることがちょっとしたテクニックでわかります。すると,
f(x)が既約でないなら,その既約因子(こんな言葉ありましたっけ?)を考えれば
Z[x]の既約多項式で,集合Aの要素だけを解に持つものが存在し,しかも1次式でない
ことが分かります。あとは,その既約多項式=0の解を有理数全体がなす体Qに付け加
える事を考えれば,集合Aの中に1つだけ有理数があるということに矛盾するので,結
局集合Aの要素はすべて無理数という事になります。

さて,この証明に不備はあるでしょうか?あったらお教えください。
また,高校生にも説明できるエレガントな方法はありますか?あったらお教えください。
さらに,どうせ,高校の範囲を超えていいならもっと簡単な証明法があるという場合はお教え願えますか。
明日の朝まで掲示板をのぞけないので,レスできなくなりますがよろしくお願いします。
141132人目の素数さん:01/09/27 16:30
y=arctan(x)を
微分すると
どうなるのですか
??
142さる:01/09/27 16:36
>Z[x]の既約多項式で,集合Aの要素だけを解に持つものが存在し,しかも1次式でない
>ことが分かります。
のところの,「しかも1次式でない」が,全然成り立ってませんね。残念。
143もったいないお化け:01/09/27 16:37
>>136
自然数じゃなくて自然“対”数の底
んで無理数だから無理数なのまた無駄な議論が始まるから、そういうことは聞かないの。
144もったいないお化け:01/09/27 16:50
141
1/(1+x^2) 常識。
145もったいないお化け:01/09/27 16:55
>>139
常識で考えても1/3はねーだろ。
箱Aから白2つを取り出す確率は1/57
箱Bから白2つを取り出す確率は24/57
だもんで白が2つ出てきたときは1/25の確率で箱A
3連続カキコだ〜
>>143
何故、無駄な議論が始まるの?
キミがeが無理数であることの証明を一つも知らないだけじゃん?
147132人目の素数さん:01/09/27 17:14
>>146
既出です。最近もどっかで見た
148136:01/09/27 17:30
じゃあ、ちょっと議論転換。

e=2.71・・・
ってなんで、この値なんですか?
>147
そういうことは>143に言ってくれよん

>148
微分は知ってる?
150132人目の素数さん:01/09/27 17:35
不変測度の件は。。
151教えて!yoo:01/09/27 17:38
△ABCの辺ABを内分する点をDとする。辺BC,BAを隣合う2辺とする平行四辺形の残りの頂点をE,平行四辺形BDECの対角線の交点をFとするとき、AEベクトルとAFベクトルをABベクトルとACベクトルで表せ。
152教えて!yoo:01/09/27 17:38
△ABCの辺ABを内分する点をDとする。辺BC,BAを隣合う2辺とする平行四辺形の残りの頂点をE,平行四辺形BDECの対角線の交点をFとするとき、AEベクトルとAFベクトルをABベクトルとACベクトルで表せ。
153はなう:01/09/27 17:39
>>121
>>111
小学生よう屁理屈で言うと、これ、
>1匹のカタツムリが、まっすぐに6分間進んでいったとします。
>そのカタツムリを何人かの子どもたちが観察していました。
>子どもたちは相談して、いつも少なくとも一人は観察していました。
>子どもたちはそれぞれ1分間ずつカタツムリを観察しました。
「連続して」1分間とか書いてないから、答えは「限りなく大きい」じゃないかのぅ。
一人の子供が合計1分観察すれば、1分を二回に分けても良いとすると、ってこと。

例えば301人の子供がいてこの子ども達が全員最初の59秒間観察していたとする。で、その間はカタツムリは全く動かなかったとして、
残りの301秒を一人1秒ずつ観察して、その間カタツムリが1秒あたり30cm動いたら題意を満たすよね。で、その時の動いた距離は
30cm*301=90.3mって大きくなる。こんな風にしてけば何人いても居れば居るだけ大きくなるよね。
>>148
Taylor-Maclaurin 展開によると、

e = Σ_[k=0,∞](1^k/k!)

でさぁね?しかし、そもそも何だってことは考えたこともない。
155132人目の素数さん:01/09/27 17:41
曲率の問題です。
曲率Kが0でない定数である平面曲線は、円である。

うーん、なぜだろう??
授業では明らか。
証明できません。
どなたかおねがいします
156148:01/09/27 17:42
>>149
高卒程度ならしっかり分かってます。<微分

>>154
Taylor-Maclaurin 展開ってなんすか?
157はなう:01/09/27 17:47
>>151
BDECは平行四辺形にならんぞい。多分問題写しミス
158三角関数:01/09/27 17:48
sinシータ+sin2シータ+sin3シータ>0
(シータは0度以上180度以下)
>156
微分方程式の初期値問題

y=y(x)
dy/dx = y
y(0)=1

においてy(1)の値をeと定義します。
微分して変わらない形の関数のx=1での値という意味です。

この微分方程式は次のような差分方程式でh→0としたものと思えます。
(y(x+h)-y(x))/h = y(x)

y(x+h)=(1+h)y(x)

x=nhとすると(nは整数)

y(x)=y(nh)= (1+h)^n y(0)=(1+h)^(x/h) y(0) = ((1+h)^(1/h))^x y(0)

(1+h)^(1/h)は高校でやるように h→0とするとeに収束しますので

y(x)=y(0) e^x = e^x
となります。これがeの意味です。

因みに、年金を複利で計算するときなんかにも(1+h)^(1/h)が出てきます。
その時は、年m回に分けて利子が入る場合元本は1年で(1+(i/m))^m倍
なんかになるのですが、m→∞として連続的に利子が繰り込まれ、元本が増えていく
場合などにもeが活躍します。
160はなう:01/09/27 18:06
>>158 しいたをxにしますと。
左辺=sinx+2sinxcosx+4sinxcos^2x-sinx
  =2sinx(cosx+2cos^2x)
だからπ/4<x<π/3でマイナスにならんかのう。わしまたなんか間違ったか??
161148:01/09/27 18:11
>>159
ほうほう。確かに微分方程式の紐を解けばeに収束することは
なんとなく分かりました。ありがとうございました。


でも、2.71・・に収束すること自体が不思議でならないのです。
何故、無理数なのか・・とか。πについても同様です。
他の事例にもよく出現するこの値。どこか謎がありそうで。。。
>>161
>何故、無理数なのか・・とか。πについても同様です。

っていうかさ
有理数みたいな性質のいい、実数の中でも少ないものに収束してたりしたら
その方が怖いんだけど、、、綺麗な比率になるための構造があるってことでしょ?
例えばπが有理数だったら、πの分母を払うように整数倍の拡大を単位円に施せば
円周と、ある正n角形の周長が同じになっちゃったりしてそれこそ奇妙ってものだよ

微分方程式でいろいろな関数を定義できるからこそ、eがいろいろな所に現れるのも
自然な帰結

↓無理数になる証明についてはここを読んでください。
ttp://www1.ocn.ne.jp/~yoshiiz/dvi_1.html
>>156
高卒程度の微分を「しっかりわかってる」やつが、
なんで自然対数の底の定義をしらず、しかも「自然数」とか言ってるの?
わけわからん。
164132人目の素数さん:01/09/27 18:54
e が超越数である事の証明はウェブ上にある?
165132人目の素数さん:01/09/27 18:57
>164
白痴でなければ2つ上のレスくらい読んでくれ・・・
166教えてyoo:01/09/27 19:02
平行四辺形になります!BDECです!
167もったいないお化け:01/09/27 19:04
とか言いながら議論始まっちゃったよ。
私としてはeが無理数である事の証明は、とりあえず受験にもでないし、
議論するんならなら他のスレでやるべきだと思ったんだが、、、
無駄って言ったのは謝ります、すいませんでした。
168164:01/09/27 19:05
>165 スマソ。色々おいてあるんだね。
>>155
パラメータ表示(x(t),y(t))を(x')^2+(y')^2=1...(*)ととる。
曲率一定だからR={(x')^2+(y')^2}/√{(x'')^2+(y'')^2}が一定。
よって(x'')^2+(y'')^2が一定。
(*)よりx'=cosu(t),y'=sinu(t)とおける。
するとx''=-sinu(t)u',y''=cosu(t)u'から(u')^2が一定から
u(t)=At+Bを得てそれから
x=(1/A)cos(At+B)+P,y=(1/A)sin(At+B)+Q
でいけるんではなかろか。自身なsage。
>>155
曲率って、ある曲線の任意の点で、接する半径rの円を考えたときに、
その曲線をもっともよく近似するするようなrに対して1/rで与えられるもの
ってゆーイメージを持ってますがどうか?

曲線がその任意の点で一定の曲率kを取るなら半径1/kの円になるのは自明では?
曲率の正式な定義ってどんなんだっけ?
171もったいないお化け:01/09/27 19:19
>>166
辺BC,BDを隣合う2辺とする平行四辺形ってことか?
それからDは辺ABを t:1-t にないぶんするとしてtを使ってもいいんだよな。
172132人目の素数さん:01/09/27 19:23
>>167
はぁ?
証明自体が転がってるんだから読めば終わり
>167
>とりあえず受験にもでないし、

受験問題のためだけのスレではアリマセン
仕切るのはやめましょう。
>>166
わからん。
>辺BC,BAを隣合う2辺とする平行四辺形の残りの頂点をE
とするのであれば、平行四辺形なのはBAECであって、BDECではないだろう。
DはABを内分する点なんだろ?AD=tAB(0<t<1)なんだよね?
なんでBDECが平行四辺形になりうるんだ。

それでもなるんだと主張するならD=Aってことか?
>>169
すまぬ。いま教科書よんだら曲率の定義まちごうとった。再挑戦。
x'=cosu(t),y'=sinu(t)とおくまではよい。パラメータが
標準助変数だから
1/曲率半径=曲率=|x'y''-x''y'|=|u'|
が一定だから...
以下同じ。
176132人目の素数さん:01/09/27 19:52
155>>
こっちの証明でよいのではないかな?

ベクトルpの曲率k ≡ constant > 0 のとき、pが円周だとすると、
e2は円の中心に向かい、その半径が1/kになるので、
c(s) = p(s) + 1/k・e2
は定点(曲率中心)になるはずである。
実際、sで微分すると、kが定数だから、
c'(s) = p'(s) + 1/k・e2'
= e1 + 1/k・-k・e1
= e1 + 1/k・-k・e1
= 0
となり、c が定点であることが分かる。

この c を使って、
|p(s) - c| = |1/k|
を表すと、pは半径(曲率半径)|1/k|の曲率円であることが分かる。
177はなう:01/09/27 20:30
>>171>>174
そうですよね。って、ことで、>>151さんなんらかの問題間違いです。作った平行四辺形はBDとBCを2辺にするんじゃないでしょうか?
そのときは、BD=tBAとすると、
BE=-tAB-AB+ACより
AE=AB+BE=-tAB+AC
AF=AB+BF=AB+(1/2)BE=(1/2-t)AB+AC (ベクトルです)
>>132
>おいおい・・・R^3で稠密って言葉の意味分かってる?

あなたよりは分かっています。

>閉包とったらR^3を埋め尽くしちゃう場合について聞いてるのか?

そういう事になります。
12の410の例2のgでも稠密になっています。

>じゃぁ何で表現したいの?

「...という条件を満たすg」という形ではなく
「g(x,y)=...」という形で表したいのです。
何を使えば表せるのかは分かりません。
(分かっているなら質問はしない。)

>前スレ>410-411には一例が載ってる筈
>・・・・・・ちゃんと読んでる?

ちゃんと読んでます。
だから12の707で
>答の一部を知りたいわけでもありません。
と書いたんです。
179もったいないお化け:01/09/27 21:11
>>173
いろいろと誤解を生む暴言をはいてしまい、申し訳ありませんでした。
質問をされたふひゅうさんはeを始めて知った方のようだったので、
おそらく高3かなと勝手に思い、先ほどのように発言しました。

またこれらは、私個人の考え方ですので、ほっといてやってください。私も仕切るつもりはありません。
180いまさら、156:01/09/27 21:18
>>163

ドーセ3リュウダイガクザイガクチュウデスヨゥゥゥゥ〜

もっと親切な人が多いと思っていたのに。
残念です。(T_T)

しっかりやってたっていうか、しっかりやってたつもりと言うように
描いておけば良かったですね。すみません。
181いまさら、156:01/09/27 21:20
>>162
確かに多角形と円が同じになるって奇妙ですね。納得しました。
182132人目の素数さん:01/09/27 21:41
∫[C]zcosz/(z-π/2)^3 C:|z-i|=2
∫[C](z^3-2)/(z^2-1) C:|z|=2

の答えはそれぞれ-2πi,2πi
で合ってますか?
183132人目の素数さん:01/09/27 21:56
>182 計算したらそうなった
184132人目の素数さん:01/09/27 22:01
>>183
ありがとう。解答なくしちゃったから困ってたのです。

∫[C](e^z)/(e^z-1) C:|z|=2
これはどうやって解けばいいんでしょう?
185163:01/09/27 22:06
>>いまさら156さん
すまん、すまん。悪気はないんだ。
なんとなく厨房的雰囲気か漂ってたもので。
こっちのほうがよっぽど厨房か。スマソ。

で、高校の教科書は読んでみた?つっても、そんなのとっとかないか。
とりあえず、Web検索でいろいろわかると思うんだよねー。
聞くのはそれからでもいーんじゃないかなぁ?
特に、e=2.71・・・・やπ=3.14・・・・は禿しくがいしゅつなんだよ。

πは円周の長さを直径で割ったら割り切れなかった。

e=2.71・・・・は、自然界において指数関数的に増減する現象を記述するために
y=a^xをいう関数を作って、微分という計算をするときに、
y'=a^xとなるaを探したら、やっぱり中途半端な数字になっちゃった。
それだけのことなんだよ。

その中途半端な数字が、無理数か、超越数かって議論は深い知識と考察が必要。
それで本が書けるんだから、興味があれば本を読みなよ。
こんなとこで聞いてる場合ではない。(藁

ttp://www.ee.t-kougei.ac.jp/tuushin/lecture/math/function/exp/drawE.html

ま、せっかくだから、この辺見てみれ
186111:01/09/27 22:12
>>113 >>121 >>153
レスありがとうございます。ボクも121さんのように考えたんですが
なんかこの問題、読めば読むほど???な部分が多くて。

153さんがおっしゃるように「連続して」1分間とは書いてないんで
読み手によっていくらでも解釈の幅が広がる気がするんです。
この手の文章題が最近の小学生の算数の傾向なんでしょうか。

ご解答ありがとうございました。また息子の宿題でお聞きするかもしれません。
そのときもひとつよろしくお願いします。
187132人目の素数さん:01/09/27 22:16
>>178
>「...という条件を満たすg」という形ではなく
>「g(x,y)=...」という形で表したいのです。
>何を使えば表せるのかは分かりません。
>(分かっているなら質問はしない。)

一価関数なのにほとんど無限多価であるものを構成しろってこと?
188およびでない165:01/09/27 22:22
>>185さん
見てみます。ありがとう。
189132人目の素数さん:01/09/27 22:43
>>184
log(e^z-1) の変化をしらべるか
e^z/(e^z-1)={ze^z/(e^z-1)}/z と考えて 2πi かな
>>184
留数計算する。原点での留数は有理型関数f(z)/g(z)のばあい
f(z)=z^m(a+bz+cz^2+...),g(z)=z^n(p+qz+rz^2+...)
の原点での留数はm≧nなら0、m<nなら(a+bz+cz^2+...)/(p+qz+rz^2+...)
のテーラー展開のn-m-1次の係数。>>184の問題なら
f(z)=1+z+z^2/2+....,g(z)=z(1+z/2+z^2/6+z^3/24+...)
だから(1+z+z^2/2+...)/(1+z/2+z^2/6+z^3/24+...)の1-0-1次の係数を
かんがえればよい。
191132人目の素数さん:01/09/27 22:47
>>189
{ze^z/(e^z-1)}/z
これをどうすればいいんですか??
>>189
かぶすま。
193132人目の素数さん:01/09/27 22:50
>>131
>{(x,y,g(x,y)|x∈R,y∈R}がR^3で稠密でないなら
>g(x,y)=xyとなることは質問する前から分かってますし

yについてg(x,y)が連続の場合g(x,y)=xyに限るという証明は
あったけど、
{(x,y,g(x,y)|x∈R,y∈R}がR^3で稠密でない場合
まで広げた証明ってありましたっけ?

かなりギャップがありますよね?この2つは
194132人目の素数さん:01/09/27 23:11
>>191
z=0 は e^z/(e^z-1) の1位の極で、積分は
2πiRes(0)=2πilim[z→0]{ze^z/(e^z-1)}=2πi
195151です:01/09/27 23:30
すみません。質問文 大間違いしておりました。こうです↓

△ABCの辺ABを3:2に内分する点をDとする。辺BC,BDを隣り合う2辺とする平行四辺形の残りの頂点をE,平行四辺形BDECの対角線の交点をFとするとき、AEベクトルとAFベクトルをABベクトルとACベクトルで表せ。

でした。m(__)m
よろしくお願いします
196132人目の素数さん:01/09/27 23:37

lim(x→a)f(x)=b ならばlim(x→a)(-f(x))=-b となることを
ε-δ論方をつかって証明していただけないでしょうか、
お願い致します


>>187
gは一価関数です。無限多価とかは関係ありません。
(df/dx)(x)=f(x)という条件を満たすfではなくて
f(x)=a・exp(x)(aは定数)というようなものを求めたいんです。

>>193
g(a,b)≠abとなるa,bがあるなら
任意の実数p,q,r任意の正の実数eに対して
有理数s,t,uをうまくとれば
|sa−p|<e
|tb+u−q|<e
|g(sa,tb+u)−r|<e
となるようにできます。
198132人目の素数さん:01/09/27 23:42
>196 がいしゅつ
ε>0 に対し、ある δ>0 があって、|x-a|<δ ⇒ |f(x)-b|<ε。
したがって、この δ に対し、|x-a|<δ ⇒|-f(x)-(-b)|=|f(x)-b|<ε。
199174:01/09/27 23:53
>>195
そしたら>>177で答えが出るだろ。あとは自分で考えれ。
200はなう:01/09/28 00:07
>>195 私の>>177が違ってなければ
AE=(-2/5)AB+AC
AF=(3/5)AB+AC
>>186
最近の小学生にはむしろひねくれた子供になって欲しくないと言う教育方針が明確に出ている気がします。言葉の裏までよみとれってことは減りましたですね〜。

まあかりかりせずにいこうや、みんな(w
201縞栗鼠(シマリス)の親方:01/09/28 00:13
>>196
lim(x→a)f(x)=b とは、
太郎君がどんな小さな数δを持ってきて、狭い範囲(b−δ、b+δ)
の陣地を築いても、その陣地を見たあと、次郎君が、
十分小さなεをもってきて、範囲(a−ε、a+ε)を決めたとき、
その範囲の中のxを持ってくれば、その範囲のf(x)は
全部太郎君が最初に決めた陣地(b−δ、b+δ)
の範囲にはいてしまいます。という意味ですね?
 でこうなっているとき、今の問題は、
「こんどは三郎君が小さなδを持ってきて
(-b-δ、-b+δ)※の陣地を設けたとき、
四郎君がある小さなεを持ってきて、(a−ε、a+ε)
の範囲のどんなxを持ってきてもーf(x)
が全部三郎君の陣地の中の数になるようにできる」
ことを証明しなさい、という問題ですね。
 [答え] 四郎君が次郎君に相談すればよい。
まず三郎君がある小さいδを決め、陣地(b−δ、b+δ)★
を設定します。次郎君は、次のようなεを決めることができる、
と主張する人です。どんなεかというと、「(aーε、a+ε)☆
の範囲の中のxを持ってくればそれで計算したf(x)は全部★
の陣地に入る、」そのようなεです。で、四郎君は、
次郎君に教えてもらって次郎君と同じεを用意すると、
☆の陣地の中のxを持ってくれば、それで計算したf(x)
は全部★のなかに入る(次郎君の保障)。すると、−f(x)は、
★の正負逆の範囲である、(−b−δ、-b+δ)
の範囲に入るはずである。これは※のはんいである。つまり、
四郎君は次郎君のイプシロンをそのまま使えば、
それで計算したーf(x)はすべて※の範囲に入る。
これは証明すべき事実そのものである。
 以上証明終わり。

−−−−I−−−−−I−−−−−−0−−−−−−I−−−−−Iーーーーーー
  −b−δ  −b+δ           b−δ   b+δ
      ※                   ★
202132人目の素数さん:01/09/28 00:17
>201
丁寧な説明は良いのですが、εとδの使い方が慣例と逆です。
教育的配慮をお願いします
203牛蒡の天婦羅:01/09/28 00:27
実数列a(i)(1≦i≦n)が与えられたときに、なぜ
lim[p→∞](納i=1,n]|a(i)|^p)^(1/p)=max(1≦i≦n)|a(i)|
になるんですか?
イメージとしてはぼんやりわかるんですが、数学的(?)には
どのような証明になるのかわかりません。
どうかよろしくお願いします。
204132人目の素数さん:01/09/28 00:28
(1-cos2z)/z
をz=0を中心としてローラン展開せよ。
またz=0はどのような特異点であるか。

わかんねぇよゴルァ!
205132人目の素数さん:01/09/28 00:33
不等式の評価に関する質問です。(nは十分に大きく、zは複素数です)

|R(z)|≦2|f|(n-2)!(n+[|z|]+2)!/(2n-1)!

log(|f|)<2n(log(2)-ε)(εは正定数)
log(n!)=nlog(n)-n+O(log(n)) (g(n)=O(h(n))は|g(n)/h(n)|が有界であることを意味する)
を用いて
log(|R(z)|)<-2nε+O(log(n)) (n→∞)
とできるそうなのですが、O(log(n))できれいにまとめているところが
どうも腑に落ちません。二つ目の式は「スターリングの公式」の書き換えで、
O(log(n)の部分を正直に書いて計算してみましたが、さっぱりまとまりませんでした。
どうかこの評価の仕組みをお教えください。お願いします。
206132人目の素数さん:01/09/28 00:35
>204
-納k=1,∞](-1)^k(2z)^(2k-1)/(2k)! 1位の零点
>>204
(1-cos2z)/z
=1-(1-(2z)^2/2+(2z)^4/24-(2z)^6/720+...)/z
=4z/2-16z^3/24+64z^5/720-...
じゃゴルァ!
z=0では正則じゃゴルァ!
208204:01/09/28 00:42
>>206
詳細きぼーん

宿題聞いてごめんYO!
209 ◆pvySbQO2 :01/09/28 00:43
>>203
m=max_{1≦i≦n}|a(i)|
s=煤Q{1≦i≦n}|a(i)|^p
とすると
m^p≦s≦n・m^p
m≦s^(1/p)≦n^(1/p)・m
lim_{p−>∞}n^(1/p)=1なので
lim_{p−>∞}s^(1/p)=m。
210204:01/09/28 00:43
>>207
わかったぜゴルァ!
211206:01/09/28 00:43
ごめん、2 を掛けといて
212204:01/09/28 00:49
>>211
おけー
213牛蒡の天婦羅:01/09/28 00:56
>>209
なるほど!!激分かりました。
ノルムの勉強しててふと不思議に思ってたんです。
ありがとうございました。
納得。
214高校2年:01/09/28 01:00
今、微分を習っていて
単調に増加するといったら、「f'(x)>=0」なのですかそれとも 「f'(x)>0」ですか?
参考書には、=を含む場合が書いてあったり、注意書きで=を含まないときもある・・・
などとあってはっきりしてません。

あと、「f(x)が増加関数となるように」とか、
「f(x)=x^3-xが増加するxの範囲を求めよ。」とか、
=をつけるかつけないかを教えてください。
>>214
≧だと思うよ
216さる再び:01/09/28 01:27
ある掲示板で√2+√3+√5+√7が無理数である事の証明をたずねているところが
ありました。何かエレガントな証明法(できれば一般化しやすい証明法)を考
えてみたのですが,高校生にも説明できてかつ,エレガントな証明というのがどうも
思いつきません。どなたかわかる方いらっしゃいませんか。ちなみに,高校の範囲を
超えて良ければ,次のような証明を思いつきました。概略で述べます。

まず,±√2±√3±√5±√7なる数字を考えます。ここで±の記号は複号同順とい
うわけではありません。つまり,16個の数字を考えているわけです。(その16個の数は
どれも整数でないですよね)その16個の数すべてを集めて作った集合Aを考えます。集
合Aの任意の異なる2数x,yについて,その和あるいは差をつくると,無理数になる事が
簡単にわかるので,集合Aのなかに有理数があるとしても1つしかない事がすぐに分か
ります。つぎに,集合Aの要素すべてを解とする16次方程式f(x)=0を考えると
係数がすべて整数になることがちょっとしたテクニックでわかります。すると,
f(x)が既約でないなら,その既約因子(こんな言葉ありましたっけ?)を考えれば
Z[x]の既約多項式で,集合Aの要素だけを解に持つものが存在し,しかも1次式でない
ことが分かります。あとは,その既約多項式=0の解を有理数全体がなす体Qに付け加
える事を考えれば,集合Aの中に1つだけ有理数があるということに矛盾するので,結
局集合Aの要素はすべて無理数という事になります。

さて,この証明に不備はあるでしょうか?あったらお教えください。
また,高校生にも説明できるエレガントな方法はありますか?あったらお教えください。
さらに,どうせ,高校の範囲を超えていいならもっと簡単な証明法があるという場合はお教え願えますか。
明日の朝まで掲示板をのぞけないので,レスできなくなりますがよろしくお願いします。
>>202 スマソ 了解です
218もったいないお化けからの提案:01/09/28 02:35
提案なんだけどさ、
大学入試までの問題と大学以上の問題でスレ分けない?
>>216
>√2+√3+√5+√7が無理数である事の証明

>まず,±√2±√3±√5±√7なる数字を考えます。
>集合Aの任意の異なる2数x,yについて,その和あるいは差をつくると,無理数になる事が
>簡単にわかる

例えば符号が逆のもの
√2+√3+√5+√7と-(√2+√3+√5+√7)の和は0差は2(√2+√3+√5+√7)
これが無理数になることが、簡単に分かるのなら証明は終わっている。
>218
スレ違い。
仕切るつもりがないなら
雑談系スレでも行ってくれ
>215
この手はすべて「=」をつけると考えた方が
よいということで結論ですか?

「f(x)が増加関数となるように」も=つきで、
「f(x)=x^3-xが増加するxの範囲を求めよ。」も=つきで
OKですね。
222132人目の素数さん:01/09/28 03:20
公務員板で聞いたけど、イマイチ確証持てず、
その道の専門家の皆さんに聞くべきと、今更気付きました。
お願いします。

正方形の各頂点から、その各頂点を中心に、正方形の1辺の長さを
半径とした、頂点をはさむ2辺による扇形の弧を、それぞれ書いて下さい。
ココまでは大丈夫?
1辺の長さを1としたときの、各弧にはさまれている真中の部分の
面積を求めよ。

こんな感じです。
一度、sinやらcos使って強引に解いてみたものの、
結局正しいのかどうかもよく分からなくなってしまいました。

どうやら、中学〜高校くらいの問題らしいのですが・・。
223132人目の素数さん:01/09/28 03:33
弧は向かい合った頂点からふたつ描くの?それなら
四分円+四部円−正方形
で(π-2)/2.
余談だが,中学入試ではこの手の問題は頻出で,π=3.14のとき
もとの正方形の面積の0.57倍になる。
消防時代はこれを暗記させられたが,これからは正方形の丁度半分か。
鬱だ。
224132人目の素数さん:01/09/28 03:40
>223
弧、4つの頂点から1本ずつ、4本です。
その4本に囲まれた部分なんですけど、
どうですか?
225132人目の素数さん:01/09/28 03:41
積分∫[0,π]log{(sinx)^4+a^4}dx (a>0)の値を求めよ。
226132人目の素数さん:01/09/28 03:43
225だけど、
π(パイ)の表示がおかしい・・・・
227≠216:01/09/28 03:46
>>219
「√2+√3=a と √2-√3=b が共に有理数である」と仮定(1)
すると a+b=2√2=有理数となり矛盾.
ゆえに,(1)が否定され「a,bの片方または両方が無理数」
片方が有理数(他方は無理数)とすれば、a*b=-1=無理数となり矛盾.
ゆえに,a,bともに無理数

・・・これの応用形でどうかな?
228 ◆pvySbQO2 :01/09/28 03:48
>>222
正方形の一つの頂点をAとする。
正方形の向かい合う辺の中点を結んだ線分をp,qとし
Aを中心とした円とp,qの交点をB,C
pとqの交点をDとすれば
求める面積はBCDの4倍になる。
 BCD
=ABC−ABD−ACD
=π/12−((√3−1)/2)×(1/2)
=π/12−(√3−1)/4
これを4倍してπ/3−√3+1。
229132人目の素数さん:01/09/28 03:58
>>224
π/3−√3+1だと思う。
というか、初頭幾何的に簡単にできるが、図が描けない・・・
角度が30°の扇形から、二つの三角形を切り取って、四つ合わせるんだけど。
230132人目の素数さん:01/09/28 03:58
>>228
カブスマです。
231132人目の素数さん:01/09/28 04:23
2桁以上の、全部同じ数字で並ぶ自然数(例えば777777777とか)で
自然数の累乗(例えば23^5とか)になるものはないように思えるのですが、
これは証明されているのでしょうか。

もちろん、1の累乗や11の1乗は除いての話です。
232132人目の素数さん:01/09/28 04:23
質問です。
「空間内の直交変換fによって原点を中心とする球は
自分自身に移されることを証明せよ。」
って問題です。
233222/224:01/09/28 04:24
おおっ!
もう解答が!
まだ良く読んでないからよくわからない
(そもそも理解できるのか?)けど、とりあえず
ありがとうです!
さすが数学板!
「2桁以上」と断っているのだから1の累乗はありませんでした。すみません。
235通りすがりの学生さん。:01/09/28 04:25
積分(高校の数V程度)を使って計算してみました。

簡単のため、正方形の一辺の長さを1とする(aならばa^2倍せよ)
正方形の一つの頂点を原点とし、正方形が第一象限およびx,y軸上におさまるような座標を導入する。
すると題意の弧のうち(0,1)と(1,0)を結ぶ曲線の方程式は
y=(1-x^2)^1/2 (0≦x≦1)
であり、これと直線y=1/2との交点は(√3/2,1/2)となる。
求める面積をSとおくと、対象性よりS/4は
(1-x^2)^1/2 - 1/2 をx=1/2から√3/2までxで積分すれば得られるから、
・・・・・・
S=π/3+1-√3
計算は自信ないです。
236132人目の素数さん:01/09/28 04:31
>>214
>参考書には、=を含む場合が書いてあったり、注意書きで=を含まないときもある・・・
>などとあってはっきりしてません。

大学に入ると習いますが、単調増加には2種類あります。

広義単調増加と狭義単調増加です。
広義単調増加は=が入り、狭義単調増加は=が入りません。

広義単調増加だと、定数関数のようなものも入ってしまいますが、
参考書や教科書に=を含む場合がデフォルトでかかれているなら
=を含めておいていいと思います。
>>233
とりあえず、図を書いてください。
中学生の頃確かに小テストでやった覚えがあります。
同じ問題を
>>232
直交行列の行列式は1ですから。
239132人目の素数さん:01/09/28 05:38
>>227
2つのときはそれでいいとしよう
つまり±√2±√3は全て無理数であることがいえるし√5や√7のときも同様

√2±√3±√5(複号任意)
の4個の数についてどの2つをとっても和と差の少なくとも一方は無理数なので
任意の2数のうち少なくとも一方は無理数
ということは、4個の数の内、有理数は高々1個でなければならない。

なんとかなるかねぇ?
240227:01/09/28 06:03
>>239
227と同じ論法で「(√2+√3)+√5=a と (√2+√3)-√5=b が共に有理数である」と仮定
和 a+bで矛盾するのは同じで、
積 a*b=(√2+√3)^2-5=(2√6+5)-5=有理数となって矛盾
以下同様
241132人目の素数さん:01/09/28 06:57
>>240
>積 a*b=(√2+√3)^2-5=(2√6+5)-5=有理数となって矛盾

それが片方が有理数の時矛盾してないんだよ。
有理数×無理数=無理数になってるだけだから
227の論法そのままではだめ
242227:01/09/28 07:02
>>241 スマん まちがえた
243132人目の素数さん:01/09/28 07:44
>241とその元の問題
一般にQ(√p1,√p2,…,√pn)(p1〜pnは相異なる素数)
はQ上galoisで拡大次数は2^nである事が示せる.(結構めんどいかも?)
つまり、±√p1±√p2±…±√pnがベクトル空間としての基底なので、
全部無理数.
244132人目の素数さん:01/09/28 08:04
-∞から∞までの定積分
∫ exp(-(ax)^2) dx = √(π) / a
の解き方を教えてください。
245なるほどね:01/09/28 08:17
>>111

300cm ということか。
246もったいないお化けからの提案:01/09/28 08:32
>>220
う〜ん。
ほら、板の回りが速すぎて、どれがどの回答なのか分かりにくいし、いちいち上下するのめんどいと思わない?
だから二つに分けたらいいんじゃないかと思ったんだが。
>>243
元の問題を見てください。
そんな解答でよければ苦労しません。
248243:01/09/28 08:39
あ、間違えた。。。
e1√p1+…+en√pn ei=0or1 がベクトル空間の基底です
249132人目の素数さん:01/09/28 08:43
>>246
そういう意見だらけになると邪魔だから他所のスレでやってちょうよ
かちゅで見てる分には上下の必要も殆ど茄子
250132人目の素数さん:01/09/28 08:44
>>248
e1=...=en=0でもいいのか(w
251132人目の素数さん:01/09/28 08:48
>>244
∬ exp{-(x^2)-(y^2)} dxdy  (−∞<x<∞,−∞<y<∞)
= (∫ exp{-(x)^2} dx)^2  (−∞<x<∞).

∬ exp{-(x^2)-(y^2)} dxdy  (−∞<x<∞,−∞<y<∞)
= ∬ exp{-(r^2)} r drdθ  (0<r<∞,0<θ<2π)
=2π∫exp{-(r^2)} r dr  (0<r<∞)


よって
∫ exp{-(x)^2} dx  (−∞<x<∞)
=√π


>>246
これも稼働中だ.
http://cheese.2ch.net/test/read.cgi?bbs=math&key=998671485
更に質問スレが欲しいなら勝手に作っちゃえばいいとおもうが.
作ったらそこでしばらく解答者してないとたぶん認知されない(w
252なし:01/09/28 09:01
>>231

級数を使わない例を挙げて説明しましょう。
例えば、5*999999 / 9 = 5 (10^7 - 1) / 9 となります。
一般に a 10^b = 9 m^n + a とおいて検証してみてください。
253132人目の素数さん:01/09/28 09:28
課題で解の公式の証明やってたんですが、
(x+b/2a)^2=b^2-4ac/4a^2
から、左辺の2乗をとると
右辺の分母が2aになるのかわかりません。
√4a^2って絶対値の2aになるのと違います?
254さる:01/09/28 09:42
>>219
>例えば符号が逆のもの
>√2+√3+√5+√7と-(√2+√3+√5+√7)の和は0差は2(√2+√3+√5+√7)
>これが無理数になることが、簡単に分かるのなら証明は終わっている。
だはは,そだね。でも,これは修正可能範囲だよね。
なんか,いろいろ考えてくれている人達,感謝感謝!!
>>243
へぇーーーーー。まだフォローはしてないけど,なんかよさそう。
あと,やっぱり,高校の範囲での証明は無理かなあ。
255132人目の素数さん:01/09/28 10:09
>>253
>右辺の分母が2aになるのかわかりません。

「右辺の分母が2aになる」と言うよりも
「右辺の分母は2aとしてもいい」と言ったほうがいいです。

b^2 - 4ac > 0 の場合・・・・☆

>√4a^2って絶対値の2aになるのと違います?

そのとおり。√(4a^2)=|2a| です。だから
(x + b/(2a))^2 = (b^2 - 4ac)/(4a^2)
より
x + b/(2a) = √(b^2 - 4ac)/|2a| または -√(b^2 - 4ac)/|2a|
です。従って
x + b/(2a) = √(b^2 - 4ac)/(2a) または -√(b^2 - 4ac)/(2a)
が成り立ちます。(b^2 - 4ac≦0 の場合も同様です。)
256255:01/09/28 10:09
↑こういうところで悩みたくないなら
以下のようにやればいい。

a(x^2) + bx + c = 0
両辺に -c を加える。
a(x^2) + bx = -c
両辺に 4a を掛ける。
4(a^2)(x^2) + 4ab = -4ac
両辺に b^2 を加える。
4(a^2)(x^2) + 4ab + b^2 = b^2 - 4ac
左辺を因数分解する。
(2ax + b)^2 = b^2 - 4ac
両辺の平方根は…
2ax + b = ±√(b^2 - 4ac)
両辺に -b を加える。
2ax = -b ±√(b^2 - 4ac)
両辺に 1/(2a) を掛ける。
x = {-b ±√(b^2 - 4ac)}/(2a)
257174:01/09/28 10:22
>>200
えっと、>>177が間違ってなければ、なんだが
間違ってるような気がするんで確認(^^;

>AE=AB+BE=-tAB+AC
は、いーんだが、
>AF=AB+BF=AB+(1/2)BE=(1/2-t)AB+AC
これは最後、(1/2-t)AB+1/2AC
であるとおもわれ。検算よろしく。

151さんも、ここの答えを鵜呑みにしないで自分で考えてね。
じゃないと、いつまでたっても実力あぷしないよ。
258>250:01/09/28 10:25
1+e1√p1+…en√pn かな?
どちらにしても、√p1+√p2+…+√pn はQ(√p1,…,√pn-1)の元でないから無理数.

こんな解答ではあかんってのは、高校生にも分かるようにってことなんかな?
>>205
log|R|
<2n(log2-ε)
+(n-2)log(n-2)-(n-2)+O(log(n-2))
+(n+C)log(n+C)-(n+C)+O(log(n+C))
−(2n+1)log(2n+1)+(2n+1)+O(log(2n-1))
=n{2log2-2ε+log(n-2)+log(n+C)-2log(2n-1)}+O(logn)...(*)
(O(log(n+k)) の形のラージOは一つにまとまることに注意!)で
2log2-2ε+log(n-2)+log(n+C)-2log(2n-1)
=-2ε+log(n-2)-logn+log(n+C)-logn-2log(2n-1)+2logn-log2
=-2ε+log(1-2/n)+log(1+C/n)-2log(1-1/2n)
=-2ε+O(2/n)+O(1/n)+O(1/2n)
=-2ε+O(1/n)
から
(*)=-2nε+O(1)+O(logn)=-2nε+O(logn)
>>258
おれ>>250さんじゃないけどヨコレス。>>250さんのいいたいのは
e1√p1+…+en√pn ei=0or1 が基底じゃないってことだよ。
たとえばQ(√2,√3,√5)の基底は
1,√2,√3,√5,√(2×3),√(3×5),√(5×2),√(2×3×5)
などをとらないと。しかもこれが基底になることをしめせが題意とおもわれ。
261132人目の素数さん:01/09/28 11:38
>>228,>>229
えと、ごめん、ちょっとおしえて。
扇形ABCの中心角が30度であることはどうやって示せばいいのかな?
それがわかれば、あとは図をみりゃわかるんだけどさ。

なんか、簡単に示せる定理のようなものがあったっけ?
262はなう:01/09/28 11:49
>>257
スマソ。その通り。鬱だシノウ
AE=(-2/5)AB+AC
AF=(3/5)AB+(1/2)AC
です
263132人目の素数さん:01/09/28 12:17
SINθ+SIN2θ+SIN3θ>2(θの範囲は0゜以上180゜以下) 
264132人目の素数さん:01/09/28 12:18
225

積分∫[0,π]log{(sinx)^4+a^4}dx (a>0)の値を求めよ。
265132人目の素数さん:01/09/28 12:24
他人にものを頼むときにはどんなふうな言い方をするべきなのか…
数学やるまえにそっちの勉強しろと言いたくなることがある。
266132人目の素数さん:01/09/28 12:30
4a+9b+c=0 ,2a-3b+2c=0 ,a+b+4c=35
からa b c の値の求め方を教えて下さい。
267266:01/09/28 12:34
すいません。解決しました。
268132人目の素数さん:01/09/28 12:37
現在、離散フーリエ変換を勉強しているのですが、定義にバリエーションがあって混乱しています。
混乱してしまって、間違っているっぽいのがあっても真偽の区別がつきません。
そこで、皆さんの知っている離散フーリエ変換の定義を教えてください。
よろしくおねがいします。
269132人目の素数さん:01/09/28 12:52
>>263
θ→0のとき左辺→0になるので正しくない。
270132人目の素数さん:01/09/28 13:22
SINθ+SIN2θ+SIN3θ>2(θの範囲は0゜以上180゜以下) このθの値をできるだけはやくもとめてくれませんか。お願いします。
271単位ヤバメさん@緊急:01/09/28 13:28

xy"-y'=x^2

yの一般解

お願いします。急いでます。
272>260:01/09/28 13:39
しょうもない解答にレスサンクス
でも、とりあえず、基底は√(p1^e1*p2^e2*…*pn^en) の形ってことやね.
これを示すためには、√pnがQ(√p1,…,√pn-1)に含まれなければよい.
(別に基底を示す必要もないが。。。)
(Kn=Q(√p1,…,√pn)として、[Kn:Kn-1]=2 を示せば、基底の形も分かる)
(↑[L:K]=[L:M][M:K]の証明(多分)と同じこと)
これを示すのは前にも書いたがちとめんどうだが可能.
(帰納法っぽくすれば簡単)
これで一様答えにはなるでしょ?
高校生には説明無理やけど.
273>260:01/09/28 13:50
あと、一応上にも書いてるんですが、
√p1+…+√pnが無理数である事を示せばいいので、
Kn≠Kn-1 が常に成り立つ事を示せば十分です.
一応さっき頑張って考えて基底の部分も示せたつもりですが.
274名無し募集中。。。:01/09/28 14:27
>>270
22.27°<θ<55.34°(w
275132人目の素数さん :01/09/28 14:33
GCD(a,b)=GCD(a+b,LCM(a,b))

証明お願いします。
276222・224:01/09/28 15:01
222で、こんな問題に答えてもらった者です。

正方形の各頂点から、その各頂点を中心に、正方形の1辺の長さを
半径とした、頂点をはさむ2辺による扇形の弧を、それぞれ書いて下さい。
ココまでは大丈夫?
1辺の長さを1としたときの、各弧にはさまれている真中の部分の
面積を求めよ。

その結果、228さん・229さんにこんな解答を頂きました。
お二人とも同じ解なので、数字的にはあってるのだと思うのですが、
なにぶん、盆暗頭なもんで・・。
良く分からず。

228 : ◆pvySbQO2 :01/09/28 03:48
>>222
正方形の一つの頂点をAとする。
正方形の向かい合う辺の中点を結んだ線分をp,qとし
Aを中心とした円とp,qの交点をB,C
pとqの交点をDとすれば
求める面積はBCDの4倍になる。
 BCD
=ABC−ABD−ACD
=π/12−((√3−1)/2)×(1/2)
=π/12−(√3−1)/4
これを4倍してπ/3−√3+1。


229 :132人目の素数さん :01/09/28 03:58
>>224
π/3−√3+1だと思う。
というか、初頭幾何的に簡単にできるが、図が描けない・・・
角度が30°の扇形から、二つの三角形を切り取って、四つ合わせるんだけど。

具体的には、228さんの
ABC−ABD−ACD=
の後の数式に至る過程と、

229さんの
扇形が30度になる説明

がよくわからんのです。
30度〜の方は、その後のレスで、どなたかが
質問されていたようなのですが、
どれがどの質問の答えとして対応しているのかすら
理解できず・・。

親切な方、いたらお願い致します。
277132人目の素数さん:01/09/28 15:03
244です。
251さんありがとうございます。
>>271
y=x^3/3+px^2+q
>>276
そもそも問題文がわからん。こういうこと?

 問題 一辺1の正方形ABCDをとる。A,Cを中心とする半径1の円の
    正方形の内部にふくまれる円弧をとるときこの2円弧で
    かこまれる部分の面積をもとめよ?

かな?
280単位ヤバメさん@緊急:01/09/28 15:31
>>278
実際には,
xy"-(2x+1)y'+(x+1)y=x^2・e^x

y=ze^x
とおいて、zの微分方程式を解けという問題なのです。
そこで、y,y',y"を代入してみた結果が、>>271
xz"-z'=x^2  (分かりやすくするためzをyと書き換えたのです)
という式になったのです。しかし、ここからどうすればいいのかが分かりません。
もしかして、この時点で違うのかもしれないし、、、よろしくお願いしますm(__)m
>>279

A,B,C,Dを中心とする4円弧が囲む領域
282282:01/09/28 15:45
0.1mmの厚さの新聞紙を100回折ると、
厚さはどれくらいになるんですか?
100回折った結果、1平方cmになるためには
どれくらいの広さの新聞紙が必要なんですか??
283さる:01/09/28 15:46
>>216の質問をさせてもらっているものです。ほんと皆さんには感謝してます。
私が厨房なばっかりに,適切にレスできなくて申し訳なく思っております。
今は,>>273
>Kn≠Kn-1 が常に成り立つ事を示せば十分です.
をフォローして見ようと思っています。が,厨房なもんで,これから代数の本を
見ながらでないとできないと思っています。
もし,概略だけでも教えていただけると助かりますが,それでもかなりめんどくさい
かもしれないので,期待はしないで待ってます。
>一応さっき頑張って考えて基底の部分も示せたつもりですが.
も興味ありますね。頑張ってもらって本当にありがとうございます。
こちらもよろしけば,概略だけでも期待しないで待ってます。
>>283
まあ外出の方針でやってもいいけどトレース写像をつかったほうが楽と思われ。
√2+√3+√5+√7=pが有理数とし L=Q(√2,√3,√5,√7)として
tr(L/Q;√2+√3+√5+√7)=0、tr(L/Q;p)=[L:K]p≠0。
ちなみにtr(L/K,x)=納g∈Gal(L/K)]g(x)。
tr(L/Q;√2)=0などはtr(L/Q;√2)=[L:Q(√2)]tr(Q(√2)/Q;√2)=0
からでる。
>>272
[Kn:Kn−1]=2かKn≠Kn−1を示すためには
√pnがKn−1に含まれないことを示さなくてはいけないので前進していない。
>>276
A,Bを中心として
半径の長さをABとしたときの
二つの円の交点をCとすると
ABCは正三角形。
287なし:01/09/28 16:07
>>282
折るたびに厚みが2倍になると仮定すれば

2^100 × 0.1 mm.
= 2^100 × 10^-4 m.
= 126 7650 6002 2822 9401 4967 0320.5376 m.
およそ 1.27 × 10^26 m.

折るたびに面積が 1/2 = 2^(-1) 倍になると仮定すれば、逆算して

1 cm^2 / 2^(-100).
= 1 (m/100)^2 / 2^(-100)
= 126 7650 6002 2822 9401 4967 0320.5376 m^2.
およそ 1.27 × 10^26 m^2.

物理的な量については、理科年表を参照すること。
288132人目の素数さん:01/09/28 16:16
こんかいも参考書から質問です 問題解答ともに参考書からの抜粋です
問題
a, bを正の整数とする。a^2が7桁 ab^3が20桁のときa,bはそれぞれ
何桁か?
---------------------------------------------------------------
 解答 10^6≦a^2<10^7  10^19≦ab^3<10^20 であるので
これらの2式の両辺にそれぞれ底が10の対数をとると
 3≦loga<7/2 …(1)    19≦loga+3logb<20 …(2)

(1)の両辺に-1をかけると

-7/2<-loga≦-3 …(3)     これを(2)に辺へん加えると
     

31/5<logb<17/3           よってaは4けた bは6けた
-------------------------------------------------------------------
ここからが僕の質問です
この解答では (3)を(2)に辺へん加えていますが ここで同値が崩れてしまっていますね
つまり一般に
w>x と y>zが成立しているとき 辺へんくわえたもの
w+y>x+z は w>x かつ y>zと同値でありません
つまり
w>x かつ y>z⇒ w+y>x+z が成り立ちますよね

ってことは(3)を(2)に辺へん加えたけっか ここで同値が崩れてしまい
範囲的には正確ではない範囲が出されてしまうのではないでしょうか?どうでしょうか?
289なし:01/09/28 16:23
>>288
しかし w+y>x+z かつ w>x ならば y>z です。
おいおい。
291261:01/09/28 16:31
>>286
おおー、なるほど!目から鱗。ちっとも気がつかなかったヨ。

で、おいらはいちおー228、229の回答で理解できたんだが、
この問題の質問者(=224さん)は理解できたかな?
もう一度、図を書いてみるといいよ。

>ABC−ABD−ACD=
>の後の数式に至る過程と、

は、ABCは中心角30度、半径1の円弧の面積でπ/12。
ABD、ACDはともに底辺(√3−1)/2、高さ1/2の三角形の面積。
これでわかってもらえないかな?
>>280
xy''-y'=0 ・・・(1)
(1)の解は
y'=cx
y=cx^2/2+d
c,d:定数
xy''-y'=x^2 ・・・(2)
(2)の適当な解を一つ持ってくる。
y=x^3/3
(2)の一般解は特解+(1)の解より、
y=x^3/3+cx^2/2+d
cを2eで置き換えると
y=x^3/3+ex^2+d
293名無し募集中。。。:01/09/28 17:04
>>288

> w>x かつ y>z⇒ w+y>x+z
これは正しい? 間違ってる?
では記号をそれぞれ置き換えて、

-w>-x かつ w+y>x+z⇒-w+(w+y)>-x+(x+z)

は正しい? 間違ってる?
294なし:01/09/28 17:17
>>288
>>289
>>293

任意の a,b,c,d について a>b かつ c>d ならば a+c > b+d は正しい。
なぜなら、a-b>0 かつ c-d>0 ならば a-b + c-d > 0 ではないか。
295名無し募集中。。。:01/09/28 17:33
>>294
いや、>>288はその事は理解しているでしょ。
それを使っただけの解答である事に理解していないから、

「w>x かつ y>z⇔ w+y>x+z
 を使った解答は論理的におかしい」

と言っているんだと思う。
296282:01/09/28 18:18
>287
ありがとうございます。疑問が解けました。
297evi:01/09/28 18:26
>274
大間違い!逝ってよし!
298288:01/09/28 19:06
>295さんへ
そうです ぼくのいいたい事はそれなんです
w>x かつ y>z⇔ w+y>x+zは不成立ですね w>x かつ y>z⇒w+y>x+zはもちろん成立してますがね
解答は w>x かつ y>z⇒w+y>x+zをつかって範囲をだしてますね だから同値関係が成立していない
式をつかって答えをだしてるわけです 普通は 同値が成立していない式でだすと
範囲がずれてくるはずです。なのに そのまま答えにしてるはなぜなのでしょうか?といいたいのです
299名無し募集中。。。:01/09/28 20:13
おいおい。
>>298の言ってる普通って何だ?
どうやら俺は異常らしい。

x=1 かつ y=2 ⇒ x+y=3
は論理的に正しいよな。

x=1 かつ y=2 ⇔ x+y=3
は正しくないよな。
ここまでの話は中学生でも分かっていると思うんだが。
何が引っかかるのかいまいち把握できん。パス>>309
300132人目の素数さん:01/09/28 20:55
>>299
おいおい。
等式と不等式では、根本的にちがうぞ。
>>288君は、必要条件は満たしているが、十分条件を満たしているかどうか
検証していない答えは、答えとしてみとめられん、といっているわけだよ。

そうだろ?

bがすくなくとも6桁であれば題意を満たすことは求めた。
だが、bが5桁や7桁である可能性を否定していないのがおかしい。
そういってるだけだと思うが?
301>285:01/09/28 21:54
だから、√pnがKn-1に含まれない事は示せます.
a1,a2,…,anが体K上代数的な時、K(a1,a2,…,an)=K[a1,a2,…,an]
となります.(そんなに難しくはない)
さて、√pnがKn-1に含まれないことを帰納法で示します.
n=1 のとき明らか.
n≦m のとき成立を仮定する.
 即ち、√piはKi-1に含まれない.
n=m+1 の時 √pm+1∈Q(√p1,…,√pm)=Q[√p1,…,√pm-1][√pm]
になってしまったと仮定する.
      √pm+1=a1+a2√pm と表せる.(a1a2はKm-1の元 上の事より環の元としてよい)
 a1*a2≠0の時、両辺を2乗すると帰納法の仮定に反します.
これをどんどん続けると
√(pm+1/q1*q2*…*qk)=q (q1,…,qkはp1,…,pnのいずれか)
             (qは有理数)
となる.これは2乗すれば、相異なる素数であることに反する.
よって成立.
ほら、めんどくさい。。。これでもはしょってるのに。。。
302132人目の素数さん:01/09/28 22:05
正方形のブロックをn個組み合わせた図形を考えます。
正方形は辺同士で接していて連結です。
n=4の場合はテトリスのブロックで、7通りです。
一般のnで何通りの図形があるかっていうのはどうや
ったら求められるんでしょうか?
教えてください。 
>>301
>>284でいってたようにトレース写像つかったほうがらくだよ。
つまり

 定理 {q_i} を q_iq_j (i≠j) が平方数にならない正の有理数の組とするとき
    {√q_i}はQ上一次独立。

∵)蚤_q√q_i=0 (a_i∈Q)とする。a_1≠0としてよい。√q_1でわって
q_1=1 としてよい。このときL=Q(√q_i)とするとき

 0=tr(L/Q;蚤_i√q_i)=[L:Q]a_1≠0

矛盾。
304288:01/09/28 22:59
>300さん そのとうりよ うまく説明してくれてありがと
僕の説明わかりにくですかね
みんな誤解してるみたいですね
>>301,>>303
訂正と補足。a_1≠0と仮定して矛盾をみちびけばよい。だ。
正の有理数じゃなくてもいいね。0でなければ。
ちなみにこれからあと少しがんばるとGal(L/K)がZ/2Zの有限直和
になるまでいえるね。本文ではそこまでとわれてないけど。
>>305
ごめん。また変なこと書いた。Gal(L/K)がZ/2Zの有限直和は
すぐいえるんだった。ゴメソ。
307亀レス:01/09/28 23:09
>>150= >>134
不変測度とは何か(与えられた)変換に対して不変な測度
ということ
変換というのは例えば実数上の測度ならば
実数から実数への変換という意味

だから何か変換が与えられていないと
不変測度って何よ
と聞かれても答えられない
308132人目の素数さん:01/09/28 23:09
>bがすくなくとも6桁であれば題意を満たすことは求めた。
>だが、bが5桁や7桁である可能性を否定していないのがおかしい。
>そういってるだけだと思うが?

はぁ?逆じゃねえの?
a^2が7桁、ab^3が20桁ならば、aは4桁、bは6桁を示したんだろ。
問題は、実際、そんなa,bが存在するかなんだろ。
309132人目の素数さん:01/09/28 23:16
数学オリンピックの問題で
1^2001+2^2001+3^2001+....+2001^2001を13で割るときの余りを求めよ
で、答えは0なんですが求め方が分かりません
コンピューターを使わずに解ける方法を教えてください
310132人目の素数さん:01/09/28 23:22
フェルマーの小定理
a≡0でなければa^12≡1(mod 13)
を使えばすぐできるでしょ。
>>309
i=2002-i(mod 13)だから
1^2001+2^2001+3^2001+....+2001^2001
+1^2001+2^2001+3^2001+....+2001^2001
=1^2001+2^2001+3^2001+....+2001^2001
2001^2001+2000^2001...+1^2001
≡1^2001+2^2001+3^2001+....+2001^2001
+(-1)^2001+(-2)^2001+3^2001+....+(-2001)^2001
≡0+0+...+0
=0 (mod 13)
312132人目の素数さん:01/09/28 23:23
>>309
フェルマーの小定理しってる?。
xがpと互いに素なとき、X^p≡X mod p ってやつ。
それを使うと、(mod p の計算で)すぐにできるよ。
2001≡12 mod 13 に注意。
>>310
かぶった。そっちのほうが正攻法だね。
314お節介:01/09/28 23:26
>>218 = >>246
もったいないお化けさま

雑談スレ2
http://cheese.2ch.net/test/read.cgi?bbs=math&key=989344065
の 365 にコピペしておきました

私はあまりこの板に来れないのであとをよろしく
315132人目の素数さん:01/09/28 23:35
D:n→n+1なる線形変換
I:恒等変換とする。
kはk∈Rをみたす定数、nは任意の自然数である
このとき((D^2)-2D-kI){A_n}={0}で定まる数列{A_n}について
geometric progressionsを満たさないが
{A_3n}についてはgeometric progressionsになるようなkを求めよ。
ただし初期条件はA_0=-1、A_1=2とせよ

これを一題お願いいたします
316132人目の素数さん:01/09/28 23:39
「球面上で、任意の2点間を結ぶ最短経路はその2点を通る大円(の弧)である」
これの証明が出来ません。
少なくとも高校レベルまでで証明したいのですが…
お願いします。
317132人目の素数さん:01/09/28 23:53
>>310=>>312
答えていただいてありがとうございます。
フェルマーの小定理を知らないので
できればそれを使わないで解きたいのですが
もし無理だったらすみません
>>317
S(n)=Σ[k=1,n]k^rとする(rは自然数)。このときS(n)はr+1次の多項式。
S(0)=0、S(-1)=S(0)-0^r=0よりS(n)はn(n+1)で割り切れる(因数定理)。
したがって与式は2002で割り切れる。したがって13で割り切れる
>>318
n(n+1)/2はn(n+1)で割り切れるけど
2001×2002/2は2001×2002で割り切れないよ。
320はなう:01/09/29 00:33
>>317
311さん風にやればぁ?
そもそも、1から順にmod13では
1,2,3,4,5,6,7,8,9,10,11,12,0,1,2,3,4,5,6,・・・12なんで、
mod13とると1から12が154回繰り返されてることがわかる。
で、一セットの1〜12をそれぞれ2001乗すると、
1^2001+2^2001+・・・12^2001
≡1^2001+2^2001+・・・6^2001+(-6)^2001+(-5)^2001+・・・(-1)^2001
≡0だから
それが何セットあっても0ってこと。
321はなう:01/09/29 00:35
>>270>>302、結構むずいね。わしゃぁ明日も仕事なので寝るが起きたら解けていることを切に願いますの。
322:132人目の素数さん :01/09/29 00:53
積分の宿題で悩んでしまって。良かったら御教授ください。
問題は、
[0,1]×[0,1]上の関数
f(x,y)={x x¬∈{1/2^1|n=1,2,3,4...}の時
     {0 x∈{1/2^1|n=1,2,3,4...}の時
は、[0,1]×[0,1]上で、そのRiemannにおける下積分をもとめよ。
なのですが、どうでしょうか?
よろしくお願いします
323132人目の素数さん:01/09/29 01:12
>>205に答えてくださった>>259さんへ。
どうもありがとうございました。(感謝)^2。
324132人目の素数さん:01/09/29 01:16
>>311 >>320
オレが数学オリンピックの審査員だったら減点するね。
n≡2002-n (mod13) はいいんだけど、
n^m+(-n)^m≡0 (mod13) が 常に成り立つこととしてるもの。
言葉(または考え)が足りないので、そう受取られてもしかたない?
325はなう:01/09/29 01:22
>>324
ほい。スマソ。まあ、もちろんmが奇数ならってことです。
>>324
n≡2002−n(mod.13)は間違い。
n≡n−2002(mod.13)が正しい。
mが奇数のときn^m+(−n)^m=0は当たり前。
327132人目の素数さん:01/09/29 02:47
あまり数学ぽくない質問ゴメンなさい。
日本(1億2000万人)で6年間に人口10万人中120人自殺者が出るとして
日本のある地域6万人の場所で自殺者数が6年間で16人であることは、
どの程度の稀少さなのでしょうか?
地域による自殺のし易さは均一として考えていただいて結構です。
それ以外で何か必要な情報があれば『こうゆうモデルで考えた』と勝手に想定
していただいて結構です。

よろしくお願いします。
328231:01/09/29 04:48
>>252
ありがとうございました。続きを自分でやってみます。

与式 a 10^b = 9 m^n + a …(1) (a は1桁の自然数、b,m,n は2以上の自然数) において、
m = 10p + q (p は自然数、q は1桁の負でない整数) とする。

q = 0 のとき、a を満たす数がないため不適。

q ≠ 0 のとき、q の値により、10r + s = 9 (10p + q)^n (r は自然数、s は1桁の自然数) を満たす
s の取りうる値は次のようになる。
q = 1 → s = 9
q = 2 → s = 2,4,6,8
q = 3 → s = 1,3,7,9
q = 4 → s = 4,6
q = 5 → s = 5
q = 6 → s = 4
q = 7 → s = 1,3,7,9
q = 8 → s = 2,4,6,8
q = 9 → s = 1,9
329231:01/09/29 04:50
a 10^b は10の倍数であることから、a = 10 - s となる。
q の値により、a の取りうる値は次のようになる。
q = 1 → a = 1
q = 2 → a = 2,4,6,8
q = 3 → a = 1,3,7,9
q = 4 → a = 4,6
q = 5 → a = 5
q = 6 → a = 6
q = 7 → a = 1,3,7,9
q = 8 → a = 2,4,6,8
q = 9 → a = 1,9

与式 (1) を変形すると a 10^b + a = 9 (10p + q)^n …(2) となる。
このとき右辺は9の倍数なので、9 (10p + q)^n = 9t (t は自然数) と置くことができる。

また左辺は
10^b = (9 + 1)^b = 9^b + 9^(b - 1) + … + 9^2 + 9 + 1
   = 9 (9^(b - 1) + 9^(b - 2)+ … + 9 + 1) + 1
と変形できるので、10^b = 9u + 1 (u は自然数) と置くことができる。

すると与式 (2) は次のように変形できる。
a (9u + 1) + a = 9t
9ua + 2a = 9t
2a = 9 (t - ua) …(3)

このとき右辺は9の倍数を表している。したがって、条件より a = 9 となる。


ここまで出来ました。続きはこれから考えます。
330名無しさん:01/09/29 05:56
ワルラスの一般均衡論は循環論であったマルクス経済の労働価値説に
どのような影響をあたえたのでしょうか?
331231:01/09/29 06:11
>>329の続き
a = 9 のとき、q の取りうる値は q = 3,7,9 となる。

m = 10p + 3 のとき a = 9 すなわち s = 1 となるためには n = 4v (v は負でない整数)
であることが必要である。…(A)
m = 10p + 7 のとき s = 1 となるためには n = 4v (v は負でない整数)
であることが必要である。…(B)
m = 10p + 9 のとき s = 1 となるためには n = 2v (v は負でない整数)
であることが必要である。…(C)

(A),(B),(C) のいずれも、n は偶数である。


どうも奇数の偶数乗は十の位が偶数にしかならないようなので、
m^n が 999…99のように十の位が奇数になることはない、
したがって題意を満たす m^n は存在しない…となりそうです。

あとは「奇数の偶数乗は十の位が偶数」の証明ですね。
332244:01/09/29 06:52
244
{
-∞から∞までの定積分
∫ exp(-(ax)^2) dx = √(π) / a
の解き方を教えてください。
}

って書いたものですけど、

251
{
∬ exp{-(x^2)-(y^2)} dxdy  (−∞<x<∞,−∞<y<∞)
= (∫ exp{-(x)^2} dx)^2  (−∞<x<∞).

∬ exp{-(x^2)-(y^2)} dxdy  (−∞<x<∞,−∞<y<∞)
= ∬ exp{-(r^2)} r drdθ  (0<r<∞,0<θ<2π)
=2π∫exp{-(r^2)} r dr  (0<r<∞)


よって
∫ exp{-(x)^2} dx  (−∞<x<∞)
√π
}

って答えてもらえたのですが、
実際紙に書いてやってみたら、さっぱりわかりませんでした。
検索しようにも、名前がついてる公式なのかすらわかんなくて
もだえとります。
どなたか、
解説か,別解か,検索で引っかかりそうな名前(公式,関連名など)
を教えてください。
お願いします。
333 ◆pvySbQO2 :01/09/29 07:10
>>332
y=|a|xと変数変換して
∫_(−∞,∞)exp(−x^2)dx=√π
を使えば
∫_(−∞,∞)exp(−(ax)^2)dx=√π/|a|
がでます。
334単位ヤバメさん@緊急:01/09/29 07:59
>>292
何とかできそうです。
335132人目の素数さん:01/09/29 10:07
さくらスレ分割問題
>>218 >>220 >>246 >>249 >>251

を雑談スレ2

雑談はここに書け!【2】
http://cheese.2ch.net/test/read.cgi?bbs=math&key=989344065

に移動しました(365以下)

いろいろご意見をお持ちと思います
さくらスレ13に立ち寄られたかたは
お帰りの際に雑談スレ2にもお立ち寄り下さって
論点をお寄せいただければ
と思います
336300:01/09/29 10:08
なんかさー、連立不等式を理解してない子供がいっぱいいるんですが。
とりあえず、>>288の疑問はそのとおりで、間違ってるのは参考書。
参考書は必要条件は求めたが、十分条件にはなってない。
連立不等式はグラフを書いて領域で答えなきゃ。
この問題は整数問題に帰着するので、グラフ上の格子点で求めなきゃだめよ。

さて、まず>>289
>しかし w+y>x+z かつ w>x ならば y>z です。
じゃ、反例。W=100、y=1、x=1、z=10としよう。w+y>x+zかつw>xだが、
y<zだぞ?どうしてくれる?ネタだっつーならいーけどさ。
まぁ、>>290がつっこみだと気がついてないならしかたないが。

>>293
>-w>-x かつ w+y>x+z⇒-w+(w+y)>-x+(x+z)
これを変形しても、
w<x かつ w+y>x+z ⇒ y>z にしかなりませんが?
不等号の向きに注意。

>>294
>任意の a,b,c,d について a>b かつ c>d ならば a+c > b+d は正しい。
>なぜなら、a-b>0 かつ c-d>0 ならば a-b + c-d > 0 ではないか。

だから、それは必要条件は満たしてるという意味で正しい。
が、十分条件について言及されてない。

>>298
せっかくなんで、「必要」「十分」ってことばを覚えましょう。
同値云々っていうと、混乱するひとが出てくるようなんで。

>>299
そうそう。その論理的に正しくない答えを参考書が採用してるからひっかかってんでしょ?

>>308
>はぁ?逆じゃねえの?
逆じゃないよ。きみが書いていることと同じ事を俺は言ってるじゃん。

>a^2が7桁、ab^3が20桁ならば、aは4桁、bは6桁を示したんだろ。

それは必要条件。十分条件じゃない。

>問題は、実際、そんなa,bが存在するかなんだろ。

そりゃ、存在するんじゃない?aが4桁であることとbが6桁であることに矛盾がなければ。
問題は、bの範囲はaを動かすとそれ以外に広がり得る。ってことなんだよ。
これはグラフを書いて、視覚的に求めて見ればわかる。

連立不等式の解きかたって、中学でやるだろー?
337132人目の素数さん:01/09/29 10:21
322番もおねがいします
338300:01/09/29 10:23
んで、ちゃんとした答えを求めるにはだ。
3≦loga<7/2 …(1)
19≦loga+3logb<20 …(2)
ここで、loga=x、logb=yとおくね。
(2)より、
19≦x+3y ⇒ y≧-1/3x+19/3
x+3y<20 ⇒ y<-1/3x+20/3
これをまずxy平面状のグラフに書く。
(1)より3≦x<7/2なので、x=3、x=7/2の直線も追加。
これで平行四辺形ができるんで、このなかの格子点の座標が、
求める、題意を満たすx、yの組み合わせとなる。

グラフ書くのめんどうなので、あとは各人で検証のこと。
もしかすると、結局aは4桁、bは6桁って答えしか出てこないだろう。
(つーか、たぶんそうだろう)が、その求めるプロセスがぜんぜん違うからね。
339132人目の素数さん:01/09/29 11:02
>>245

遅レスですまんが

> >>111
>
>300cm ということか。

なんでこうなるの?
解法が知りたい。
340132人目の素数さん:01/09/29 11:16
>>231 231さんへ >>331の時点で
>あとは「奇数の偶数乗は十の位が偶数」の証明ですね。
となって、すぐに証明終了となると思ってましたが(表計算ソフトでコピペすると反例が無いことがすぐ判った)、
まだみたい。

考えてみると、累乗される数mの「下2桁」のみの考察によって
元の予想が肯定的に証明されそうなところが、まず引っかかりました。
気になって証明を読んでみたんですが、a (9u + 1) + a = 9t が勘違いで、
正しくはa (9u + 1) - a = 9t ですよね。
これでは aの値に関わらず左辺は9の倍数だし、そもそも等号が成り立つように-aが右辺にあったんですよね。

途中なのにわるいけど、現行の方針では証明できないとおもったんですが・・・。
341工房:01/09/29 11:33
すみません。別スレで書いてあったこの問題
がどうしてもできません。だれか願いします。

>次の式を実数係数の因数に分解せよ。

>X^4 - 4*X^3 + 2*X^2 + 4*X + 4
342132人目の素数さん:01/09/29 11:51
>>341
くだらんスレの413に解等を書いておきました。
できれば、因数分解などはくだらんスレに書かれると、さくらスレの負担が
軽くなると思われます・・・
343工房:01/09/29 14:10
>>342
わかりました。
ありがとうございます。
344 ◆pvySbQO2 :01/09/29 14:35
>>338
a^2が7桁でab^3が20桁=>aは4桁でbは6桁
であることは >>288 の解答で出ています。
ネタでなければ、300は本当の馬鹿かもしれん。
346132人目の素数さん:01/09/29 14:47
>>345
>>344が理解できなければ、ほんとにヴァカかもな(w
347 ◆pvySbQO2 :01/09/29 15:00
>>338
求めるのは格子点ではありません。
348132人目の素数さん:01/09/29 15:38
誰か>>225の問題解いて。
>>225
与式=[xlog(sin^4 x +a^4)]_0^π -4∫[0,π](x sin^3 x cos x)/(sin^4 x + a^4) dx
=4π log a -∫[0,π](x sin^3 x cos x)/(sin^4 x + a^4) dx

ここで、第2項の積分は、x=π-tと置換すれば、

∫[0,π](x sin^3 x cos x)/(sin^4 x + a^4) dx
=∫[0,π]{(π-t) sin^3 t cos t}/(sin^4 t+ a^4) dt
(tをxと書き直して)
=∫[0,π]{(π-x) sin^3 x cos x}/(sin^4 x+ a^4) dt
この等式の最初と最後の項を足して2で割れば、
∫[0,π](x sin^3 x cos x)/(sin^4 x + a^4) dx
=π/2∫[0,π](sin^3 x cos x)/(sin^4 x + a^4) dx
となるが、最後の定積分は、u=sin x と置換すれば、積分区間を見て値が0となることが見える。

従って、与式=4π log a (答)
350132人目の素数さん:01/09/29 16:36
348です。
>>349
>>∫[0,π](x sin^3 x cos x)/(sin^4 x + a^4) dx
>>=π/2∫[0,π](sin^3 x cos x)/(sin^4 x + a^4) dx

cos(π-t)=-costとなってそのようにはならないと思うですが・・・・
351132人目の素数さん:01/09/29 16:41
2cos(2π/7)=(-1+(7(2+ω))^(1/3)+(7(2+ω^2))^(1/3))/3
の証明ってどうやるの?
>>351
x=2π/7のとき

   sin3x+sin4x=0
⇔ sinx{8(cosx)^3+4(cosx)^2-4(cosx)-1}=0
⇔ t^3+t^2-2t-1=0 , t=2cosx=2cos(2π/7)>0

同値変形いいかげん。適当に修正して。
あとは解の公式?俺はここでギブ。
353Yoshi:01/09/29 19:30
"f(x)=0 が x=a を3重解に持つ"→"f'(x)=0 は x=a を2重解に持つ"
のはどうして?誰か一般化して。
354132人目の素数さん:01/09/29 19:38
>353
f(x)=(x-a)^3*g(x)
355名無しゲノムのクローンさん:01/09/29 19:40
>>353
f(x)=P(x)(x-3)^3=0
df(x)/dx=3P(x)(x-3)^2+P'(x)(x-3)^3
={3P(x)+P'(x)(x-3)}(x-3)^2
Q.E.D
356ちむ氏の使途:01/09/29 19:59
ジョーカー一枚を含む一組53枚のトランプの中から、3枚を選ぶとき、
3枚の中にジョーカーを含む選び方は何通りあるか。

a,a,a,b,b,cの6文字を一列に並べるとき、全部で何通りの
並び方があるか。

1,1,2,2,3,3,3,の7個の数字を一列に並べてできる7桁の自然数は、
全部で何通りか。

ある鉄道には、20の駅がある。この鉄道で、発駅、着駅、を明記した切符を
つくるとすると、何通りの切符ができるか。

数学板のみなさん、どのようにC、Pをすかえば解けるかがわかりません。
解説、お願いします。また、公式があれば、紹介してください。
集合と位相の問題なんですけど、教えてください。

Xをコンパクトな距離空間、f:X→Xを連続写像とするとき
Y⊂Xで、f(Y)=Yかつ、YはXの閉部分集合となり、
Yのどんな空でない真部分集合Y'⊂Y(Y'≠Y)もこれらの性質を持たないようなもの
(極小集合)が存在することをZornの補題を用いて示せ。
358132人目の素数さん:01/09/29 20:47
>>356

C(52,2) 通り

6!/(3!・2!) 通り

7!/(2!・2!・3!) 通り

P(20,2) 通り
359132人目の素数さん:01/09/29 20:54
この微分方程式の解き方を教えて下さい。
fo(t)=1 (tは[0,T] T<∞)
d/dt(f_(i+1)(t)=−f_i(t)f_(i+1)(t)
f_i(0)=1   (i=0,1,・・・)
としたとき
lim f_i(t)はどんな関数になりますか?
i→∞
360132人目の素数さん:01/09/29 21:41
231>>340
>>あとは「奇数の偶数乗は十の位が偶数」の証明ですね。
>となって、すぐに証明終了となると思ってましたが(表計算ソフトでコピペすると反例が無いことがすぐ判った)、
>まだみたい。

数学的帰納法とか試してみたけど、なかなかうまくいかないの。

>気になって証明を読んでみたんですが、a (9u + 1) + a = 9t が勘違いで、
>正しくはa (9u + 1) - a = 9t ですよね。
>これでは aの値に関わらず左辺は9の倍数だし、そもそも等号が成り立つように-aが右辺にあったんですよね。

そうでしたね。(^^;
また証明のやり直しか…。
231の与式をまとめてみると
与式 (1) a 10^b = 9 m^n + a
与式 (2) a 10^b + a = 9 (10p + q)^n
与式 (3) 2a = 9 (t - ua)

だから、確かに (2) 以降が変でした。

しかも>>329
>また左辺は
> 10^b = (9 + 1)^b = 9^b + 9^(b - 1) + … + 9^2 + 9 + 1
>   = 9 (9^(b - 1) + 9^(b - 2)+ … + 9 + 1) + 1
> と変形できるので、10^b = 9u + 1 (u は自然数) と置くことができる。
もおかしかったですね。
>>361
式の変形はおかしくても、
>10^b = 9u + 1 (u は自然数) と置くことができる。
ここは合っていたようです。念のため。
363132人目の素数さん:01/09/30 00:16
本当に322番お願いします。
364231:01/09/30 01:11
仕切りなおしてみます。

与式 (1) … a 10^b = 9 m^n + a (a は1桁の自然数、b,m,n は2以上の自然数)

左辺が10の倍数であるので右辺も10の倍数である。

m = 10p + q , m^n = 10r + s (p , r は負でない整数、q , s は1桁の自然数)
の形に表したとき、q および n に対応する s および a の値は次の表のようになる。

q = 1 → s = 1 → a = 9

q = 2 & n = 4t + 1 → s = 2 → a = 8
q = 2 & n = 4t + 2 → s = 4 → a = 6
q = 2 & n = 4t + 3 → s = 8 → a = 2
q = 2 & n = 4t → s = 6 → a = 4

q = 3 & n = 4t + 1 → s = 3 → a = 7
q = 3 & n = 4t + 2 → s = 9 → a = 1
q = 3 & n = 4t + 3 → s = 7 → a = 3
q = 3 & n = 4t → s = 1 → a = 9

q = 4 & n = 2t + 1 → s = 4 → a = 6
q = 4 & n = 2t → s = 6 → a = 4

q = 5 → s = 5 → a = 5

q = 6 → s = 6 → a = 4
365231:01/09/30 01:11
q = 7 & n = 4t + 1 → s = 7 → a = 3
q = 7 & n = 4t + 2 → s = 9 → a = 1
q = 7 & n = 4t + 3 → s = 3 → a = 7
q = 7 & n = 4t → s = 1 → a = 9

q = 8 & n = 4t + 1 → s = 8 → a = 2
q = 8 & n = 4t + 2 → s = 4 → a = 6
q = 8 & n = 4t + 3 → s = 2 → a = 8
q = 8 & n = 4t → s = 6 → a = 4

q = 9 & n = 2t + 1 → s = 9 → a = 1
q = 9 & n = 2t → s = 1 → a = 9

(但し、t は自然数)

ここで、与式 (1) を変形および代入して得られる式
a (10^b - 1) / 9 = 10r + s … (2)
の左辺は全ての位が a である自然数を表しており、その一の位も当然 a である。

したがって a = s である。
また、上の対応表のうち、a = s となるのは q = 5 のときのみである。

よって、与式 (1) を満たす m の値は m = 10p + 5 のように表すことができる。
366231:01/09/30 01:33
a = 5 および m = 10p + 5 を与式 (1) に代入すると、

5 10^b = 9 (10p + 5)^n + 5
5 10^b - 5 = 9 (10p + 5)^n
5 (10^b - 1) = 9 (10p + 5)^n
5 (10^b - 1) = 9 (5 (2p + 1)^n)
5 (10^b - 1) = 9 (5^n) ((2p + 1)^n)
10^b - 1 = 9 (5^(n - 1)) ((2p + 1)^n)
(5^b) (2^b) - 1 = 9 (5^(n - 1)) ((2p + 1)^n) … (3)

このとき、左辺は5の倍数から1を引いた数を表している。
一方、右辺が5の倍数でないのは n = 1 のときのみである。
これは条件を満たさない。

よって与式 (1) は成立せず、したがって、2桁以上で全ての位が同じ数字である自然数は
自然数の累乗とはなりえない。


これでいいのかな?
367231:01/09/30 01:40
>>366
>一方、右辺が5の倍数でないのは n = 1 のときのみである。
>これは条件を満たさない。

一方、n は2以上の自然数なので、右辺は常に5の倍数になる。
これは矛盾である。


このように訂正します。
368 ◆pvySbQO2 :01/09/30 01:45
>>364
a=sですよ。
369 ◆pvySbQO2 :01/09/30 02:40
>>315
DはA(n)(0≦n)をA(n+1)(0≦n)に
するものだと思うのでそうだとして書きます。
そうするとA(n)(0≦n)は
A(0)=−1
A(1)=2
A(n+2)−2A(n+1)−kA(n)=0(0≦n)
A(n)(0≦n)は等比数列ではない。
A(3n)(0≦n)は等比数列である。
となります。

xについての方程式x^2−2x−k=0が重根を持つとき
k=−1になる。
このときA(n)(0≦n)は等差数列なのでA(n)=3n−1となるが
9n−1(0≦n)は等比数列ではないので条件に合わない。

xについての方程式x^2−2x−k=0が重根を持たないとき
根をb,dとするとA(n)=a・b^n+c・d^nと表される。
A(n)(0≦n)は等比数列ではないのでac≠0となる。

B(n)(0≦n)が等比数列。
<=>
B(n)B(n+2)=B(n+1)^2(0≦n)。
なので
A(n)(0≦n)は等比数列ではない。<=>b≠d。
A(3n)(0≦n)は等比数列である。<=>b^3=d^3。
となる。

x^3=(x^2−2x−k)(x+2)+(k+4)x+2k
とb≠dとb^3=d^3からk=−4となる。
このとき
A(n)=2^(n+1)・cos((n−2)π/3)
となるので条件を満たす。

よって求めるkは−4。
370ちむ氏の使途:01/09/30 11:36
356だけど、途中式もかいて。わかんない。
371132人目の素数さん:01/09/30 11:37
教えてください。

V:有限次元実線形空間
とした時に、教科書ではEnd(V)を
「linear maps from V to itself」
と定義してあったのですが、これはHom(V,V)の定義のような気がするのですが
これで良いのでしょうか?
>>371
そだよ。
>>370
>356だけど、途中式もかいて。わかんない。

教科書や参考書見るなりして、ちょっとは自分で考えろよ。
>>372
有難う御座います。
375132人目の素数さん:01/09/30 14:18
1
関数f(x)= ----------- -ax が極値をもつように定数aの値の
1+e^-px
範囲を求めよ。ただし、pは正の定数で、eは自然対数の底である。
376132人目の素数さん:01/09/30 14:21
,,,,,,,,,,,,,,1
関数f(x)= ----------- -ax が極値をもつように定数aの値の
,,,,,,,,,,,,1+e^-px
範囲を求めよ。ただし、pは正の定数で、eは自然対数の底である。
,は関係ありませんので無視してください。
377132人目の素数さん:01/09/30 14:23
,,,,,,,,,,,,,,,,,,,,,,,,,,1
関数f(x)= ----------- -ax が極値をもつように定数aの値の
,,,,,,,,,,,,,,,,,,,,,,,,1+e^-px
範囲を求めよ。ただし、pは正の定数で、eは自然対数の底である。
,は関係ありませんので無視してください。 何度もすみません。
378132人目の素数さん:01/09/30 14:37
フェルマーの最終定理が解けません
だれかおしえて
379>378:01/09/30 14:39
あははは。
これでいい??
380132人目の素数さん:01/09/30 15:10
>>377
f'(x)=0 が解で、その前後でf'(x)の符号が変わるものがあればよい。

f(x)を微分すると、(以下e^(-px) = T とおく。T>0 である。)
 f'(x) = pT/(1+T)^2 - a
なのでまずa>0 であることが必要(でないとつねにf'(x)>0)。
さらに
 f'(x) =-{aT^2 + (2a-p)T + a }/(1+T)^2
となるので、あとは、(a>0のもとで)

 Tの2次方程式aT^2 + (2a-p)T + a =0 が、
 T>0 の範囲に「重解でない」解をもつための条件

を求めればよい。
381埼玉県立K高校:01/09/30 16:52
分数の計算方法と小数の計算の仕方がわかりません教えてくらはい。
382132人目の素数さん:01/09/30 17:13
方程式z^5-1+3i=0を解け。但し、iは虚数関数である。
解けそうで解けません。教えてください。
383132人目の素数さん:01/09/30 18:32
4元1次方程式を作り、解け。検算もせよ。

っていう問題が出たんだけど全くわかりません。
どなたか教えていただけないでしょうか?
384ちむ氏の使途:01/09/30 18:53
教えてよ。調べたけど、載ってないの。
ちなみに356だよ。
お願いだよ〜。質問板でしょ。
3851:01/09/30 19:13
その画像ならココ
http://www.f2.dion.ne.jp/~impact14/
386132人目の素数さん:01/09/30 20:07
>>383

『数学の部屋』の掲示板に同じ質問が出てるよ。
387 ◆pvySbQO2 :01/09/30 20:07
>>111
>>339
s(0)=0とする。
s(i)で観察している子のうち最後に観察しはじめた子が
観察を始めた時刻をt(i)としs(i+1)=t(i)+1とする。
するとs(n)=6となるnがある。
t(0)=0でt(i)+1<t(i+2)なので
kを正の整数とするとk<t(2k)となる。
もし10<nとすると
5<t(10)≦t(n−1)=s(n)−1=5
となってしまうのでn≦10となる。
 カタツムリの進んだ距離
=煤Q{0≦i<n}[s(i),s(i+1)]に進んだ距離
≦煤Q{0≦i<n}30cm
=30cm×n
≦300cm
なのでカタツムリの進んだ距離は300cm以下になり
4/9分間休んで1/9分間に30cm進むをくりかえしたとし
10人の子が10個の
4/9分間休んで1/9分間に30cm進んで4/9分間休むというところを
観察していたとすれば問題の条件を満たし
カタツムリは300cm進んでいる。
よってカタツムリの進んだ距離の最長は300cm。
> 384

>>358 に即答してあるぞ。文句言うまえにちゃんと読めよ。
389132人目の素数さん :01/09/30 20:39
322です。
やっぱり、解けません。
上限はできたのですが、下限がやはりでません。
390132人目の素数さん:01/09/30 20:45
>>386
ありがとうございます。
yahooで検索したら「数学の部屋」って
二つあるんですけど、どっちでしょうか?
もしよろしければURL教えてもらえませんか。
391132人目の素数さん:01/09/30 20:57
『数学の部屋』
ttp://web2.incl.ne.jp/yaoki/
392383、390:01/09/30 21:00
>>386
自分で見つけました。
ありがとうございます。
393383、390:01/09/30 21:07
>>391
行き違いになってしまいましたね。すいません。

もう一問教えてもらいたいのですが
「2連立2元一次方程式が不定、不能になる場合について
行列の視点から述べよ」という問題なのですが。
どなたか教えてくれませんでしょうか?
394整数論:01/10/01 00:35
だれか頭の良い方、数学の出来る方教えてください。
解説ではなく解答をお願いします。

1.x^2+x+4≡0 (mod 8) (mod 8)で異なるすべての解を求めよ。

2. x^(p-1) -1≡(x+1)(x+2)(x+3).....+(x+p-1) (poly mod p)

(1)この両辺のx^(p-2)の係数を等しいとすると、どんな結果が得られるか。
      また、p=2のときも正しいか。それはなぜか。

   (2)この両辺のx^(p-3)の係数を等しいとすると、どんな結果が得られるか。
      また、p=2,p=3,p>3の場合に分けて考えよ。p>3に対する結果を
      多項式を使わずに証明せよ。

   (3)この両辺のxの係数を利用して、p≧3ならば、plaであることを証明せよ。
      ここで、
            a/b=1+1/2+1/3+.......1/(p-1)
395>394:01/10/01 02:36
1.x(mod8)で場合分けするだけで、x≡3,4
実際、x^2+x+4≡(x-3)(x-4)(mod8)
2.係数を等しいとすると。。。というか、上の等式は正しい.
(1) p>2のとき、0≡1+2+…+(p-1)(modp)
  p=2のとき、左辺=x-1で、x^(p-2)の係数は定数項で-1
        右辺=(x+1)だから(mod2)では確かに定数項は一致してる.
        が、何と比べて正しいのかよく分からん.
(2) p=2のとき、意味あんの??x^(-1)の係数って。。。
  p=3のとき、左辺=x^2-1で、x^(p-3)の係数は定数項で-1 
右辺=(x+1)(x+2)だから定数項は2で(mod3)で一致してる.
        これも、何が正しいのかよく分からん.
  p>3のとき、0≡Σij(modp)(1≦i<j≦p-1)
       これを示したければ、(Σi)^2-Σ(i^2)がpの倍数であればよい.
       (p>3より 2Σij がpの倍数であればよい)
       (Σi)^2は(1)よりpの倍数
        Σi^2=p(p-1)(2p-1)/6 は当然整数で、かつp>3 よりpの倍数
       よって示せた.
(3)p≧3より 0≡Σ1*2*…*(p-1)/i(modp) (1≦i≦p-1)
       0≡(a/b)*(p-1)!(modp) 両辺にbをかければ
       0≡a*(p-1)!(modp) (p-1)!はpと互いに素なので
       0≡a(modp)
>>359
これってどこの問題?
397132人目の素数さん:01/10/01 04:41
すみません、哲学板で助言されてきました。
具体的に無限に存在するものを教えてくれませんか?
よろしくお願いします。
398397:01/10/01 04:47
すみません、解決しました。どうもです。
399整数論:01/10/01 05:16
>>395さん、素晴らしい解答ありがとうございます。

またまたですが、頭の良い方、数学が得意な方、賢い方お願いします。
(解説ではなくて解答で、スイマセン・・・)

1. x^85≡1 (mod 102)には、(mod 102)で異なるいくつの解があるか。

2. (a,n)=1とする。
       b≡c (mod ordn(a))のとき、そのときだけ、
   
         a^b≡a^c (mod n)
   であることを証明せよ。

3. (a,15)=1ならば、
       a^{φ(15)/2}≡1 (mod 15)
したがって、15は原始根を持たないことを示せ。
   (ヒント:(mod 3)と(mod 5)の合同を調べよ)

4. gをnの1つの原始根とする。
     g, g^2, g^3,..........g^φ(n)
    は、(mod n)の既約剰余系をなすことを示せ。

賢い方、お願いします。
400132人目の素数さん:01/10/01 06:28
質問
ディスプレイには表示できる。画像のパターンは無限でしょうか?
もし有限なら何種類ぐらいですか?
>>400
解像度(幅*高さ)*色数(RGB各8ビットなら2^(8*3))
402132人目の素数さん:01/10/01 08:03
>>401
有り難うございました。
これで自慢できます。
403339:01/10/01 11:03
>>387

アリガト
404132人目の素数さん:01/10/01 11:03
この問題誰か解いて下さい

トム 君の三人の子供、年はいくつだったっけ?
ジェリー 三人の子供の年の「積」は36だよ。
トム まだ分からないな。
ジェリー 子供の年を全部合わせた数と、君の家族の数とは同じだよ。
トム もう少しヒントを。
ジェリー 一番の年長は赤髪だよ。
トム ああ、これで全部分かったよ。

ジェリーの子供達の年は?
405>399:01/10/01 11:05
1.x^85≡1(mod2)かつx^85≡1(mod3)かつx^85≡1(mod17)
  x≡1(mod2)かつx≡1(mod3)はすぐ分かり
  x≠0(mod17)ならばフェルマーの小定理よりx^16≡1(mod17)
これよりx^85≡x^5≡1(mod17)
x≡2(17)⇒x^5≡‐2 x≡3(17)⇒x^5≡5
  x≡4(17)⇒x^5≡4 x≡5(17)⇒x^5≡-3
x≡6(17)⇒x^5≡7 x≡7(17)⇒x^5≡11
x≡8(17)⇒x^5≡-8
x≡-k(17)⇒x^5≡-k^5 だから上より解はx≡1(17)のみ
  以上とC.R.Thより全部で1個.
(別解)(Z/102Z)^× での位数85の元を求めればよいが、
  この群の位数はφ(102)=1*2*16=32
  であり、任意の元の位数は32の約数となる.
  (85,32)=1 だから1のみ解である.
2.b≡c(mod ordn(a))の時明らかにa^b≡a^c(modn)
逆に、a^b≡a^cとし、b≠c(modm) (m=ordn(a))
と仮定する.
  この時、b=pm+q c=rm+s と表せば
  a^q≡a^s(modn) 仮定よりq≠sで小さい方で割ってやれば
  位数mの最小性に反するので、題意は示した.
  ((a,n)=1なのでOK)
3.φ(15)=8 a^4≡1(mod15) を示せばよく、
  a^4≡1(mod3) a^4≡1(mod5) と分けて示せばよい.
  a≡2(3)⇒a^4≡1(3)…等とすればいい(めんどい。。。)
4.原始根の定義からg,g^2,…,g^φ(n)=1 はmodnで全て異なる.
  もし同じ物があれば、割り算すれば(群なので可)
  位数の最小性に反する.
  つまり、剰余系をなしている.
406さるです:01/10/01 11:16
√2+√3+√5+√7が無理数?の さる です。週末はちょっとネットをのぞけな
かったもので,すごい亀レスになってしまってすみません。
トレース写像を使えば簡単とのこと,ご教授ありがとうございます。でもまだ私は
トレース写像まで理解していません。もちろんがんばって理解するつもりです。
あとは>>301の方法に興味を持っています。高校生にももしかしたら説明できる雰囲気
がありそうだからです。さて,>>301
>   √pm+1=a1+a2√pm と表せる.(a1a2はKm-1の元 上の事より環の元としてよい)
> a1*a2≠0の時、両辺を2乗すると帰納法の仮定に反します.
は簡単に理解できます。そして問題なのは「a1*a2=0の時」です。それが,次の
>これをどんどん続けると
>√(pm+1/q1*q2*…*qk)=q (q1,…,qkはp1,…,pnのいずれか)
>             (qは有理数)
>となる.これは2乗すれば、相異なる素数であることに反する.
>よって成立.
が理解できないのです。もし,よろしければお教えください。
具体的に,√5∈Q(√2)(√3)として,つまり,
    √5=a1+a2√3  a1,a2∈Q(√2)
として矛盾を導こうと思ったのですが,無理やりなら矛盾は出てくるのですけど,
一般化できる形ではできなかったのです。
本当に厨房ですみません。よろしくお願いします。
407132人目の素数さん:01/10/01 12:09
Xの範囲0゜以上180以下において,関数f(X)=a(sinX+cosX)+2sin2X+1とする。ただし,aは正の定数とする。
(1)t=sinX+cosXの値の範囲を求めよ。
(2)f(X)の最大値,最小値を求めよ。
この問題がわからないので解いてもらえませんか。
408132人目の素数さん:01/10/01 12:22
>>407
(1) 「三角関数の合成」を使って t=(√2)sin(x+45°) を導け。
(2) 「t^2 = 1 + sin2x 」に注意して、
  f(x)をtの2次関数で表してみよ。
409132人目の素数さん:01/10/01 14:33
だれか>>357お願いします
410さるです:01/10/01 15:16
q<sub>1</sub>,q<sub>2</sub>,・・・,q<sub>m</sub>,p<sub>1</sub>,p<sub>2</sub>,・・・,p<sub>n</sub>をすべて相異なる素数とする時,<br>
    √(q<sub>1</sub>q<sub>2</sub>・・・q<sub>m</sub>)∈Q(√p<sub>1</sub>)(√p<sub>2</sub>)・・・(√p<sub>n</sub>)<br>
は成立しない。<br>
のように,一般化した命題ではそれほど難しくなく,帰納法で証明できました。皆さんどうもありがとうございました。(トレース写像についてはこれから勉強します。)
>401
(解像度)^(色数)ぢゃ?
412132人目の素数さん:01/10/01 16:11
「累乗すると2つ以上の同じ数が並ぶことはない」
 ↓ ここから始まって
>>231 >>252>>328>>329>>331>>340>>360
>>361>>362>>364>>365>>365>>366>>367
>>368
 ↑ ここで止まってます

だれか 考えて。
413412:01/10/01 16:14
↑ 1行目、言葉が変だけどゆるして
414132人目の素数さん:01/10/01 17:06
>>404
36を素因数分解すると 2*2*3*3 なので、
積が36になる3つの自然数の組み合わせは、
(1,1,36) (1,2,18) (1,4,9) (2,2,9) (3,3,4) (1,3,12) (1,6,6) (2,3,6)
の8通り。
1番目のヒントから、このどれかが答の筈。

それぞれの合計は 38,21,14,13,10,16,13,11で、
(2,2,9)と(1,6,6)の組み合わせは合計では区別できない。
だから、トムは2番目のヒントでもわからなかった。

3番目のヒントで「一番の年長」が存在する(双子ではない)ことがわかる。
ということで(1,6,6)は不適当で、
2才,2才,9才が正解。
>>357
もっといい解法があるかもしれんけど。
W={Y⊂X;f(Y)=Y,Y:closed,Y≠φ}にY≧Y'⇔Y⊂Y'で順序をいれる。これが帰納的
順序集合であることを示す。X∈WゆえW≠φ。{Y_i}をWの整列部分集合とする。
Y=∩[i]Y_iとおく。Y_iはcompact集合の閉部分集合の族で有限交叉性をもつゆえY≠φ。
f(∩[i]Y_i)⊂f(Y_i)=Y_iゆえf(∩[i]Y_i)⊂∩[i]Y_i。y∈∩[i]Y_iをとる。
Z_i=f^(-1)({y})∩Y_iとおく。Xは距離空間ゆえハウスドルフ、
よって{y}は閉集合。よってf^(-1)({y})も閉集合。
f(Y_i)=Y_iは閉集合。よってZ_iは閉集合の族で仮定y∈∩[i]Y_iより任意のiで
Z_iは空集合でない。またi>j⇒Z_i⊂Z_jゆえ有限交叉性をもつ。
ゆえφ≠∩[i](f^(-1)({y})∩Y_i)=f^(-1)({y})∩(∩[i]Y_i)。
ゆえにy∈f(∩[i]Y_i)。よって∩[i]Y_i⊂∩[i]f(∩[i]Y_i)。
以上によりf(∩[i]Y_i)=∩[i]Y_i。これからWは帰納的順序集合。
ゆえにWには極大元Yがある。これがもとめるYの性質をみたすことはあきらか。
でよいと思う。
416132人目の素数さん:01/10/01 18:26
322番どなたかおねがいします「
>>416
“Riemannにおける下積分”って
sup{芭in{f(p);p∈Δ_i}|Δ_i| ; Δは[0,1]×[0,1]の分割}
のこと?だったらどんな分割Δとその分割のひとつの部分Δ_iを
とってもΔ_iのなかにf(x)=0となる点があるから芭in{f(p);p∈Δ_i}
が分割Δ_iによらず0になるからsupも0でいいとおもうけど。
定義がちがうのかな?
>>417
4行目sup{芭in{f(p);p∈Δ_i}|Δ_i|が
に訂正すまソ。
419132人目の素数さん:01/10/01 18:50
>>415
>X∈WゆえW≠φ。
のとこなんですけど、fは全射とは限らないから
f(X)⊂Xじゃないんですか?
420132人目の素数さん:01/10/01 18:52
419
私はWが帰納的半順序集合になる事は示せたんですが、
Wが空でない事がいまいちわかりませんでした。
>>419
そのとうりだ。まちがえた。もっかいかんがえる。
422ちむ氏の使途:01/10/01 18:57
>>388
解説や途中式おねがいします。
ちなみに356です。
>>419-421
ああ、これよく考えたらもっとかんたんだね。
W={Y⊂X;f(Y)⊂Y,φ≠Y閉集合}
とおけばいいんだ。これならY_i∈W⇒∩[i]f(Y_i)⊂∩[i]Y_i
は自明、f(∩[i]Y_i)⊂∩[i]f(Y_i)も容易ゆえWが帰納的であることは
(さっきよりずっと容易に)しめせる。Wの極大元YをとってY≠f(Y)と
するとf(Y)⊂Yからf(f(Y))⊂f(Y)からf(Y)∈W。これはYの極大性に
はんする。これでどう?でも距離空間であることどこにもつかってないな。
なんでだ。どっかまちがってる?
424132人目の素数さん:01/10/01 19:13
>>422

だから>>358 に「式」が書いてあるって。
「解説」については、教科書や参考書の
「組合せ」「同じものを含む順列」「順列」のところを読め。
356に書いてる「式」の意味が書いてあるはず。


(ちなみに、「C(52,2)」は「52C2」
      「P820,2)」は「20P2」
 のことだ。)
425424:01/10/01 19:16
>>424

スマヌ。タイプミスだ。
P820,2) ⇒ P(20,2)
426132人目の素数さん:01/10/01 21:27
417>>ありがとうございます。
定義はあっています。
最終的には、Darubuxの定理で下積分を求めます。
min{f(p);p∈Δ_i}は、分割によっては、f(p)=0にならない場合があります。
f(x,y)={x x¬∈{1/2^n|n=1,2,3,4...}の時
      0 x∈{1/2^n|n=1,2,3,4...}の時
でf(p)を定義しているので、Xにのみ依存してこのf(p)がきまります。

ちなみに答えは、積分可能なので上積分 1/2に下積分が一致するはずです
427整数問題が分からない!:01/10/01 21:37
2次方程式x^2+(a-10)x+3a+10=0
が2つの整数解を持つ時
(1)aが整数であることを示せ
(2)aの値と二つの整数解を求めよ

解答をお願いします。
428整数問題が分からない!:01/10/01 21:41
xの整式f(x)=x^3+ax+bにおいて、全ての整数nに対してf(n)が6の倍数となるための定数a,bの条件はa+1とbがともに6の倍数であることを示せ
429整数問題が分からない!:01/10/01 21:44
自然数a,b,cがある。aが3より大きい素数であり、a^2+b^2=c^2が成り立つとき次の問に答えよ
(1)b,cをaを用いて表せ
(2)bは12の倍数であることを示せ
430132人目の素数さん:01/10/01 21:47
サイクロイドの方程式を導け、とのレポート。

半径a、角t
 x = a(t - sin(t)),
 y = a(1- cos(t))

サイクロイドは一番単純な、

 1直線j上を円がすべることなく転がるとき、
 この円周上の固定点のえがく線。


ヒント教えて。
431整数問題が分からない!:01/10/01 21:50
30の階乗30!について以下の問に答えよ
(1)2^kが30!を割り切るような最大の自然数kを求めよ
(2)30!の一の位は0である。ここから始めて十の位、百の位と順に左に見ていく。最初に0でない数字が現れるまでに連続していくつの0が並ぶか答えよ
(3)(2)において、最初に現れる0でない数字は何であるか理由とともに答えよ
432132人目の素数さん:01/10/01 21:53
>427
(1)解と係数の関係より終了
(2)解と係数の関係より2解をαβとすれば、α+β=10-a、αβ=3a+10
これより、αβ=3(-α-β+10)+10 (遅れたがα<βでよい)
整理すると、(α+3)(β+3)=49 よって、(α+3,β+3)=(-49,-1)(-7,-7)(1,49)(7,7)
あとはこれを解く.
>428 f(0)=b が6の倍数なのでbが6の倍数である事が必要
   f(1)=1+a+b が6の倍数なので上よりa+1が6の倍数である事が必要.
   逆に、a+1,bが6の倍数と仮定すると
   a+1=6k b=6m と書け,
   f(x)=x^3-1+6kx+6m=(x-1)x(x+1)+6(kx+m)
   ここで、(x-1)x(x+1)はxが整数の時6の倍数だから題意は成立
   (mod6で場合分けしてもよい)
>>426
すまソ。問題よみまちがえてた。
f(x,y)={x x¬∈{1/2^n|n=1,2,3,4...}の時
      0 x∈{1/2^n|n=1,2,3,4...}の時
ね。そら1/2だわ。ルベーグ測度0の領域をのぞいてf(x,y)=xだもんね。
でもルベーグ積分の理論は一切つかわずにしめさんといかんのね。
初等的に。たぶん。めんどくせ〜。
分割の形は[a,b]×[c,d]の形に限るは仮定していいの?
434132人目の素数さん:01/10/01 22:08
>>430
図を書けば自明じゃないの?
それだけじゃだめなのかな・・・
435132人目の素数さん:01/10/01 22:24
>>433さんへ
分割の形は[a,b]×[c,d]の形に限るは仮定してかまいません。
矩形で、分割するやり方で、授業は定義してました。
>>426,433
いやいや、ちょっとまて。これほんとに1/2?そんなはずない。
これルベーグ可積だけどリーマン可積でない典型例じゃないのか?
やっぱり下限積分0じゃないの?だって任意のPL分割Δにたいし
どうかんがえてもすべての分割の2単体は(1/2^n,y)なる形の
元をふくむもん。そじゃない?
>>435
うんうん。分割は矩形による分割にかぎるのね。だったらやっぱり
どうかんがえても[a,b]×[c,d](a<c,b<d)の形の集合はかならず
(1/2^n,y)の形の元をふくむよね。そこでfは最小値0をとるから
min{f(p);p∈[a,b]×[c,d]}=0だよね。
ちなみに

 定理 ルベーグ可積ならリーマン可積でその2つの積分値は一致する。

があって>>433に書いたとうりこの関数はルベーグ可積で積分値は1/2
だけど逆は一般になりたたないのでリーマン可積とはかぎらないし上限積分値と
下限積分値が1/2に一致するともかぎらない。
たぶん先生はこのルベーグ可積性とリーマン可積性がかならずしも
同値でない例としてこれをえらんだんじゃない?
438整数論:01/10/01 22:45
>>405さん、ありがとうございます!!お礼遅れてスイマセン。
頭いいんですね〜〜ホントすごいの一言です!!
またお願いします。賢い>>405さん
439132人目の素数さん:01/10/01 23:37
>>437さんへ
上限を計算すると、
縦横をn等分するとする。
もし、xが[(i-1)/n,i/n]の区間では、
sup{f(p)|(i-1)/n≦x≦i/n}は、i/nになる。
仮にf(i/n)=0なる値をとったとしても、i/n-ε (但し、∀ε>0)で、上限をとる。
故に、Σ_[i=1,n]Σ_[j=1,n]i/n * 1/n^2=1/2+n/2
となり、小矩形の幅を0に近づければ(n→∞)とすれば
1/2が従う
440ワータ:01/10/01 23:39
ad + cd >= 2√(adcd)
ac + bd >= 2√(adcd)
各不等式の両辺はともにせいであるから、辺々掛け合わせてー
と書いていたのですが、
なぜ、辺々掛け合わすことができるんでしょうか。
そんなことは、不等式の性質には、書いていませんでした。
お願いします。
441132人目の素数さん:01/10/01 23:43
437さんへ>>1/2^n (n∈N)を含まない小矩形では、0にはならないのでは?
442 ◆pvySbQO2 :01/10/01 23:45
>>436
>>437
>>322 は1/2になります。
443132人目の素数さん:01/10/01 23:48
>>440
非負な数A,B,C,D が
 A≧B
 C≧D
を満たしていれば
 AC≧BD
がいえるのはほとんど明らかだろう。
444132人目の素数さん:01/10/01 23:48
441です。
442 : ◆pvySbQO2 さん どうやりましたか?興味深いです。
>>439-442
ごめん。まちがってた。1/2^nね。k/2^nとかんちがいしてた。
逝ってきます。
446132人目の素数さん:01/10/01 23:55
322です。
大変迷惑かけてすみません。

なんとか下積分を出そうとしてますがやっぱり無理です。
どんな分割をすればいいのか・・
>>446
おれ437。ゴミレス連発でもう信用ないかもしれんけどやってみるね。
以下[0,1]×[0,1]におけるfの下積分を[f]とでもかく。
下積分などはリーマン積分などとちがってつねに定義できる。
各点でf(x,y)≦g(x,y)⇒[f]≦[g]は容易。
e>0をとる。g(x,y)をg(x,y)=f(x,y)(x≧e),0(0≦x<e)とさだめる。
g(x,y)≦f(x,y)≦xなので1/2-(e^2/2)[g]≦[f]≦[x]=1/2。
eは任意なので[f]=1/2。
でだうだらう。
>>447
すまヌ。>>447はまちがってる。これめんどくさいね。たまらん。
449 ◆pvySbQO2 :01/10/02 00:16
>>444
例えば[i/2^m,(i+1)/2^m]×[0,1]
(i∈Z,0≦i<2^m)と分ければ
f(x,y)=0となる点を含むのは
2m−1個なので下からの和をAとすると

 A
≧(1/2^m)煤Q{i∈Z,0≦i<2^m}(i/2^m)−(2m−1)/2^m
=(1/2^m)^2×2^m(2^m−1)/2−(2m−1)/2^m

となることから1/2になります。
>>449
おれ447。やっぱり俺なんかがでるまくなかった。逝ってきます。
451はなう:01/10/02 01:02
>>429
(1)a^2=(c-b)(c+b)
ここでaは素数なのでc-b=1。つまりb=c-1より
a^2=2c-1
って、ことで、c=(a^2+1)/2、b=c-1=(a^2-1)/2。

(2)aは5以上の素数なので、
a=1か5か7か11か-11か-7か-5か-1(mod24)(証明は自分で考えてください)
a^2=1(mod24)(全部2乗すればわかる)
て、ことで、a^2-1=0(mod24)で、bはその半分だけど、24の倍数の半分は必ず12の倍数だから
b=0(mod12)
452132人目の素数さん:01/10/02 01:03
499さんへ>>

≧(1/2^m)煤Q{i∈Z,0≦i<2^m}(i/2^m)−(2m−1)/2^m

−(2m−1)/2^m が良くわかりません。
453はなう:01/10/02 01:18
そーいや上の450合同式イコールで書いてた。ワシもぼけたのう。
>>431
(1)2で何回割れるかを考える。30!の中に
2の倍数・・15個
4の倍数・・7個
8の倍数・・3個
16の倍数・1個
あるので、全部で15+7+3+1=26回割れる。よってk=26。

(2)5で何回割れるかを考えればいい。30!のかけ算の中には5の倍数が6個、ただしそのなかに25があり、もう1回割れるので、7回割れる。よって7個。

(3)やる(笑)。
1*2*3*4*5*6*7*8*9*10/2/5/10≡3*4*6*7*8*9≡8(mod10)を利用して、
11*12*13*14*15*16*17*18*19*20/2/5/10≡1*6*3*4*3*6*7*8*9*2≡6*2*3*(3*4*6*7*8*9)≡8(mod109
21*22*23*24*25*26*27*28*29*30/2/5/2/5/10≡1*1*3*2*6*7*8*9*3≡2(mod10)
で、答えは8*8*2≡8(mod10)。なんかもっといい方法あった気がするけど忘れちゃった。
454 ◆pvySbQO2 :01/10/02 01:40
>>452
f(x,y)=0となる点を含むものに対しても
(1/2^m)煤Q{i∈Z,0≦i<2^m}(i/2^m)
の中で足してしまっているのでそれを引いているだけです。
ただそのまま引くと計算が面倒なので
−(1/2^m)(i/2^m)>−1/2^m
を使って多めに引いています。
455132人目の素数さん:01/10/02 02:24
>>443
AC≧AD≧BD
と書けばもっと明らか
456132人目の素数さん:01/10/02 02:25
>>443 「明らか」では回答になっていない。

 A > B > 0
 X > Y > 0

だとする。

A > 0 と X > Y より、AX > AY ⇒ @

Y > 0 と A > B より、AY > BY ⇒ A

@とAより、AX > BY
457132人目の素数さん:01/10/02 03:16
わけわかんない質問で悪いんだけどさ、
前期、後期通年の講義があって、年に2回テストするのね。
んで2つの平均点が60点以上で合格なの。
ある時センセイが「今回は2つのテストのうち、良い方に0.8、悪い方に0.2を掛けて計算する」
って言ったの。
こんな場合ってどういう数式が立つのかな?誰かおせーて。
1回目のテスト:a点、2回目のテスト:b点として
0.8*max(a,b) + 0.2*min(a,b) >= 60 で合格
例えば、
75点と0点→合格
61点と59点→不合格

対策としては、半期だけそれなりに頑張ろう(w
まちがえた、60点と59点→不合格
460132人目の素数さん:01/10/02 03:31
↑あれ?61点と59点やったら合格ちゃうの?
461132人目の素数さん:01/10/02 03:36
>>457
普通のセンセイのばあい:
数式も何もなく「全員不合格する」ということを意図している。

お馬鹿なセンセイの場合:
数式も何もなく「2回のうち片方の結果を重視したい」的な意図がある?
462461:01/10/02 03:42
>>457-460 100点満点じゃないの?
463457:01/10/02 03:47
どうもありがとう。
とりあえず75点取れば前期テスト受けてなくても合格ってわけだ!
464132人目の素数さん:01/10/02 04:00
群論の質問です。
有限群Gと素数pに関して
Gがp-群である⇔
Gと異なる任意の部分群の指数がpで割り切れる

は成り立つんでしょうか?
465132人目の素数さん:01/10/02 04:10
>>463
違うでしょ! 誰か答えてやれ!
2テスト数の平均を取られちゃうんじゃないの?
ひょっとして、
(0.8*max(a,b) + 0.2*min(a,b)) / 2 >= 60
で合格ってこと?
100点満点の試験なら鬼だ…
467132人目の素数さん:01/10/02 04:51
横軸に前期試験 縦軸に後期試験をとり
xy平面上に
(0.8*max(a,b) + 0.2*min(a,b)) / 2 >= 60
を図示せよ
468 ◆pvySbQO2 :01/10/02 05:00
>>464
成り立ちます。
<=の証明はシローの定理を使えばできます。
469457:01/10/02 05:31
>>465
いや違いますよ。
普通なら0.5掛けて合わせるとこを、今回は良い方に0.8、悪い方に0.2掛けて合わせる
ってことです。
470465 & 461:01/10/02 05:49
>>469
あ?そう? じゃあいい(ジョークでなく算数的に成立する)。
(でも日本語的には違うと思ったんだけどなぁ)
471132人目の素数さん:01/10/02 06:28
>>404誰かやって
>>471
404で検索してね。
>>412
>>368の「a=sですよ。」というのは既に>>365に「したがって a = s である。」と書いてあるでしょ。
んで、>>367で終わっている証明の検証はまだなされていない。
474 ◆pvySbQO2 :01/10/02 07:49
>>473
>>368 に書いたのは >>364 の(1)から分かるのは
a=10−sではなくa=sだということです。
475132人目の素数さん:01/10/02 12:20
>>430です。

>>434
そういえば授業では、円のパラメータを例にやりましたが、
単位円を書いてあとは
「みんなもよく知ってるとおりxはcos、yはsinなので、
x = cos(t),
y = sin(t)
だYO」

で終わりました・・・。

これをふまえてサイクロイドも図を書いて終わりなのかな・・・?
>>474
そうか。やっと意味が分かった。
>>364の与式 (1) に m^n = 10r + s を代入すると

a 10^b = 9 (10r + s) + a = 90r + 9s + a

になるのを、

a 10^b = 10r + s + a

と誤解していたから s = 10 - a が導き出せると思ってしまったんだ。

もう一度仕切りなおしか…。9 m^n = 10 r + s と置けばうまくいくのかなぁ…。
>>476を踏まえて再度仕切りなおし。

与式 (1) … a 10^b = 9 m^n + a (a は1桁の自然数、b,m,n は2以上の自然数)

左辺が10の倍数であるので右辺も10の倍数である。

m = 10p + q , 9 m^n = 10r + s (p , r は負でない整数、q , s は1桁の自然数)
の形に表したとき、q および n に対応する s および a の値は次の表のようになる。

q = 1         → s = 9 → a = 1

q = 2 & n = 4t + 1 → s = 8 → a = 2
q = 2 & n = 4t + 2 → s = 6 → a = 4
q = 2 & n = 4t + 3 → s = 2 → a = 8
q = 2 & n = 4t   → s = 4 → a = 6

q = 3 & n = 4t + 1 → s = 7 → a = 3
q = 3 & n = 4t + 2 → s = 1 → a = 9
q = 3 & n = 4t + 3 → s = 3 → a = 7
q = 3 & n = 4t   → s = 9 → a = 1

q = 4 & n = 2t + 1 → s = 6 → a = 4
q = 4 & n = 2t   → s = 4 → a = 6

q = 5         → s = 5 → a = 5

q = 6         → s = 4 → a = 6
>>477の続き

q = 7 & n = 4t + 1 → s = 3 → a = 7
q = 7 & n = 4t + 2 → s = 1 → a = 9
q = 7 & n = 4t + 3 → s = 7 → a = 3
q = 7 & n = 4t   → s = 9 → a = 7

q = 8 & n = 4t + 1 → s = 2 → a = 8
q = 8 & n = 4t + 2 → s = 6 → a = 4
q = 8 & n = 4t + 3 → s = 8 → a = 2
q = 8 & n = 4t   → s = 4 → a = 6

q = 9 & n = 2t + 1 → s = 1 → a = 9
q = 9 & n = 2t   → s = 9 → a = 1

(但し、t は自然数)

ここで、与式 (1) を変形および代入して得られる式
a (10^b - 1) / 9 = 10r + s … (2)
の左辺は全ての位が a である自然数を表しており、その一の位も当然 a である。

したがって a = s である。
また、上の対応表のうち、a = s となるのは q = 5 のときのみである。

以下は>>366-367と同じです。
またもや嘘を書いてしまいました。
>>477-478

与式 (1) を変形および代入して得られる式は
a (10^b - 1) = 10r + s … (2')
でした。

ここからは a = 10 - s 以上のことは分かりませんね。
ということは、この方法で a の値を限定するのは不可能ということでしょうか。
480:01/10/02 13:53
教えてください。

2点A(-2,0),B(1,0)からの距離の比が2:1である点Pの軌跡の方程式を求めよ。
481132人目の素数さん:01/10/02 14:10
>>480
P(x,y)とおくと、題意の条件から
 PA^2 : PB^2 = 4:1
⇔{(x+2)^2 + y^2}:{(x-1)^2 + y^2} = 4:1
がいえる。これを整理してみそ。
482132人目の素数さん:01/10/02 15:42
誰かこれお願いします。

pを素数として、位数p^3の非可換群が存在することを示せ。
483:01/10/02 16:31
y=arcsin(2x-3)
を微分してください。
計算過程もお願いします。
485132人目の素数さん:01/10/02 17:23
複素関数zについて
|∫f(z)dz|≦∫|f(z)||dz|
が成り立つ、証明略。
と複素関数論のテキストにかいてあるのですが、
どうしてこれが成り立つのかわかりません。どなたか証明わかりませんか?
486マスマニア:01/10/02 17:32
お世話になっています 質問よろしくお願いします

問題は参考書からの抜粋です
-----------------------------------------------------------------------
問題
平面上の曲線Cが媒介変数tを用いて
x=sint-tcost …(1) y=cost+tsint…(2) (0≦t≦π)のとき曲線Cはtを
動かすとどのような図形をえがくか
--------------------------------------------------------------------------
私の解答
(1)(2)を両辺二乗して足すと
x^2+y^2=t^2 となる つまり中心(0,0)の半径tの円を描く

-----------------------------------------------------------------------
さて ここからが私の質問です どうやら参考書の答えをみると私の解答は
間違っているようです。しかし なぜこの考えがあやまりなのか しっくりきません
一応 僕の解答もxとyで式が成り立っていますよね いろいろ考えた結果
『僕の答えは式の中に変数tが入ったままなので間違いなのではないか』と考えました
そしておそらくこの考えで正しいのでしょう。しかしなぜ変数が式の中にはいっていると
誤りなのか釈然としません 教えてもらえないでしょうか?

このような疑問をもった問題はほかにもあります 下記の問題です
-------------------------------------------------------------
問題
定三角形ABCがある。実数kにたいして点Pが
pa+2pb+3pc=kab を満たしている kが実数全体を動く時pのみたす
軌跡をもとめよ ただし 小文字はベクトルを表すとする つまり
paとはPAベクトルのことであり pbとはPBベクトルである
-------------------------------------------------------------------------
私の解答
全てのベクトルを始点をAとした ベクトルにかきかえると

ap={(5-k)/6}×{(2-k)ab+3ac}/( 5-k)={(5-k)/6}×{(2-k)ab+3ac}/{3+(2-k)}

一般に  (mab+nab)/(n+m)は ABをn対mに内分した点をさすベクトルであるので
この考えにしたがって
apはABを 3対(2-k)に内分した点とAを結ぶ直線 ということができる
-------------------------------------------------------------------------------------
この『私の』解答も 最終的に変数kが入って来た内容です
そして当然のように この解答は間違っているようです これらの経験から
『最終的に変数がはいる解答はあやまりである』との考えをもちました。
私の上の考え『最終的に〜〜』は正しいでしょうか?そして正しいならば
なぜ 最終的に変数が入って来た式はあやまりといえるのでしょうか?
よろしくお願いします
487:01/10/02 18:40
BC=6,CA=5,AB=7である三角形ABCの内心をI,BIの延長と辺ACの交点をDとし,ABベクトル=bベクトル,ACベクトル=cベクトルとするときAIベクトルをbベクトル,cベクトルで表せ。
488数学すき:01/10/02 18:51
お世話になっています 質問よろしくお願いします

同値変形について

一般に 『等式』(1)と(2)から(3)ができたらかならず
(3)かつ(1) ⇔(3)かつ(2) ⇔  (1)かつ(2)

は成り立つのでしょうか?
例として あげると

x^2+2y^2=1…(あ) と2x+3y^2=k…(い) の二つの式が成立するとき
Kの最大値をもとめよ という問題のなかで

(あ)を変形して y^2=1/2(1-x^2)  これを(い)に代入して

k=-3/2(xー3/2)^2+(13)/6…(う)ができる
(う)だけ単独で考えて答えをだしてはだめである なぜなら(あ)と(い)から(う)ができてるので
(あ)かつ(い)⇒(う)  はなりたち (う)単独では (あ)かつ(い)と同値では
ないからである

よって(う)かつ(あ)を考えてはじめて(あ)かつ(い)と同値になる

(あ)かつ(い)⇔(う)かつ(あ) が成立するので
(う)と(あ)を考慮に入れて Kの最小値と最大値をだせばいいのである

(あ)を変形して y^2=1/2(1-x^2)なので y^2≧0より -1≦x≦1なので
これを考慮に入れて(う)を考えると kの最大値13/6 最小値-2 となる
------------------------------------------------------------------------
この例では (あ)と(い)からできたもの(う)と(あ)を考えれば
(あ)かつ(い)⇔(う)かつ(あ) がなりたつと書いてありますね
このように一般的に
『等式』(1)と(2)から(3)ができたら『かならず』
(3)かつ(1) ⇔(3)かつ(2) ⇔  (1)かつ(2)
が成り立つのですか?
ちなみに等式の話しだということを最後に強調しておきます
不等式では こんなことは成り立たない事はしっています よろしくお願いします
489ちむ氏の使途:01/10/02 19:18
途中式がわからない。教えて。
どのような仮定でなったとか。この板の役目果たして。
490132人目の素数さん:01/10/02 19:54
>>489
偉そうだね。
答えを既に教えてもらっているし、教科書のどこを調べれば載っているかも書いている。
それなのに「途中式が分からない。この板の役目果たして」じゃあ誰も教える気にならない。
努力のかけらも見られないし、人に物を尋ねる態度でもないしね。

あーやだやだ。

態度を改めるまで放置よろしく >>491-1000
491>490:01/10/02 20:11
了解。
492132人目の素数さん:01/10/02 20:21

例えば、X=3とY=2から、(X^2)(Y^2)=36が導かれたとします。

で、以下の3つが同値かという話ですよね?

@「(X^2)(Y^2)=36かつX=3」
A「(X^2)(Y^2)=36かつY=2」
B「X=3かつY=2」

で解くと、

@「X=3、 Y=±2」
A「X=±3、Y=2」
B「X=3  Y=2」

となり、同値ではありません。

AとBからCを導く際に、
Cには、AやBとは異なる解が混ざってしまう可能性があるからです。
AやBは、単なる必要条件に過ぎません。


しかし、AかつB⇒C の変形が同値変形であった場合、
つまり、AかつB⇔C であるならば、話は別です。
この場合は、AかつB⇔BかつC⇔CかつAが成り立ちます。
例えば、BかつC⇔Bかつ(AかつB)⇔Aかつ(BかつB)⇔AかつB 等


>>488の例題の場合、(う)の導出過程が単なる代入であり、これは同値変形なので、

>(う)かつ(あ)を考えてはじめて(あ)かつ(い)と同値になる

なんて事が言える訳です。
493492:01/10/02 20:23
492は、>>488への説明ね。
494132人目の素数さん:01/10/02 21:09
>>486
最初の問題だけど、実際に書いてみようと考えればすぐ間違いに気が付く。
変数がtのとき原点からの距離がtの場所を曲線Cは通るだけじゃないの?

x=t, y=0

という問題でもx^2+y^2=t^2を満たすんだからさ。
495:01/10/02 21:17
>>485
積分は和の極限。
和で三角不等式 |z1+z2|≦|z1|+|z2|
が成り立つから極限でも成り立つ。
4961:01/10/02 21:18
497132人目の素数さん:01/10/02 21:20
449さん
ありがとうございました。
なんとか解けました^^
498132人目の素数さん:01/10/02 21:28
代数の問題です。
a_i(x),b_(x)は、多項式とする。
p(x,y)=a_0(x)y^n+a_1(x)y^(n-1)+…+a_n(x)とする。
p(x,b(x))=0ならば、xの多項式α_0(x),α_1(x),…,α_n-1(x)が
存在して、
p(x,y)=(y-b(x))(α_0(x)y^(n-1)+α_1(x)y^(n-2)+…+α_n-1(x))
と表せることを示せ。

更に、多項式c(x)¬=b(x)があって、p(x,c(x))=0ならば、
x,yの多項式Q(x,y)があって、
p(x,y)=(y-b(x))(y-c(x))Q(x,y)
と表されることを示せ。
499132人目の素数さん:01/10/02 21:50
小文字をベクトルとすると (例OPベクトルをop OBベクトルをobであらわす)

onを法線ベクトルとし cを定数とすると


2次元では op*on=C は直線を 3次元では op*on=C は平面を表すそうですが

その理由が参考書に次ぎのようにかいてありました(平面の場合)
-------------------------------------------------------------
平面αに垂直なベクトルをonとすると
α上にある定点Aとα上の任意の点Pについて
ap=on がなりたつ。これはつまり
(op-oa)=on でありon*op=oa*on=C(定点) となり

op*on=C は平面を3次元上ではあらわす
---------------------------------------------------------------------------
これをみてなるほどと最初におもったのですが しかしすぐに疑問がわいてきました
この考え方の平面という言葉を直線という言葉におきかえれば そのまま
直線を表す方程式の説明になりませんか? つまり僕がなにをいいたいかというと
上にかいてある説明だけでは 『op*on=C は平面を3次元上ではあらわす』という事が
納得いかないのです だれかもう少し
『op*on=C は平面を3次元上ではあらわす』ことの証明を僕に教えてください
500132人目の素数さん:01/10/02 21:53
誰か>>482お願いします
501132人目の素数さん:01/10/02 22:07
しつこいけど、 >>430 です。
例えば、↓のページにあるようなことをやって提出すれば
OKでしょうか?
http://www.nikonet.or.jp/spring/mery/mery_2.htm

先生に聞けってことをここで聞いてますが・・・。
502まおまお:01/10/02 22:14
>>500
限り無く概出に近いので、皆答えないのでしょう。

a, b, c ∈ Z/pZ としたとき、
1 a b
0 1 c
0 0 1
の形の3x3行列全体は、群をなすでしょう。
全部でp^3個あるやね。
もちろん非可換でありましょう。
確認されたし。
>>500

位数pの有限体を Fp と書くことにする。

G = { (a,b,c) | a,b,c ∈ Fp }
とし、G の (Fp)^2 への作用を

(x,y) → (x+cy+a,y+b)

で定める。Gの2つの元の積を、この作用の合成で定めると、Gは非可換群となる。
504なし:01/10/02 22:21
>>501
これは幾何学の問題です。図を描いて「tだけ転がったら ...だけ
下がるから、その式が出てくる」のように説明してみてください。
505503:01/10/02 22:26
>>502
かぶった。ケコーンしるか?

(注/練習問題)
502の群と503の群が同型であることを示せ。
506教えて:01/10/02 22:35
コラッツ予想が証明できません
教えてください
507名無し募集中。。。:01/10/02 22:39
>>501
たぶん、円の中心が時間でどう変わるかが重要かな?
そこからサイクロイドが求められるから。

とりあえずtのときの円の中心はa*t。その場所からx座標はa*sin(t)だけ下がるから、
x= a*t - a*sin(t)
とか書いておけば大丈夫でしょ。
508132人目の素数さん:01/10/02 22:43
498です。

お願いすることを忘れてました。
よろしくお願いします
509なし:01/10/02 22:48
>>498, >>508
yに関する因数定理をご存知か。
510なし:01/10/02 22:50
>>506
未解決問題に挑戦するとは盲目の旅人。証明を探し回るうちに
食料が尽きるとも限らない。君の方針を伝え給え。
511132人目の素数さん:01/10/02 23:06
>>501
このURLでは、かえって難しいのでは?
でも図形の問題をテキストで説明するのは、もっと難しいなあ。
一応、説明を試みてみます。

@半径aの円上の固定点と原点が一致した状態を、初期状態とします。

A次に、円を右にatだけ転がした図を書きます(便宜上、動かす距離はtでなくatとします)
 この時、固定点をP、円と地面の接点をQ、円の中心をOとします。
 当然、固定点Pは上方に動いています。

B円が右にatだけ転がりましたが、この時、弧PQの長さは?
 ・・・同じくatですね(ここが、この問題の急所です)
 角POQの大きさは、弧長÷半径=at/a=t で、tです。

CPから、半径OQに垂線を引いて下さい。これをPHとします。

DPのy座標=HQ=OQ−OH=a-OH=a-OPcos(t)=a-a・cos(t)
 Pのx座標=at-HP=at-OPsin(t)=at-a・sin(t)


・・・まあ、Aの図を書いて、よく考えてみて下さい。
512工房:01/10/02 23:10
数学分かりません。
暗記しまくらないとだめですか?
513132人目の素数さん:01/10/02 23:30
509>>
因数定理はわかります。
ちなみに証明は、
f(x)がf(a)=0となるとき、
0=f(a)=(x-a)Q(a)+h=h
従って、
となり、h=0となる。故に(x-a)で割り切れる。
逆に、f(x)が(x-a)で割り切れる(因数をもつ)ならば、
同様に上の式から、f(a)=0が明らかに言える。


因数定理:x-aが整式p(x)の因数(割り切れる)⇔P(a)
514132人目の素数さん:01/10/02 23:31
因数定理:x-aが整式p(x)の因数(割り切れる)⇔P(a)=0
515なし:01/10/02 23:34
a が数に見えて何故に b(x) が数に見えない?
516なし:01/10/02 23:41
>>512
論理的に分からないのか、それとも読解力がないのか。
暗記するかどうかは、効率と方法を検討した後、帥が決めるゆえ。
手順よりも先に規則を知れ。
517132人目の素数さん:01/10/02 23:53
499の
「xの多項式α_0(x),α_1(x),…,α_n-1(x)が存在して」
の存在の示し方がどうしてもわからなくて、それで全体を通して
聞いてます。

515>>多分、因数定理と同様の議論をすればよいのだと思いますが、
「xの多項式α_0(x),α_1(x),…,α_n-1(x)が存在して」
の存在がわかりません
518なし:01/10/03 00:17
p(x,y) は、y の n 次式で、y−b(x) は、y の 1 次式ですよね。
519はなう:01/10/03 00:21
今日はにぎやかやね
>>487
DはACを7:6に内分する点(証明は自分で考えてね。内心は角の2等分線の交点だからです)
だから、ADベクトル=7/13*c
で、ここで、AD=5*7/15=35/13で、これとAB=7との比較より、
IはBDを7:(35/13)=13:5に内分する点なので、
AI=(5/18)a+(13/18)ADベクトル=(5/18)a+(7/18)c
520132人目の素数さん:01/10/03 00:31
518さんへ:deg(f(y)g(y))=degf(y)+degg(y)を用いて、
p(x,y)=(y-b(x))Q(x)

Q(x)がyについてn-1次式ということを導くわけですか?

「更に、多項式c(x)¬=b(x)があって、p(x,c(x))=0ならば、
x,yの多項式Q(x,y)があって、
p(x,y)=(y-b(x))(y-c(x))Q(x,y)
と表されることを示せ。」
についても、先の議論と同じことをすればよいのですか?
521びちぐそ:01/10/03 00:39
z=f(x,y),x=x(t),y=y(t)のとき,d~2z/dt~2を求めよを教えてください!
dz/dt=f_x(dx/dt)+f_y(dy/dt)

,d~2z/dt~2=d/dt(f_x(dx/dt)+f_y(dy/dt))

=(∂/∂x)(f_x(dx/dt)+f_y(dy/dt))+(∂/∂y)(f_x(dx/dt)+f_y(dy/dt))

=f_xx(dx/dt)~2+2f_xy(dx/dt))dy/dt)+f_yy(dy/dt)~2

ってしたんですが、答えが合いません。どうか間違いを指摘してください。
522なし:01/10/03 00:51
>>521
? d/dt=∂/∂x +∂/∂y ?
523びちぐそ:01/10/03 00:57
>>522
書き間違えました・・・

=(∂/∂x)(f_x(dx/dt)+f_y(dy/dt))(dx/dt)+(∂/∂y)(f_x(dx/dt)+f_y(dy/dt))(dy/dt)

です。
524506:01/10/03 01:00
>>510
もちろん有名になるため
簡単そうなんだけどなー
525数学すき:01/10/03 02:16
492さん 素晴らしい解答ありがとう とってもよくわかりました
526 ◆pvySbQO2 :01/10/03 05:07
>>498
y−b(x)のyの係数は1なので商のy^kの係数がxの多項式になります。
p(x,y)をy−b(x)でyの次数が下がるように割ると
a_0(x)y^nをyで割るとa_0(x)y^(n−1)なので
p(x,y)−(y−b(x))(a_0(x)y^(n−1))を
計算すればyの次数が下がり残りのy^kの係数もxの多項式になっています。
これを繰り返していけば商のy^kの係数が
xの多項式になっていることが分かります。

別の方法だと商を狽早Qk(x)y^k,余りをr(x)として
 狽=Q(n−k)(x)y^k
=(y−b(x))狽早Qk(x)y^k+r(x)
のy^kの係数が等しいということからq_k(x)もr(x)も
xの多項式であることが分かります。

yについての2次以上の多項式で割るときでも最高次の係数が
xについての定数なら商のy^kの係数も余りのy^kの係数も
xの多項式になります。
527132人目の素数さん:01/10/03 05:20
aを整数とし、nは正の整数とする。
fは[a,a+1]で定義された実解析可能な関数とする
次の等式(Wool houseの公式)が成り立つことを示せ。

f(a)+f(a+(1/n))+f(a+(2/n))+…+f(a+1-1/n)+f(a+1)
=n{f(a)+f(a+1)}-{(n-1)/2}{f(a)+f(a+1)}
-{(n^2-1)/(12n)}{f'(a+1)-f'(a)}+{(n^4-1)/(720 n^3)}{f'''(a+1)-f'''(a)}+…

f'は一階微分f'''は3階微分をあらわします。

お化けスレの46さんのアドバイスも参考にして少し改題しましたが
まだよく分かりません。簡単のためb=a+1としました。

f(x) を a, a+1/n, ..., a+1-1/n, a+1でマクローリン展開するように言われて
考えてみたのですが、右辺にはf(a)とf(a+1)の2つがあり行き詰まって
しまいました。

何か足し合わせ方に秘訣のようなものがあるのかと思い
今一度質問させて頂きます。
528132人目の素数さん:01/10/03 05:39
>>502 >>503
ありがとうございます!例作るのってなんかコツあるんでしょうか。
529 ◆pvySbQO2 :01/10/03 06:00
>>231
10進で表したとき全て同じ数字になる10以上の累乗数について

2以上の整数nに対して
u(n)=(10^n−1)/9
と置きます。

(2)
2u(n)=2(2k+1)なので2u(n)は累乗数ではない。
(6)
6u(n)=2(2k+1)なので6u(n)は累乗数ではない。
(4)
4u(n)=4(4k+3)なので4u(n)は累乗数ではない。
(5)
5u(n)=5(5k+1)なので5u(n)は累乗数ではない。
(9)
9u(2)=99は累乗数ではない。
3≦nとする。
9u(n)=4k+3なので9u(n)は平方数ではない。
9u(n)=10^n−1=a^b
aは正の整数,bは3以上の奇数と置く。
10^n=(a+1)煤Q{0≦i<b}(−a)^i
煤Q{0≦i<b}(−a)^iは奇数なのでa+1は2^nの倍数。
2^n−1≦a
10^n−1<(2^n−1)^4なのでb=3となり
10^n=(a+1)(a^2−a+1)となるが
a^2−a+1は2の倍数でも5の倍数でもないので
これを満たすa,nは存在しない。
よって9u(n)は累乗数ではない。

u(n),3u(n),7u(n),8u(n)については
累乗数になるのかどうか分かりません。
530132人目の素数さん:01/10/03 12:03
自由加群の任意の基底の基数は等しいことの証明ってどうやるんですか?
531132人目の素数さん:01/10/03 12:40
↑教科書見たら?
532:01/10/03 13:16

三角形ABCの内接円の接点を、P、Q、Rとします。
(頂点Aの対向の接点をP、Bの対向をQ、Cの対向はRとします)

この時、直線APとBQとCRは一点で交わる様な気がするのですが、
今まで聞いた事もありません。
三角形の五心にも入っていない様です。

証明を試みましたが、全然ダメです。
誰か証明出来ますか?
533 :01/10/03 14:36
>532
証明は簡単。

AQ=AR, BR=BP, CP=CQ なので、
(AR/RB)(BP/PC)(CQ/QA) = 1
『チェバの定理の逆』よりAP,BQ,CRは一点で交わる(証明終わり)。

名前がついてるかどうかは知らないです。
534132人目の素数さん:01/10/03 18:36
うまく言えないんですけど・・・
1^(1/3)は1でいいんですか?
(-1)^(1/3)とかが一意に決まるような約束事があるんですか?

c.f.
{cos(π/6)+isin(π/6)}^3
={cos(5π/6)+isin(5π/6)}^3
={cos(3π/2)+isin(3π/2)}^3
=-1

(-1)^(1/3)=?
 

>>351-352
>2cos(2π/7)=(-1+(7(2+ω))^(1/3)+(7(2+ω^2))^(1/3))/3
>の証明ってどうやるの?

・・・というわけで右辺の計算は一意なんでしょうか?
何か条件を付け足せばこの問題は成立するんでしょうか?
535数学すき:01/10/03 18:53
お世話になっています 今回は一般性についての質問です

----------------------------------------------------------
問題

空間内において点Pおよび 点Q それぞれ原点を中心とする半径3および4
の球面上を動くものとする。 また点Rはor=op+oqをみたす点として定める
(ベクトルは小文字であらわす事にする つまりOPベクトルはopとかく)
このときPQ+OR(ベクトルで表すと |pq|+|or|)の最大値および最小値
をもとめよ またこのときPとQはどのような位置関係か
----------------------------------------------------------------
参考書の解答
Oを原点とし 直線OPをx軸 平面OPQをxy平面とし
P(3,0,0) Q(4cosθ,4sinθ、0) としても一般性を失わない
…以下続く

---------------------------------------------------------------------
さてここからが僕の質問です 物事を単純化せるためによくこの
『〜とさだめても一般性をうしなわない』という言葉がでてきます
いきなりそういわれても とよくわからないことがおおいのです
一般性とはそもそもなんなのか? この問題の場合なぜこのように
おいても一般性を失わないのか? よくわかりません 教えてください
よろしくお願いします
536 ◆MI/xG.5. :01/10/03 19:52
>>535
>なぜこのようにおいても一般性を失わないのか?
球は中心に関して全方向に対称的だから。
例えば、P(0,0,3),Q(4sinθ,0,4cosθ)としたとしても、
O,P,Qの相対的な位置関係は参考書の解答と同じ。
537工房ていうか消防:01/10/03 19:52
>>516
スレ違いな質問にこたえてくれて
正直、どうも。
5381:01/10/03 20:12
539132人目の素数さん:01/10/03 20:31
>>535
マルチポストはやめましょう。
540532:01/10/03 22:26
>>533
有難うございました
541132人目の素数さん:01/10/03 22:54
>536 ありがとう 
542132人目の素数さん:01/10/03 23:32
ナナコさんは、ブランド物の新しいバックを買いました。
友人たちが見たがっているのですが、まだお披露目していません。
『それは赤か、黒か、白よ』
とナナコさんはヒントを言いました。
『ナナコだったら、赤じゃないだろうかなあ』とヒトミさん。
『黒か白かのどちらかだと思うわ』ニーナさん。
『だったら、きっと黒よ』とミツヨさん。
するとナナコは、『あなたたちの推理は、少なくとも1つは正しくて、少なくとも1つはまちがっているわ』
では、ナナコさんのバックの色は何色ですか?
543132人目の素数さん:01/10/03 23:35
「述語論理学」ってなんですか?
544132人目の素数さん:01/10/03 23:36
>>542
>赤じゃないだろうかなあ
ってどういう意味ですか?
545132人目の素数さん:01/10/03 23:38
>>544
赤だと思うって意味じゃないでしょうか?
546132人目の素数さん:01/10/03 23:43
>>545
日本語としてはそうだろうけど
(っていうか普通こんな言い回しする奴はいないw)
それだと問題が解なしになっちゃうから
赤じゃないと思うって意味でしょう。
547546:01/10/03 23:47
ちなみにナナコの言う「正しい」「間違ってる」は
論理学における命題の「真」「偽」とは
必ずしも対応していないようですね。
548数理くるめ:01/10/04 00:06
546の言うことが正しければ、白ってことになるのか?
549546:01/10/04 00:10
>>548
だろうね〜。
(解が一つに定まるはずと仮定するのであれば)
白以外答えの候補が見つからない。
550132人目の素数さん:01/10/04 01:41
>>533
ジェルゴンヌ点と言います
551132人目の素数さん:01/10/04 01:52
ご教授おねがいします。

2変数関数f(x,y)が、
○yを任意に固定すると、xの多項式として
○xを任意に固定すると、yの多項式として
それぞれ表せるとき、
f(x,y)はxとyの多項式として表せるでしょうか。
552 ◆pvySbQO2 :01/10/04 02:41
>>551
表せます。

X(n)={x|f(x,y)がyのn次以下の多項式}
Y(n)={y|f(x,y)がxのn次以下の多項式}
としnを|X(n)|,|Y(n)|が共に無限となるようにとる。
x(i)∈X(n)(0≦i≦n)
y(i)∈Y(n)(0≦i≦n)
となるようにx(i),y(i)をとる。
gをxについてもyについてもn次以下で
g(x(i),y(j))=f(x(i),y(j))
(0≦i≦n,0≦j≦n)
となるようにとる。
f(x,y(j))もg(x,y(j))もxについてのn次以下の多項式で
x=x(i)(0≦i≦n)で一致するので
f(x,y(j))=g(x,y(j))。
a∈X(n)に対して
f(a,y)もg(a,y)もyについてのn次以下の多項式で
y=y(i)(0≦i≦n)で一致するので
f(a,y)=g(a,y)。
b∈R(又はb∈C)に対して
f(x,b)もg(x,b)もxについての多項式で
x∈X(n)で一致するので
f(x,b)=g(x,b)。

よってf(x,y)はx,yの多項式として表せる。
553132人目の素数さん:01/10/04 04:05
>>527
ヒッソリ計算してみたら俺も計算が合わん(鬱
誰か最後までちゃんと計算しきった人いる?
554132人目の素数さん:01/10/04 07:49
簡単でしょうけど誰か解いて下さい。お願いします。

問1 49068239205968386970482062を9で割った時の小数第12位の数字
問2 10000000000000000000000001を9で割った時の小数代12位の数字
問3 sin(450°)
問4 2048の約数の総和。
>>115
これ分かる人いますか。
>>554
ダブって書かなきゃ 望み(?)はあったのに・・・
557132人目の素数さん:01/10/04 07:59
>>554
問1の解 0
問2の解 2
問3の解 1
問4の解 4095
>>557
速やっ
559132人目の素数さん:01/10/04 08:17
R^nで、(x_1,x_2,・・・,x_n)を極座標変換したときの
ヤコビアンが
r^(n-1)・(sinθ_1)^(n-2)・(sinθ_2)^(n-3)・・・sinθ_(n-2)
とどうしてなるのか教えてください。
>>430です。

>>504,>>507, >>511
ありがとうございます。
実際、>>511さんの説明を図を書きながら追っていったら
わかりました。
561132人目の素数さん:01/10/04 14:03
arcsin(x+iy)など複素数で逆三角関数を扱う時に
値を求める方法を教えてください。
>>551

ちなみにこれ、有理数体上で考える場合は
「否」だね。
563132人目の素数さん:01/10/04 15:40
どなたか助けてください。>>534です。
聞き方が変だとは思いますがよろしくお願いします。
564132人目の素数さん:01/10/04 16:02
>>563

>>534は、何をききたいのか、よく分からないけど、
1の立方根も、-1の立方根も、一意ではなく、3つあるよ(虚数を許せば)。

{cos(π/6)+isin(π/6)}^3 =-1より、
cos(π/6)+isin(π/6)=-1、又は、(1±i√3)/2

どれが正しいかは、三通り検証するんだね。
この例題だと、(1+i√3)/2 かな。
565132人目の素数さん:01/10/04 16:06
>>563
{{t -> -1/3 + 7^(2/3)/(3*((1 + 3*I*Sqrt[3])/2)^(1/3)) +
((7*(1 + 3*I*Sqrt[3]))/2)^(1/3)/3},
{t -> -1/3 - ((7/2)^(2/3)*(1 + I*Sqrt[3]))/
(3*(1 + 3*I*Sqrt[3])^(1/3)) -
((1 - I*Sqrt[3])*((7*(1 + 3*I*Sqrt[3]))/2)^(1/3))/6},
{t -> -1/3 - ((7/2)^(2/3)*(1 - I*Sqrt[3]))/
(3*(1 + 3*I*Sqrt[3])^(1/3)) -
((1 + I*Sqrt[3])*((7*(1 + 3*I*Sqrt[3]))/2)^(1/3))/6}}

ちょっと対称性に疑問があったのでMathematicaにt^3+t^2-2t-1=0を解かせてみたところ
このような解答が出た

>2cos(2π/7)=(-1+(7(2+ω))^(1/3)+(7(2+ω^2))^(1/3))/3

たぶんこの右辺は間違いで
少なくとも第一項の-1は3乗根の外じゃないか?
ωとω^2を入れ替えても同じ形なら納得できるが・・・
5661:01/10/04 16:17
567132人目の素数さん:01/10/04 17:43
大学の数学が難しくてたまりません。
テストでもちょっとひねった問題を出されると、
ほとんど解けない。と言うより見たような物しか分かんない。
この応用の利かない頭を良くする問題の答えを教えて下さい。
568563:01/10/04 18:45
>>564-565
即レスありがとうございました。

>>565
>少なくとも第一項の-1は3乗根の外じゃないか?
>ωとω^2を入れ替えても同じ形なら納得できるが・・・

私も最初見まちがえたんですが、
最後にまとめて3で割るためのカッコが広範囲で
一応対称性は保たれてるみたいです。
>>351でも一応{-1+u^(1/3)+v^(1/3)}/3のかたちで,uとvは共役)

>>564
聞き方が悪くてすみません。
z^3=uの複素解zが3つあるのはわかるのですが・・・

「複素数zにたいしてz^(1/3)は3つある」
「複素数zにたいしてzの三乗根は3つある」

両方同じことだと考えていいんでしょうか?
「4の平方根は±2」,「√4=2(だけ)」のように分けたりしないものなんでしょうか?

>>534の疑問をもった理由はこうです。
例えば(z_1)^3=u,(z_2)^3=vの解の組合せ{(z_1),(z_2)}は最大で9通りですよね。
一応(z_1)+(z_2)が9通りになるとは限らないまでも、必ずただ1通りにはなりませんよね。
だとすれば「x=u^(1/3)+v^(1/3)を示せ」という問題はどう示せば?
「最大9通りのうち1つは等式を満たすので題意は示された」でいいのですか?

uとvは共役なところが鍵だと思いますが混乱しています。
さらに質問してしまいましたがどうかお助けください。
569132人目の素数さん:01/10/04 21:35
>>562
なんで?
570132人目の素数さん:01/10/04 22:51
雑談スレや分からんスレで何度か話題になった問題。
>4*f(n,m)=f(n+1,m)+f(n-1,m)+f(n,m+1)+f(n,m-1)

これを少し変え、
・f(n,m)≡f(n+1,m)+f(n-1,m)+f(n,m+1)+f(n,m-1) (mod 2)
・全てのn,mに対しf(n,m)は0か1
(n,mは整数)

f(n,m)が↑の条件を満たすとき、

(n,m)によって構成される集合Aを与えた時に
Aの全ての要素(n,m)に対しf(n+k,m+l)=f(n,m)となる整数k,lが必ず存在するかどうか、
ってのがなかなか証明出来ません。

集合Aを有限集合に限った場合だけで、さらにヒントだけでも助かりますので
どなたか教えて頂けないでしょうか。
571564:01/10/04 22:53
>>568
言いたい事が分かりました。

◎一般論

実数の平方根の、「単なる√は必ず正の方を表わす」という決まり、
複素数には存在しません。
「zの立方根のうち任意の一つをaとすると、残りは、aω、aω^2 である。」
とおいて済ます事が多いです。
複素数の場合は、問題に応じて、最も適当なものをaにとります。
(例えば、実数の立方根は、実数根を基準にとる事が多い)



◎2cos(2π/7)=-1+(7(2+ω))^(1/3)+(7(2+ω^2))^(1/3))/3

この問題は説明不足です。
どの立方根をとれ、という指定は無いから、
おっしゃる通り、右辺は九通りの値を取り得ます。
その中で真の解は、ただ一通り。(2cos(2π/7)の値は、ただ一つだから)
この問題は、「各立方根を上手に選ぶ」事が暗黙の仮定になっているのです。


◎各立方根を上手に選ぶ粗筋(上の問題は、この部分がズッポリ抜けている)

・三次方程式の解である事から、まず九通りが三通りに絞られるよね。

  詳しく書くと、
  2cos(2π/7)=(-1+a+b)/3 とおいた時、
  aとbが共役複素数の関係になっている組み合わせを取ります。
  元の方程式の解は全て実数解である為(高校レベル)、
  aとbが共役複素数でないと、(-1+a+b)/3が実数にならないからです。
  三通りのaに対し、共役複素数を為すbは、各一通りずつあるので、
  この様なa、bの選び方も三通りです。

・次に、2cos(2π/7)は、0と2の間の値である事は明らかです。
 この区間での、t^3+t^2-2t-1=0 の解は一つしかありません(高校レベル)。
 こうやって、ただ一通りに絞られます。


◎上の問題を厳密に言うならば、こんな感じ

 t=-1+(7(2+ω))^(1/3)+(7(2+ω^2))^(1/3))/3とおく。
 tは九つの値を取り得るが、実数かつ0<t<2なるtは、唯一つ存在し、
  2cos(2π/7)=t
 である事を証明せよ。


◎でも本当は・・・

「x=u^(1/3)+v^(1/3)を示せ」は、
「9通り全ての場合において、等式を満たす」事を証明しなくてはいけません。
上の問題は、明らかに説明不足です
572難問:01/10/04 23:32
三角形ABCを書く。(上にA、左にB、右にC)
角B=50度 角C=55度。
角Bを上から20度と30度に分ける線分を引く。
その線分とACの交点をDとする。
角Cを上から30度と25度に分ける線分を引く。
その線分とABの交点をEとする。
角CDAは何度ですか?
573はなう:01/10/04 23:36
>>572
このままなら180度です。問題写しミスだと思われ
574難問:01/10/04 23:48
!訂正!

三角形ABCを書く。(上にA、左にB、右にC)
角B=50度 角C=55度。
角Bを上から20度と30度に分ける線分を引く。
その線分とACの交点をDとする。
角Cを上から30度と25度に分ける線分を引く。
その線分とABの交点をEとする。
角CEDは何度ですか?
575132人目の素数さん:01/10/04 23:51
>>572
本当は、

「BDとCEの交点をFとする。角BFCは何度?」

という問題だと予想する。
576562:01/10/05 00:05
>>569
すべての有理数に
 q_1,q_2,q_3,・・・
と番号をつけておく。(有理数全体は可算)
ここで
f(x,y) = (x-q_1)(y-q_1) + (x-q_1)(x-q_2)(y-q_1)(y-q_2)
     +(x-q_1)(x-q_2)(x-q_3)(y-q_1)(y-q_2)(y-q_3)
     +・・・
という関数を考える。
577名無しさん:01/10/05 00:37
ご教授願いたいことがあります。

「f(x)を滑らかで単調増加な実数値関数とし、f(0)=0とする。このとき、f(x)
に対するニュートン法で点列Xnが発散はしないが零点x=0に収束しないような
例をあげよ」

という問題なのです。f(0)=0ならx=0に収束するからニュートン法なん
だろーがー!!って思ったのですが、そうではない場合ってのがあるという
ことですよね……。どういう場合があるのか教えてもらえないでしょうか。
>574
角CED=35度

BC=1として電卓使って出した
うまく補助線引いてBE=DEを導ければ自明
579ピアス:01/10/05 01:38
こんな時間にこんばんは。
いま「16歳のアセラが挑んだ世界最強の暗号」という本を
読んでるんですが、まんなかぐらいに「モンティのゲームショー」
という問題が出てくるんです。
答えも書いてあるんですけど、どうしてそうなるのかわかりません!!
誰か教えてくださいー!

モンティのゲームショー

3つの扉のどれか1つの向こうに、車が1台隠されている。
残りの2つの扉の向こうには、山羊がそれぞれ一頭ずつ隠されている。
あなたはある扉(ここでは仮に1番とする)を選んだとしよう。
ゲームショーのホスト、モンティはどの扉の向こうに車があるか知っていて、
あなたの答えを聞くと残りの2つの扉のうちの1つを開け山羊を見せる。
ついで、もう一度あなたに、1番のままでいいか、それとも残された
もう1つの扉に変更するか、と声をかける。
さあ、あなたは変更するべきなのだろうか?

答えは、「変更すべき」なのだそうです。どうして?(><)
>579
激しくガイシュツ問題
これと暗号とどこで関係するの?
581ピアス:01/10/05 01:46
>580さん
うー。がいしゅつなんですか〜、、
暗号との関係は、、ごめんなさい
ここから先、まだ読んでないのでわかりません(^^;
こんな問題解いたよ〜!
みたいな話で、暗号とは関係しないような感じです。
582578:01/10/05 02:03
>574
線分BC上に、角CEF=25度となるような点Fをとると
CF=EF=EB=DE=DFになる (*)
△EFDが正三角形なので
角CED=角FED−角CEF=60度−25度=35度

しかし(*)の示し方がわからない
続きは誰かよろしく
583132人目の素数さん:01/10/05 02:36
>>577
こんなのはどう?

x≧0 f(x)=[-1/(x+1)]+1 f'=1/(x+1)^2 >0 単調増加
x<0 f(x)=[-1/(x-1)]-1 f'=1/(x-1)^2 >0 単調増加
すると、f(0)=0、f'(0)=1

双曲線の二本をずらして、0でくっつけたものです。

で、点列X を-1からはじめます。即ち、X_0=-1

x=-1のとき、f(1)=-1/2、f'(1)=1/4 なので、
X_1= α-f(α)/f'(α) = -1 - (-1/2)/(1/4)=1

x=1のとき、f(1)=1/2、f'(1)=1/4なので、
X_2= α-f(α)/f'(α) = 1 - (1/2)/(1/4)=-1

X_2=-1に戻ってしまった。
よって以降は、-1,1,-1,1,-1・・・・を繰り返すのみ。
発散している訳ではないが、0には収束しない。
584ピアス:01/10/05 02:38
579です
さがしてるんですけど見つけられません。
こんな時間なのに〜
どのスレなんですか、、
見つけられるキーワードだけでもお願いしますー!
585数理くるめ:01/10/05 02:39
>>579
がいしゅつっぽいがマジレスすると・・・
最初に選んだのが車である確率は1/3、残りの2つに車がある確率は2/3。
いま、「残り2つ」が「残り1つ」になったが、確率は2/3のままだから。
まぁ車よか山羊がほしければこの限りではないが・・・
586132人目の素数さん:01/10/05 02:42
>ピアス氏
このスレを読んで。
http://cheese.2ch.net/test/read.cgi?bbs=math&key=1002198866
587132人目の素数さん:01/10/05 02:44
588587:01/10/05 02:47
カブリマシタ 2ビョウサ
589ピアス:01/10/05 02:50
585さん 586さん 587あん!
ありがとうございます!
やっと眠れるかもしれないよー(^^
2フンダベ
あん!
ビミョウに(w
592577:01/10/05 09:22
>>583
なるほど。つなげてやればよかったのかー!
思い付きませんでした。ありがとうです!
593132人目の素数さん:01/10/05 12:17
自然数nに対して,ルートnに最も近い整数をanとする。
()自然数nに対して,ルートnに最も近い整数をanとする。
(1)a7,a50を求めよ。
(2)an=100となる自然数を求めよ。
(3)20001^Σ^k=1akの値を求めよ。
この問題の解き方が分からないのでヒントでもかまわないので教えてください。
594なし:01/10/05 12:25
>>593
√の逆算は平方(2乗)だったはず。「実数に最も近い整数」という条件を形式化すると、はさみ込みの不等号で表される。
595名無しさん:01/10/05 12:31
1:√7にもっとも近い整数か。2,3どっちが近いかな。2.5^2=6.25だ。
 ってことは、3>√7>2.5、よって、a7=3。a50は√50に近い整数だけど、
 まあ、√49のお隣だし、7だろう。√25を√2倍して計算してみてもよし。
2:間違いないのは、10000。なんせルート10000に最も近い整数は100だからね。
 多分、10000プラスマイマスちょこっとくらいってのもnになりうるね。
 んじゃ、境界を求めればいい。境界は、上が100.5、下が99.5だろう。
 境界を含むかどうかは計算したあとかんがえればよし。
 100.5^2= 10100.25かな。
 99.5^2=9900.25
 並べると、
 9900.25 < 9901 < ・・・ 10000 ・・・ < 10100.25 <10101
 √99.5  <    <     √100     <  √100.5 <
よーって、n=9901、9902、、、、10100(すべて自然数)が答えかな。

3:ごめんw 問題ようわからん。20001のシグマk乗?
596593です。:01/10/05 12:51
解説ありがとうございます。(3)はどうあらわしたらいいかわかりませんでした。え〜とΣの上に2001とあり、下にk=1とあります。
597132人目の素数さん:01/10/05 13:33
ak=1 となるようなkはいくつあるかな?
ak=2は? ak=3は?
a2001=45だからしらみつぶしにやれば答えはでるけどさ
ak=pとなるk:(p-0.5)^2〜(p+0.5)^2
この区間に含まれる整数の数はいくつさ?
598132人目の素数さん:01/10/05 14:15
>>596
>(3)20001^Σ^k=1ak
>(3)はどうあらわしたらいいかわかりませんでした。え〜とΣの上に2001とあり、下にk=1とあります
上の書き方だとわからないまたは違った意味に取られそうなので、
納k=1,20001]ak などと書いてください。
ちなみに納k=1,20001]ak の意味しているもの(解き方でなく)は わかってる?
>>577
任意の初期値に対して?それとも特定の初期値でいいの?
特定の初期値でいいなら繰り返しになるような例を作ればいい。
例えば
f(1)=1 f'(1)=1/2
f(-1)=-1 f'(-1)=1/2
を満たす適当な関数
600563=568:01/10/05 16:41
>>571
私のうまく聞けないところまでフォローが行き届いていて感動しました。
段落にわかれていて非常にわかりやすかったです。
おかげさまで混乱はきれいさっぱり消えました。
本当にありがとうございました。
601577:01/10/05 19:28
>>599
問題文には、577のようにしか書かれておらず、判断がつきませんでした。
私には、「任意の初期値で」成り立つような関数ってのが想像できなかったので、
「これは特定の初期値で」成り立てばそれで良いという問題なのだろうか?と
思っていたのですが。

仮に、「任意の初期値で」ということだった場合、どのような関数があげられる
のでしょうか?
>>577 >>599 >>601
「任意の初期値で」成り立つような関数:

f(x)=sign x |x|^(1/2)
f(x)が奇関数だからf'(x)=偶関数=1/2|x|^(-1/2)
f(x)/f'(x)=奇関数=-2x
x-f(x)/f'(x)=-x
まちがい,f(x)/f'(x)=奇関数=2x
605132人目の素数さん:01/10/05 22:21
正方形を奇数×奇数の桝目に分けて,四隅が黒になるように
市松模様で塗りわける。この升目の各行各列にひとつづつ石を置く。
すると黒枡においてある石は奇数個でありそうですが,これを示してください。
606数理くるめ:01/10/05 22:33
>>605
>正方形を奇数×奇数の桝目に分けて
この「奇数」というのは両方同じ数でいいんですか?
607ちむ教の信者:01/10/05 22:45
お願いです。教えてください。皆様。
8人の生徒がいる。
2人ずつの4つのグループにわけろ。
これは、x=4!=2520
なんですけど。なんでこの方程式ができるのかわかりません。
608132人目の素数さん:01/10/05 22:46
1組のトランプ52枚の中から、2枚のカードをひいてならべる。
なんとおりあるか。また、ハートは何通りあるか。
が、わかりません。どうか、教えて。
609数理くるめ:01/10/05 22:57
>>606のうだとすると・・・
奇数を2n+1(nは負でない整数)とおく。
まず、((行番号)−(列番号))が偶数ならば黒であり、奇数ならば白である。
(ここでいう奇数、偶数は負の数もふくむ)
ここで、k行目ではa_k列目に石をおくとすると、
黒におかれた石の数は(1-a_1),・・・,(k-a_k),・・・,(2n+1-a_(2n+1))のうち偶数であるものの個数。
これを偶数と仮定すると、2n+1が奇数であることから、(1-a_1),・・・,(k-a_k),・・・,(2n+1-a_(2n+1))のうち
奇数であるものの数は奇数となる。
このとき、(1-a_1)+・・・+(k-a_k)+・・・+(2n+1-a_(2n+1))は(偶数*偶数)+(奇数*奇数)で奇数となる。
しかし題意より、(1-a_1)+・・・+(k-a_k)+・・・+(2n+1-a_(2n+1))=0(偶数)
よって矛盾する。
以上より背理法から奇数。
610数理くるめ:01/10/05 23:00
冒頭部分
>>605
606のようだとすると・・・
に訂正。スマソ。
611132人目の素数さん:01/10/05 23:03
おねがいといて。
612132人目の素数さん:01/10/05 23:04
ん。
613数理くるめ:01/10/05 23:17
>>607
>x=4!=2520
これどういう意味?(4!=24)
たしかに2520になるけど(グループを区別するなら)
式は 8C2 * 6C2 * 4C2 * 2C2 =2520じゃん?
614数理くるめ:01/10/05 23:21
>>608
>1組のトランプ52枚の中から、2枚のカードをひいてならべる。なんとおりあるか
52P2 = 52*51=2652(通り)
>ハートは何通りあるか。
? ハート2枚ってこと? なら 13P2 =13*12=156(通り)
615132人目の素数さん:01/10/05 23:22
>>613

おそらく、
「求める答をx通りとすると、
 x・4!=2520
 ∴x=105」
ということだろう。
616 605:01/10/05 23:31
>>606,609
ありがとうございます。
「正方形」で両奇数が等しいことを表したつもりでしたが……
ありがとうございます。
617132人目の素数さん:01/10/05 23:39
申し訳ないんですが、緊急の質問です。
数学記号で「波線が2本」というものがあると思うんですが、
この記号は、PC上でどのようにして入力すれば良いのでしょうか?
ちなみにこの記号の意味も教えてくださると有り難いです。

ご存知の方、お手数ですがどうかご回答お願いします。
618数理くるめ:01/10/05 23:51
>>615
なるほど。
グループを区別するときが2520で区別しないときx通りとしているのか。
それで4つのグループのならべかたが4!だからx・4!=2520  ∴x=105 か。
619saas:01/10/06 00:14
唐突ですが、ある製品の面積を求めたいのですが、私は数学苦手でどうしても解決できないのでどなたかお願いいたします。

この製品は二等辺三角形で、場所に応じてカットします。
このときの左側の高さと、右側の高さ、上の水平幅、下の幅がわかっているとき、面積は出せるのでしょうか。それとももっと条件が必要なのでしょうか。歪な絵ですみません。
     上幅
     ←--→    
↑      / \     ↑     
 |      /  \    |
左|     /   \    |右
 |     \     \   |
側|     \      \   |側
高|   \       \  |高
さ|   \        \  |さ
 ↓  \____________\ ↓
   ←----------------→
      下幅
620132人目の素数さん:01/10/06 01:02
age
621はなう:01/10/06 01:18
>>617
??なんのこっちゃ。∬かのぅ。
622132人目の素数さん:01/10/06 01:26
>>619
ネタ?
数学でなくて算数。。。。

条件が必要などころか多すぎる…そんなに要りません。

(三角形の面積)=(底辺の長さ)×(高さ)÷2
あなたの図でいうと
(三角形の面積)=(下幅)×(左側高さ)÷2
で求まります。
誰かこの問題を解いてください。まったくわからないんです。
どうかお願いします。

X(エックス)を確率密度関数とすると、
XのPパーセント点(母Pパーセント点ということもある)は、
∫−∞(マイナス無限大が、∫の下)、Xp(エックスピーが
∫の上)F(X)dX=P/100を満たす値Xpとして定義される。
特に、50%点を母中中央値と呼ぶ。今、次のような分布関数
F(X)=0、X≦0F(X)=Xのn乗、0<X<1
F(X)=1、X>1但し、n≧1である。

@ 確率密度関数F(X)を求めなさい。

A Xの中央値をもとめなさい。

B Xの平均と分散を求めなさい。

よろしくお願いします。
すみません、上の投稿が汚くなったからもう一度書き込みます。

X(エックス)を確率密度関数とすると、XのPパーセント点(母Pパーセント点ということもある)は、∫−∞(マイナス無限大が、∫の下)、Xp(エックスピーが∫の上)F(X)dX=P/100
を満たす値Xpとして定義される。特に、50%点を母中央値と呼ぶ。今、次のような分布関数
F(X)をもった確率変数を考える。
F(X)=0、X≦0
F(X)=Xのn乗、0<X<1
F(X)=1、X>1
但し、n≧1である。
>>617
>>621

「〜」を縦に2つの↓これか?

   
  / ̄ ̄ ̄ ̄\
/            \
                \         
  / ̄ ̄ ̄ ̄\    \            /
/            \   \____/
                \
                  \            /
                    \____/
626132人目の素数さん:01/10/06 03:14
>623
使っている言葉の定義から押さえてください
そこまで滅茶苦茶だと問題が解けなくてアタリマエ…

Xは確率密度関数ではなくて確率変数ではないですか?
F(X)は確率密度関数なのか累積分布関数なのかどちらですか?

何かを写しているのであれば大文字と小文字はちゃんと区別すること
違うものに同じ記号を使わないこと

が大事です。
627132人目の素数さん:01/10/06 03:55
部分分数に置き換えるっていう意味がわかりません。
どうして、2x-1/x(x-1)
は1/x+1/x-1に置き換えれるんですか??
>>627
(2x−1)/x(x−1)=1/x+1/(x−1)。
629627:01/10/06 04:17
>>628
何度もすみません。
頑張って、間の過程を考えていたんですけど、
1/x+1/(x-1)
にならないので、詳しく教えてくれませんか??
630タリバン先生:01/10/06 04:31
>629
(2x - 1)/(x(x-1))=(a/x) + b/(x-1)

左辺が分母が0になる点、つまりx=0 とx=1で値を持たないことを踏まえて
右辺は分母をx、x-1にした分数の和と仮定します。

aとbを決めてみます。

両辺にx(x-1)をかけて分母をはらうと
2x-1 = a (x-1) + b x
= (a+b)x -a
となります。

よって、a=1、b=1でなければならないことがわかります。
>>627

置き換え前  (px+q)/{(x+a)(x+b)}
置き換え後  m/(x+a) + n/(x+b)

  m/(x+a) + n/(x+b)
= {m(x+b) + n(x+a)}/{(x+a)(x+b)}
= {(m+n)x + (na+mb)}/{(x+a)(x+b)}
   ↑     ↑
   p      q

m+n = p
na+mb = q

この連立方程式が解けてmとnが計算できるときに
置き換えができます。

>>627の場合は
置き換え前  (2x-1)/{(x+0)(x-1)}
置き換え後  m/(x+0) + n/(x-1) = {(m+n)x + (n*0+m*(-1))}/{(x+0)(x+(-1))}

m+n = 2
-m = -1

これを解くとm = n = 1なので
(2x-1)/{x(x-1)} = 1/x + 1/(x-1)と置き換えできます。
歓マ
633通りすがりのbunny_star:01/10/06 04:36
>>629
(2x-1)/{x(x-1)}=(A/x)+{B(x-1)}とおく。
(A/x)+{B(x-1)}={A(x-1)+Bx}/{x(x-1)}={(A+B)x-A}/{x(x-1)}だから、
係数を比較してA+B=2、A=1ゆえにB=1
従って(2x-1)/{x(x-1)}=(1/x)+{1(x-1)}
轄ユワショーイ
かぶりすぎやちゅーの。
636627:01/10/06 04:59
>>630
>>631
>>633
ご丁寧にありがとうございました。
12分に理解することができました。
637617:01/10/06 10:00
 > 数学記号で「波線が2本」というものがあると思うんですが、
 > この記号は、PC上でどのようにして入力すれば良いのでしょうか?

>>625
それです!
∬(二重積分)ではないです。縦に波線を重ねたものです。
すみませんが、その625の記号について、入力方法と意味を解説願えませんでしょうか?
231>>529
やっぱり1つずつやってくしかないみたいね…
>>638
おっと御礼を忘れていました。証明いただきありがとうございました。
>>605
もうちょっとてっとり早くて感覚的に分かりやすい証明。
点対称に180度ひっくり返すと、
真ん中の升以外は別の黒升に対応するので、それらの数は偶数個。
真ん中の升を含めると奇数個。
641132人目の素数さん:01/10/06 15:24
1/2(x√(a^2-x^2)+a^2arcsinx/|a|)の微分はどう解けばいいのですか?
>>641
それは

1/(2(x√(a^2-x^2)+a^2arcsinx/|a|))
(1/2)(x√(a^2-x^2)+a^2arcsinx/|a|)

のどちらの意味ですか?
643タリバン先生:01/10/06 18:50
>641
x√(a^2-x^2)をxで微分すると
√(a^2-x^2)-(x^2)/√(a^2-x^2)=(a^2-2 x^2)/√(a^2-x^2)
arcsinxをxで微分すると
1/√(1-x^2)だから

(x√(a^2-x^2)+a^2arcsinx/|a|)をxで微分すると

(a^2-2 x^2)/√(a^2-x^2)+a^2 / (|a|√(1-x^2))
=((1+|a|)a^2-2 x^2)/(|a|√(1-x^2))

(1/2)(x√(a^2-x^2)+a^2arcsinx/|a|) をxで微分すれば
((1+|a|)a^2-2 x^2)/(2|a|√(1-x^2))

1/(2(x√(a^2-x^2)+a^2arcsinx/|a|)) をxで微分する場合は

1/(2f(x))をxで微分すると
-f'(x)/(2f(x))^2に、
f'(x)= ((1+|a|)a^2-2 x^2)/(|a|√(1-x^2))
f(x)= (x√(a^2-x^2)+a^2arcsinx/|a|)
を入れて終わり
644ちむ教の信者:01/10/06 20:44
a,a,a,b,b,c,cの7文字を一列に
並べるときの総数は?

n!/p!q!r!・・・を
使うのはわかるんですけどなんでこれが使えるのかが
わかりません。理屈を教えてください。
お願いします。
645132人目の素数さん:01/10/06 20:46
642>>
説明不足ですみません。後者の問題です。この問題を微分すると答えが√(a^2-x^2)
になるのですが、解き方がわかりません。教えてください。
646132人目の素数さん:01/10/06 21:03
変な質問ですが、トランプでポーカーをしている時手持ちのカードが全てバラで
これからワンペアを作ろうとする場合一枚だけ変える場合・二枚変える場合と考えていって
ワンペアになる確率が一番高いのは何枚変えでしょうか?
そのカード手持ち含めジョーカー抜きの全52枚、他のカードの混ざり具合は不確定
>>644
例えばさ、

A1,B1,B2
の並べかたが3!通りなのはわかるかい?
で、
B1,B2
の並べかたが2!通りなのもわかるよね?

だったら、
A,B,B
の並べかたは3!/2!通りになるってのはわかる?
648132人目の素数さん:01/10/06 23:58
http://kaba.2ch.net/test/read.cgi/wres/1002308913/l50

に書いてある通りなんですが、
ある枚数の種類のトレーディングカードを集めるのに必要なカード購入枚数の
期待値はいくつになるのでしょう?
レアカードとかは考えなくてもいいので、お願いします。
649132人目の素数さん:01/10/07 00:14
>>646
ワンペアを包含する上位の役(ツーペア)はワンペアに含みますか?
650132人目の素数さん:01/10/07 00:27
おじゃまします、突然ですが問題です。
それぞれの□に0から9までの数字から5つを入れて答えが一番大きくなる時は?

           □□□
           × □□
         −−−−−
651名無し:01/10/07 00:31
>>650
875*96=84000
652132人目の素数さん:01/10/07 00:33
失礼ですが、それが最大となる理由を教えていただけますが?
653132人目の素数さん:01/10/07 00:33
が>か
654132人目の素数さん:01/10/07 00:35
>>650
999×99=98901
655132人目の素数さん:01/10/07 00:37
すいません。各数字一つずつです。
>>652
8□□×9□
9□□×8□
で全通り試してみるのが最も説得力があると思われ
>>656
全通りと言っても、4通り試せば十分なわけだが。
658132人目の素数さん:01/10/07 00:51
確認できました。ご指導ありがとうございます。
659132人目の素数さん:01/10/07 01:20
>>649
はい、含みます。
それとこの質問でフラッシュは抜きで数字のみの組み合わせでお願いします。
660 ◆pvySbQO2 :01/10/07 05:00
>>648
毎回確率aで起きることが起きるまでの平均回数は1/aなので
n種類でそれぞれ1/nの確率で出るなら全て出るまでの平均回数は
煤Q{1≦i≦n}(n/i)で
約(n・log(n)+0.5772・n+1/2)回です。
661 ◆pvySbQO2 :01/10/07 07:00
>>641
 d(x√(a^2−x^2))/dx
=√(a^2−x^2)+x(−2x)/2√(a^2−x^2)
=√(a^2−x^2)−x^2/√(a^2−x^2)。

y=arcsin(x/|a|)とする。
sin(y)=x/|a|。
cos(y)dy/dx=1/|a|。
(√(a^2−x^2)/|a|)dy/dx=1/|a|。
dy/dx=1/√(a^2−x^2)。

 d(x√(a^2−x^2)+a^2・arcsin(x/|a|))/dx
=√(a^2−x^2)−x^2/√(a^2−x^2)
 +a^2/√(a^2−x^2)
=2√(a^2−x^2)。
662132人目の素数さん:01/10/07 14:18
あげ 
663ななげ:01/10/07 16:03
工学部1回生です。火曜に提出する宿題をするために該当部分の教科書を読んでいて分からないことがあったので教えてください。
1、R^nの集合Aに属する任意の2点が、集合A内を通る曲線で結ぶことができる時、
集合Aは連結であるという
というのが良く分からないのです。イメージとして沸いてきません。
2、連続と滑らかとはどう違うのでしょうか?
お願いします。
664132人目の素数さん:01/10/07 16:28
>>663 1は弧状連結の定義だろ。その教科書おかしいよ。
連結であって弧状連結でない例:
A={(0,y)|-1≦y≦1}∪{(x, x sin x)|0<x≦1}
665ななげ:01/10/07 16:32
>>664
サイエンス社のテキストなんですが、おかしいんですか?
666132人目の素数さん:01/10/07 16:34
おかしい
667ななげ:01/10/07 16:36
>>666
工学部なんで、このへんの細かい言葉などは詳しく理解する必要はないでしょうか?
まあ、分かるにこしたことはありませんが。
668132人目の素数さん:01/10/07 16:43
君の問題じゃなくて著者が問題だ。
まともな人なら,「連結の定義」じゃなくて
「弧状連結の定義」と書くはずだ。
669132人目の素数さん:01/10/07 16:43
>>663
弧状連結のことを簡単に連結とかいてある本もある。
集合Aがつながっていないということの例を思い浮かべると
この定義はとても自然だと思いますが?
2の連続は関数についての概念ですがそれは了解してますか?
滑らかっていうのは微分可能、あるいは微分可能でかつ微分
した関数が連続であること。これは定義が教科書に書いてある
のが普通。ものの動きを時間の関数で記述すると、ゆっくり
考えればわかると思う。グラフを書いて、非連続だとどう動くか?
微分不可能だとどう動くか?などやってみること。
670ななげ:01/10/07 16:50
>>669
>2の連続は関数についての概念ですがそれは了解してますか?
それは分かってます。
滑らかについては、教科書には 納1,n]φ’_κ(t)^2 ≠0 って書いてました。
671664訂正:01/10/07 16:51
連結であって弧状連結でない例:
A={(0,y)|-1≦y≦1}∪{(x, x sin(1/x))|0<x≦1}
672664,671さらに訂正:01/10/07 16:57
連結であって弧状連結でない例:
A={(0,y)|-1≦y≦1}∪{(x,sin(1/x))|0<x≦1}

これでやっと・・・
なさけない。アセッテたみたいです。
673132人目の素数さん:01/10/07 17:06
>>670
教科書に書いてあるその式が何だかわからないが、その式のうち
何が滑らかなのかいえますか?主語と述語をよく押さえて本を読んで
下さい。また概念は「そうであること」「そうでないこと」両方
考えてみることが大切です。ともかく、669の定義で考えてみて
ください。
674ななげ:01/10/07 17:12
>>673
いろいろと助言ありがたいです。
納1,n]φ’_κ(t)^2 ≠0は
2変数の場合は、
C={(φ(t),ψ(t)):α≦t≦β}に対して、φ(t)とψ(t)はC^1級で、かつ
φ'(t)+ψ'(t)≠0となるとき、曲線Cは滑らかである ッてことです。
675 :01/10/07 17:35
>>637 >>625

『〜』を縦に二つ並べた記号は、
多分windows等の文字としては準備されていないと思う。
『≒』(大体等しい)と同じ意味、あるいは似た意味で使っている本が多い。
同値関係の記号が複数必要になったときに、そのうちの一つとして使っていることもある。
676132人目の素数さん:01/10/07 17:44
>>674
事情はわかりました。2変数の場合2乗がぬけています。
いいですか、曲線Cが滑らかっていうその定義は例えば、
物体が平面を動くとき時間の関数としてその動きを記述
したとき、「動きが滑らか」の定義です。動きというのは
関数そのものではありません。連続の場合はその関数の
連続性と同じになりますが、その場合「動き」です。です
から微係数がx座標とy座標が同時の0にならないという
条件がついています。これがないとどっかで止まって90度
方向転換ってことも起きますね。これ考えてみてください。
677ななげ:01/10/07 17:54
>>676
うっ、2乗が抜けてましたね・・・
微係数というのは、微分係数のことでしょうか?
文を読ませていただきました。
自分なりの解釈を書きたいと思いますので、間違いなどご教授お願いします。

滑らか:パラメータで表示できる。表示できないものは滑らかではない。
     ただし連続の場合はある。

あと、φ'(t)^2+ψ'(t)^2≠0で滑らかってのが良く分かりません。
・・・俺って全然ダメジャン。鬱
678533:01/10/07 18:03
>>550
サンクス。
679132人目の素数さん:01/10/07 18:08
>>677
微係数は微分係数と同じ、昔はそういったんだよ。
C^1級というのが微分可能でかつ微分した関数が連続ということ。
関数が滑らかというときはこのC^1級のことをいうのが普通だと
思う。この場合の主語は「動き」、その教科書では「曲線」だね。
それで、両方が同時に0となるっていう条件は点の動きが止まる
ってことだ。0にならないってことは動きつづけるってことだ。
微分不可能な関数や、この条件を落とすと直角や急ターンなんでも
ありで滑らかでなくなる。少し頭を冷やして、時間をかけて理解
しなさい。よくわかったら他の人によく教えてあげなさい。どうせ
皆んなわかっちゃいないんだから。
680ななげ:01/10/07 18:36
>>679
もうちょっとじっくり考えてます。
仰るとおり、たぶん友達どころか、クラスの99%が分かってないと思います。
工学部ってこともあるし。
681132人目の素数さん:01/10/07 19:36
680の「工学部ってこともあるし」ってコメントは無意味。
数学科だって同じだよ、上に書いた説明をキチット理解で
きる人は研究者になれる素質があるよ。
682ななげ:01/10/07 19:51
>>681
そうですね。心の奥にある甘えみたいなのが出ちゃいましたね。
頑張ってみます。また、どうしても分からない時は宜しくお願いします。
683132人目の素数さん:01/10/07 21:20
(1)log|x+√(x^2+A)|
(2)(1/2){x√(x^2+A)+Alog|x+√(x^2+A)|}
この二つの問題の微分の解き方がわかりません。
微分したらどうなりますか?教えてください。
684ななげ:01/10/07 21:46
>>683
1、1/√(x^2+A)
685ちむ教の信者:01/10/07 22:26
あの〜「×」、「・」を一緒に同じ式に
使っていいのですか?
例 3×4・2

また、720通りあるとき、これをいっぺんにいうときは
すべての場合の数というのですか?
686132人目の素数さん:01/10/07 22:53
×と・は別の記号です。今井数学では
687名無しさん:01/10/07 23:09
>>685
広義の積について「・」ってのを使いますね。
狭義の積(ただの掛け算)を「x」と書いたりします。

広義の積ってのは、対象にあるものを作用させるという意味で使ってますね。
例えば、(四則演算の)足し算ってのもこの積の1つです。
a・e=e・a=aとなるようなeを単位元といいますが、
この積を掛け算とした場合は、axe=exa=aってことですから、e=1が単位元です。
この積を足し算とした場合は、a+e=e+a=aってことですから、e=0が単位元です。
>>685-686
内積と外積(w
>>685
>また、720通りあるとき、これをいっぺんにいうときは
>すべての場合の数というのですか?

何が720通りあって、何をいっぺんに使うの?

>>644の延長線上にあるのだとは思うけど。
690648:01/10/07 23:23
>>660
正直、何故そうなるのかは飲み込めていませんが
どうもありがとうございました。自分でも考えてみます。
691 ◆pvySbQO2 :01/10/08 00:00
>>648
確率aで起きる出来事が起きたo,起きなかったxと表すと
xxxoは4回で起きoは1回で起きxxxxxxoは7回で起きています。
これらを並べていくと
xxxooxxxxxxo...xxo
のようになり全体の回数をm回とするとoはam回で
起きるまでの平均回数はm/am=1/a回になります。

n種類でそれぞれ1/nの確率で出るものが残りi種類のとき
i種類の中から出る確率はi/nなので平均回数はn/i。

残りn種類から残りn−1種類までの平均回数はn/n回。
残りn−1種類から残りn−2種類までの平均回数はn/(n−1)回。
...
残り1種類から残り0種類までの平均回数はn/1回。
なので全て出るまでの平均回数は
n/n+n/(n−1)+...+n/1回。
692132人目の素数さん:01/10/08 00:24
>>605
>>640がピンとこなかったので、もすこしわかるのを。

証明:

@マス座標を左上を(1,1)、右下を(N,N)(Nは盤のサイズ)とする。
どっちがX方向かY方向かは気にしない。
以下、X座標とY座標の和を「ポジション値」と表記する。

A一行一列一個ずつを守って置くとき、
すべての石のX座標を合計すると1+2+…+N、
すべての石のY座標を合計すると1+2+…+N、
上2つをあわせたものは、石のポジション値の合計に他ならない。
当然その値は2(1+2+…+N)であり偶数。

B石のポジション値の内訳に注目。
奇数個の整数の合計が偶数のときその内訳は
偶数が奇数個、奇数が偶数個でなければならない。
つまりポジション値が偶数の石が奇数個あることになる。

Cポジション値が偶数[奇数]のすべてのマスは
黒[白]マスであることは明らか。

−−−−−−−−終了−−−−−−−−
693132人目の素数さん:01/10/08 00:29
>>659
計算上ストレートも無視したいでーす。
694132人目の素数さん:01/10/08 03:13
ちょっと表現の質問です。(Q(α,β)は代数体を表します)

「K=Q(α,β)に対し、正整数qをqα,qβが”Kの整数”になるようにとる。」

”Kの整数”って一体?どうかお教え願います。
695132人目の素数さん:01/10/08 04:53
>694
代数体Kの整数環の元のことじゃないの?
696ななげ:01/10/08 13:01
次の関数の原点(x,y)=(0,0)における連続性と偏微分可能性について調べよ
f(x,y)=xy^2/(x^2+y^2),(x,y)≠(0,0),f(0,0)=0
って問題で、(x,y)≠(0,0)なのにf(0,0)=0っていうのが意味わかりません。
問題も原点について調べろって書いてあるのに、,(x,y)≠(0,0)なんでしょ??
697132人目の素数さん:01/10/08 13:53
>>684
途中の式はどうすればいいのですか?
698タリバン先生:01/10/08 14:01
>696
>f(x,y)=xy^2/(x^2+y^2),(x,y)≠(0,0)


(x,y)≠(0,0)のときf(x,y)=xy^2/(x^2+y^2)
(x,y)=(0,0)のときf(x,y)=0
と読む
699ななげ:01/10/08 15:37
>>698
water!タリバン先生ありがとう!
なんとなくそうかなーとは思ってたんです。
700コリオリ:01/10/08 15:48
z=f(x,y),x=x(t),y=y(t)のとき、d^2z/dt^2を求めよってやつが分かりません。
合成関数の微分っていうのは分かるんですが・・・
おいらの解答を書くので、どこが間違ってるのか教えてください。

dz/dt=f_x(dx/dt)+f_y(dy/dt)

d^2/dt^2=d/dt{f_x(dx/dt)+f_y(dy/dt)}
      =(∂/∂x){f_x(dx/dt)+f_y(dy/dt)}(dx/dt)+(∂/∂y){f_x(dx/dt)+f_y(dy/dt)}(dy/dt)
      =f_xx(dx/dt)^2+2f_xy(dx/dt)(dy/dt)+f_yy(dy/dt)^2

ってなったんですが、答えとあわないんです・・・
701タリバン先生:01/10/08 16:31
>>700
例えばf_x(dx/dt)をちゃんと書くと
f_x(x(t),y(t)) (dx(t)/dt)
なのでf_x(x(t),y(t)) というtの関数と(dx(t)/dt)というtの関数の積のことだから
これをtで微分すると積の微分公式により
d/dt{f_x(dx/dt)}={d/dt f_x}(dx/dt)+f_x(d^2x/(dt)^2)}
で右辺第二項の2回微分がでるあたりの計算が足りません。

ちなみに

>d/dt{f_x(dx/dt)+f_y(dy/dt)}
>=(∂/∂x){f_x(dx/dt)+f_y(dy/dt)}(dx/dt)+(∂/∂y){f_x(dx/dt)+f_y(dy/dt)}(dy/dt)

この等号が間違いで、{f_x(dx/dt)+f_y(dy/dt)} に含まれる文字はx,y,(dx/dt),(dy/dt)の四つ
(dx/dt)はxで表されているわけではないことに注意!
この書き方で行くのであれば(dx/dt)をv、(dy/dt)をwと置いて、x,y,(dx/dt),(dy/dt)は全て独立な変数だと思って
d/dt{f_x v +f_y w}
=(∂/∂x){f_x v+f_y w}v+(∂/∂y){f_x v +f_y w}w +(∂/∂v){f_x v+f_y w}(dv/dt)+(∂/∂w){f_x v +f_y w}(dw/dt)
とする。
702132人目の素数さん:01/10/08 16:37
教えてください!

環RのイデアルIは環になるとは限らないのでしょうか?
イデアルが環になる事も示せず、
イデアルであって環でないものの例も見つからずに困っています。
703コリオリ:01/10/08 16:40
>>701
う〜ん、さすがです。
ありがとうございました。本当に良く分かりました。
704タリバン先生:01/10/08 16:53
>702
あなたの言っている「環」の定義に積の単位元1の存在が入っているならば
イデアルに単位元が入ってしまうとイデアルの定義により自明なイデアルRになってしまうので
イデアル{0}及び自明で無いイデアルには積の単位元は無く「環」ではない

「環」の定義に積の単位元の存在を仮定しない場合は、イデアルは「環」と言っても構わない
705132人目の素数さん:01/10/08 17:04
>>704
有難う御座います!
私は環の定義に単位元1の存在を含めていないのですが、
イデアルが環である事を上手く示せません。
加法に関する逆元の存在以外は全て示せたのですが、
加法に関する逆元の存在だけがどうしても示せません。
どうやって示せば良いのでしょうか?
706名無し:01/10/08 17:14
・n,m共に自然数で4f(n,m)=f(n+1,m)+f(n-1,m)+f(n,m+1)+f(n,m-1)
全てのn,mに対しf(n,m)は実数でf(n,m)≧0のとき、
全てのn,mに対しf(n,m)はある定数cとなることを証明せよ。

っていう問題,みんな覚えてる?
で,さっき,『f(n,m)=n+mとすると条件を満たすから題意は成り立たない』
って書いてあるのを受験板で見つけたんだけど,どう思う?
>>705
加法に関する逆元の存在は普通イデアルの定義に含まれている
と思うんですけど・・・
あなたの本のイデアルの定義を書いてみてください。
708タリバン先生:01/10/08 17:16
>705
定義によってイデアルは加群として環の部分加群だよ?

あるいは0の存在は 0∈R、a∈I → 0・a ∈Iで分かるから
Iが加法によって閉じているならば逆元は環の加法として持つ筈だけれど?
709名無し:01/10/08 17:35
>>706
どう思う?
710132人目の素数さん:01/10/08 17:43
>>707-708
教科書にはこう定義してありました。

R:環
I:Rの部分集合

Iがイデアル
(1)a,b∈I ==> a+b∈I
(2)a∈I,r∈R ==> ra,ar∈I

0∈Iは分かります。
任意の元a∈Iを取ってきた時に
-a∈Iである事をどうやって示せば良いのかが良く分からないのです。
>>706
>n,m共に自然数で
これならこれで正しいよ。
難しいのは m,n が整数のときだ。
Rが単位環なら(2)でOK。
普通は(1)が (1)a,b∈I ==> a-b∈I
となってると思うんだけど・・・
713同型定理の証明で:01/10/08 17:50
Gを群
HをGの部分群
NをGの正規部分群
f:G∋g→gN∈G/Nとする。
このときIm(f|H)=HN/Nになることがよくわかりません。
(f|HはfをHに制限するという意味。)
誰か詳しくおしえてくれませんか?
Im(f|H)=H/Nのような気がするんですけど。。。ちがいますか?
714名無し:01/10/08 17:51
・n,m共に整数で4f(n,m)=f(n+1,m)+f(n-1,m)+f(n,m+1)+f(n,m-1)
ってのは,直感的には,xyz空間ですべての点(n,m,f(n,m))が
同一平面上にあるってことだよね。これがきちんといえれば,
証明はやさしいんだけど。
715132人目の素数さん:01/10/08 17:55
>>712
有難う御座います。
確かに
(1)a,b∈I ==> a-b∈I
であればすぐに示せます。
教科書に問題があると言う事でしょうか?
でも、こっちの定義は今まで聞いた事が無いです・・・。
>>712
ちがうよ。
たとえば f(m,n)=mn
717716:01/10/08 17:58
まちがい
>>714
ちがうよ。
たとえば f(m,n)=mn
718タリバン先生:01/10/08 18:51
>715
部分群の定義って(もちろん単位元はあるけど)
a,b∈H ⇒ a・b∈H
a∈H ⇒ a^(-1)∈H

だけど

この2つの条件と同値な条件として
a,b∈H ⇒ a・b^(-1)∈H

というのがある
加法で言えば
a,b∈H ⇒ a-b∈H

ということ
単位環でない場合はイデアルが部分加群であることを示すには
>(1)a,b∈I ==> a+b∈I
>(2)a∈I,r∈R ==> ra,ar∈I

では不十分だと思う。
719 ◆pvySbQO2 :01/10/08 19:00
>>710
教科書にそう書いてあるのならその教科書は間違っています。
R={X・f(X)|f(X)∈Z[X]}
I={nX+X^2・f(X)|n∈Z,0≦n,f(X)∈Z[X]}
とするとa∈I=>−a∈Iが成り立ちません。
720132人目の素数さん:01/10/08 19:05
>713
>Im(f|H)=H/Nのような気がするんですけど。。。ちがいますか?

H⊇Nでない場合もそう思いますか?
>>719
あんたよく考えたね。暇人?
でもこれではっきりしたのはお手柄。

たぶん (1)a,b∈I ==> a-b∈I のミスプリだったんでしょうね。
722132人目の素数さん:01/10/08 19:49
(1)(1/2){x√(x^2+A)+Alog|x+√(x^2+A)|}
この微分がどうしてもわかりません。答えは√(x^2+A)になるのですが、解き方が
わかりません。教えてください。
723132人目の素数さん:01/10/08 19:50
>>718-719,>>721
有難う御座います。

つまり、教科書が間違っていて、
イデアルは環であることが一般に成り立つ、と言う事ですね?
724132人目の素数さん:01/10/08 20:03
>>722
P=(1/2){x√(x^2+A)+Alog|x+√(x^2+A)|}
d(2P)/dx=√(x^2+A)+x^2/{√(x^2+A)}+A*(1+x/{√(x^2+A)})/{x+√(x^2+A)}
第三項をQと置く。
Q=A*(1+x/{√(x^2+A)})/{x+√(x^2+A)}
=A*(1+x/{√(x^2+A)})*{x-√(x^2+A)}/({x+√(x^2+A)}*{x-√(x^2+A)})
=A*(1+x/{√(x^2+A)})*{x-√(x^2+A)}/{x^2-(x^2+A)}
=-1*(1+x/{√(x^2+A)})*{x-√(x^2+A)}
=-1*{x+x^2/{√(x^2+A)}-√(x^2+A)-x}
=√(x^2+A)-x^2/{√(x^2+A)}

よって
d(2P)/dx=√(x^2+A)+x^2/{√(x^2+A)}+√(x^2+A)-x^2/{√(x^2+A)}
=2√(x^2+A)

故に
d(P)/dx=√(x^2+A)
725同型定理の証明で:01/10/08 20:37
>>729
Im(f|H)=H/Nではないですね。。。
ではIm(f|H)=HN/Nを示せばよいのでしょうか??
726同型定理の証明で:01/10/08 20:38
>>720
↑のまちがい。
727:01/10/08 20:41
>>720
のまちがい。
728132人目の素数さん:01/10/08 20:47
『^』この記号ってどんな意味で使ってるの?
729132人目の素数さん:01/10/08 20:59
>724
ありがとうございます
730 ◆pvySbQO2 :01/10/08 22:00
>>721
このくらいの例なら考えるのに5分もかからないでしょう。
731132人目の素数さん:01/10/08 22:10
>>728
x^y で xのy乗の意
732132人目の素数さん:01/10/08 22:13
事象AとBが独立、P(A)=1/3  P(AまたはB)=3/4 のとき
P(AでないまたはBでない)=を求めよ

=1−P(AかつB)
=1−P(A)P(B)

ってのはわかるんだけど P(B) っていくら?
733:01/10/08 22:37
P(AまたはB)=1-(1-P(A))(1-P(B))=3/4
より P(B)=5/8
734:01/10/08 22:43
ありがと〜
735整数論:01/10/08 22:50
頭の良い方お願いします。(解説ではなく解答で)

1.(1) (a,15)=1 ならば、
       a{φ(15)/2}≡1 (mod 15)
したがって15は原始根をもたないことを示せ。

  (2) 21は原始根をもたないことを示せ。(前問参照)

  (3) 35は原始根をもたないことを示せ。(前問参照)

2.pは素数で、p≡1 (mod 4)、gをpの一つの原始根とする。
   g{(p-1)/4}は合同式
             x^2≡-1 (mod p)
の一つの解であることを示せ。

3. x^2 +1≡0 (mod 65)
には、4個の解がある。(mod 5)及び、(mod 13)でこの合同式を解き、
   中国の余剰定理を利用して、これら4個の解を求めよ。


数学が得意な方お願いします。m(_ _)m
736整数論:01/10/08 23:03
4. 合同式 x^10 ≡1 (mod 14) のすべての解を求めよ。

5. 3が31の原始根であることを知って、
       3^5 , 3^10 , 3^15 , 3^20 , 3^25 , 3^30
は合同方程式
         x^6≡1 (mod 31)
の6個の異なる根であることを示せ。
  また、
    (x^6-1)=(x^3-1)(x^3+1)=(x-1)(x^2+x+1)(x+1)(x^2-x+1)
であるから、上の6個の数は、合同式
     (x-1)(x^2+x+1)(x+1)(x^2-x+1)≡0 (mod 31)
   を満足する。どの解がどの因子に対応するか(できれば、解を因子に
    代入しないで調べよ)

6. nが原始根をもつならば、nはちょうどφ(φ(n))個の原始根をもつことを
   示せ。

数学が得意な方お願いします。m(_ _)m
全部わかったけど、俺ヴァカだからやめとく
738132人目の素数さん:01/10/08 23:26
>>695
ふ〜む、まず「整数環」から調べてみます。
どうもでした。
739132人目の素数さん :01/10/08 23:29
ヴァカ?オレなんかもっとヴァカ。
でも一つわかった。
>>735 1.φ(15)=8,a^4≡1(mod15)を示せばいいんでないか?
        a^4≡1(mod3) a^4≡1(mod5)てなかんじで。
740132人目の素数さん:01/10/08 23:31
簡単な不定積分なのですが、
∫cos^2 x sin^3 x dx
∫1/√(x^2 +1) dx
∫e^x sinxcosx dx
この三つの積分の最も美しくかつ速い解き方は何ですか?
741132人目の素数さん:01/10/08 23:37
>>706

>『f(n,m)=n+mとすると条件を満たすから題意は成り立たない』
>って書いてあるのを受験板で見つけたんだけど,どう思う?
その意見は正しい。問題は間違っている。多分写し間違いだろう。

>・n,m共に自然数で4f(n,m)=f(n+1,m)+f(n-1,m)+f(n,m+1)+f(n,m-1)
>全てのn,mに対しf(n,m)は実数でf(n,m)≧0のとき、
>全てのn,mに対しf(n,m)はある定数cとなることを証明せよ。
×自然数 → ○整数 だとおもわれ。
742132人目の素数さん:01/10/08 23:38
x^4+x^2+1=?
x^2-1/4x+1/4=?

上の因数分解の解き方を教えてください。
743741:01/10/08 23:39
しまった。>>711 を見逃していた。スマソ。
744737:01/10/08 23:40
>>739
ヴァカはお呼びでないらしいよ、お互い(w
>>742
複二次式

x^4+x^2+1=(x^4+2x^2+1)-x^2=(x^2+1)^2-x^2=略
x^2-1/4x+1/4=略
746132人目の素数さん:01/10/08 23:48
>>742
x^4+x^2+1=x^4+2x^2+1-x^2=(x^2+1)^2-x^2=(x^2+1+x)(x^2+1-x)

x^2-1/4x+1/4は、実数の範囲では解けないから、
二次方程式を解いて、(x-α)(x-β)とするしかない。
747132人目の素数さん:01/10/08 23:48
>>745
カブッタ
748132人目の素数さん:01/10/08 23:59
>>745-746
ありがとうございます、考えてみて、どうしても分からない時はまた来ます。
749132人目の素数さん :01/10/09 00:05
初歩的な問題(らしいの)だが、いくら考えても答えと合わないんだ。
答えが分かってる場合って、書いたほうがいいかな?

男子 A、B、C 3人と、女子 d、e、f、g、h 5人がいる。
この8人を横に一直線上に並べるとして、次の並び方の総数を求めよ。

1.少なくとも一方に男子が来る並び方
2.男子が絶対に隣り合わない並び方
3.右から数えて、奇数番に女子が来る並び方
750749:01/10/09 00:07
>>749
ちょっと1が意味不明だったな。
正しくは、

1.少なくとも一方の端に男子が来る並び方

だわ。両端に来てもいいってことね。
>1.少なくとも一方に男子が来る並び方

ワケ ・ ワカ ・ ラン
752751:01/10/09 00:08
負けた(ワラ
753はなう:01/10/09 00:18
>>749
1.3*7!+5*3*6!=25920
2.6C3*3!*5!=14400
3.ありえなくない?
>>753
女子が奇数番に来るでなくて奇数番に女子が来るなので
AdBeCfghなどがある。
>3.ありえなくない?

× 女は必ず奇数番
○ 偶数番に女が入っても可

・・・じゃなかっぺ?
756132人目の素数さん:01/10/09 00:34
>>753
もう何人かの方がレスしてくれているが、3番に関しては
その通りです。
1、2についてはありがとう! 答え合ってます。すげぇや。
しかし1の答えは納得できたんだが、2の6C3ってのはどっから出てくるの?
757:01/10/09 00:35
>>749です。
>>756
□女□女□女□女□女□
759749:01/10/09 00:54
>>758
分かった! ありがとう!
760はなう:01/10/09 01:20
>>754-758
なるほど、わしゃバカだのう
じゃあ、3.は、4C3*3!*5!=2880 先いっとくと4C3は偶数番から男子が入るのをえらぶので。
761132人目の素数さん:01/10/09 01:44
>735
とりあえず6だけ(他は簡単そう)
(Z/nZ)^*=Z/φ(n)Z だから(原始根が存在するから)
これより、右辺で考えれば、(a,φ(n))=1 のとき、amodφ(n)は右辺の原始根になるから
この個数は、φ(φ(n))

ちなみにこのレベルの整数論なら
「初学者のための整数論」Aヴェイユ著、現代数学社 がお勧め
762132人目の素数さん:01/10/09 02:04
>725
まず群の剰余というものの定義を考えてみましょう
G/Hというのは群Gの部分群Hを持ってきて
群GをaH、bH、… (a、b、…∈G)という集合に分けようということです。
元で見れば、aHとbHと…を全部合わせたものはGになります。
では、NH/Nの意味は?と言えば
f:G∋g→gN∈G/N
の行き先のgNを、g∈Hの時に全部集めたら?というのがN・Hという群になりますよね?
この群の元をaNという形の集合に切り分けて類別しなさい。そうすれば
改めてaNというのを元だと思えば、それらは群になりそれをNH/Nと書きましょうと
いうことです。
763132人目の素数さん:01/10/09 02:29
手元の本に載っていないのでお聞きしたいのですが・・・。
αが代数体Q(θ)(次数はn)の整数とは
α=c_{0}+c_{1}θ+…+c_{n-1}θ^{n-1} (c_{i}∈Z)
でいいのでしょうか?
解釈の誤り等をご指摘いただきたく思います。
764132人目の素数さん:01/10/09 02:42
>763
Q(θ)の整数環の元のことでは?<Q(θ)の整数
765132人目の素数さん:01/10/09 02:58
>>740
◆ ∫(cos x)^2 (sin x)^3 dx ◆
これは3倍角の公式や5倍角の公式(←加法定理から自分でつくる)で次数を1まで落としてから積分する方法と
t=tan(x/2)と置換して解く方法があるが少々面倒くさいので以下の漸化式を用いて求める。

I(m,n)=∫(sin x)^m (cos x)^n dx (但しm,nは負でない整数)とする。
n≧2のとき部分積分より
I(m,n)={1/(m+1)}(sin x)^(m+1) (cos x)^(n-1) + {(n-1)/(m+1)} ∫(sin x)^(m+2) (cos x)^(n-2) dx ……@
(※ ∫(sin x)^m (cos x)^n dx=∫{(sin x)^(m+1)/(m+1)}'(cos x)^(n-1) dx と考えて部分積分した)
ここで上式右辺の第2項は
(sin x)^m {1-(cos x)^2} (cos x)^(n-2)=(sin x)^m (cos x)^(n-2) − (sin x)^m (cos x)^n
と変形できるから@式は
I(m,n)={1/(m+1)}(sin x)^(m+1) (cos x)^(n-1) + {(n-1)/(m+1)}{I(m,n-2)−(m,n)}
となりこの式より
I(m,n)={1/(m+n)}(sin x)^(m+1) (cos x)^(n-1) + {(n-1)/(m+n)}I(m,n-2)  (但しn≧2)……A
同様に
I(m,n)=−{1/(m+n)}(sin x)^(m-1) (cos x)^(n+1) + {(m-1)/(m+n)}I(m-2,n)  (但しm≧2)……B
(※ ∫(sin x)^m (cos x)^n dx=∫(sin x)^(m-1) {−(cos x)^(n+1)/(n+1)}' dx と考えて部分積分した)
この二つの漸化式ABを用いて求める。

∫(cos x)^2 (sin x)^3 dx=I(3,2)
               =(1/5)(sin x)^4 cos x + (1/5)I(3,0) (∵A)
ここでI(3,0)はB式より求めて上式に代入。
以下省略。
766132人目の素数さん:01/10/09 02:58
>>740
◆ ∫1/√(x^2 +1) dx ◆
t=tan(x/2)と置換して解く。
半角の公式 {tan(x/2)}^2=(1-cos x)/(1+cos x) を用いれば奇麗な形になって積分できる。
以下省略。

◆ ∫e^x sin x cos x dx ◆
∫e^x sin x cos x dx=(1/2)∫e^x sin 2x dx なので(倍角の公式)以下 ∫e^x sin 2x dx を求める。
ここで、
(e^x sin 2x)'=e^x sin 2x + 2e^x cos 2x ……@
(e^x cos 2x)'=e^x cos 2x − 2e^x sin 2x ……A
となるから @−2×A を計算すると
(e^x sin 2x)'−2(e^x cos 2x)'=5e^x sin2x となり
e^x sin 2x={(e^x sin 2x−2e^x cos 2x)/5}'
上式の両辺をxについて積分すると
∫e^x sin 2x dx=(e^x sin 2x−2e^x cos 2x)/5 +C (C:積分定数)
となり、あとはこれに 1/2 を掛ければ答。
以下省略。
767765 :01/10/09 03:06
つーか、∫(cos x)^2 (sin x)^3 dx くらいだとそのまま部分積分2回した方が速いかな?
わざわざ漸化式つくらんでも…。
∫(cos x)^5 (sin x)^4 dx くらいだったら圧倒的に漸化式使うのが速いけど…。
>∫cos^2 x sin^3 x dx

cosx=t
∫((cosx)^2(sinx)^3)dx=∫t^2(t^2-1)dt=略
769>763:01/10/09 03:13
764の言ってる通りだが、つまり
αがQ(θ)の整数
⇔α∈Q(θ)かつθはあるf(x)∈Z[x]の根
770>763:01/10/09 03:20
ちなみに763さんの解釈は間違いで
不分岐とか完全分解の時は正しかった気がする(調べる気なし。。。)
あ、違うわ。。。なんか局所化した時は同じかな。。。
でも円分体の場合は正しい.
が、簡単な例で、2次体、Q(√M)をとったとき、
M≡1(mod4)の時、整数環はQ[(1+√M)/2]になる.
771132人目の素数さん:01/10/09 03:23
>763
教科書を変えなさい。
772数学すき:01/10/09 07:35
非常に任意とか言葉がでてくるとやっかいですね 大数に次のような問題がでていました
------------------------------------------------------------------
(1)任意の自然数nについて『anが偶数ならば』、aは偶数である
(2)任意の自然数nについて、『anが偶数ならば』aは偶数である
(1)(2)の命題の真偽をたしかめよ
-------------------------------------------------------------------------------
(1)(2)を見比べてください 一見同じようにみえますが 点の位置がちがうのです
そしてこの位置によって意味が全然ちがってくると解答にはにはかいてあります
ちなみに
解答
(1)はn=1の場合からaは偶数が導かれ真であり
(2)はn=2のときaが奇数でも『anは偶数』となるので偽

なのだそうです。いってることがわかりそうでわからないのです
頭の中は?????です 助けてください
773名無しさん:01/10/09 07:52
(1)は真っての明らか。
(2)だけど、「任意の自然数nについて」という条件はどこにいったんだ?w
  n=2という特別な条件のもとで偽になるから命題は偽ってのはおかしいぞ。

大数も落ちたもんだなーに一票。
774132人目の素数さん:01/10/09 07:54
>>772
マルチポストはやめろに1票
>>772
数学の部屋掲示板で答えてもらいなさい。
>>773
n=2のとき成り立たないから
任意の自然数nについても成り立たないので
>>772 でいいんだよ。
777132人目の素数さん:01/10/09 08:05
それよりaって何?
778132人目の素数さん:01/10/09 08:14
大数…もともと落ちるほどの高さは無い

っつーか、たまにいい年もあるけどな
779正解:01/10/09 09:21
------------------------------------------------------------------
(1)任意の自然数nについて『anが偶数ならば』、aは偶数である
(2)任意の自然数nについて、『anが偶数ならば』aは偶数である
(1)(2)の命題の真偽をたしかめよ
-------------------------------------------------------------------------------
nは自然数とする。
(1) ⇔ 『anが偶数』という命題がすべてのnで成り立つならば,aは偶数である。
(2) ⇔ 『anが偶数ならばaは偶数である』という命題が,すべてのnで成り立つ。
>>772 は点の位置ではなく,『』のつけ方が悪いからこんがらがるんだよ。
(1) はnが奇数のときを考えばaは偶数でなければならず,真。
(2) はnが偶数のときはaが奇数でもよいから,偽
>>773

おまえだいじょうぶか?
781132人目の素数さん:01/10/09 12:34
頭の体操です。誤植ではありません。
「Nが4で割って1あまる偶数とすると、Nは3の倍数である。」
この命題の真偽は?
782なし:01/10/09 12:40
「Nが4で割って1あまる偶数」は矛盾です。
矛盾した仮定からは真偽を判定することができません。
>>782
基礎論的に言えば、
矛盾した仮定から導かれる命題は、全て真
784132人目の素数さん:01/10/09 12:48
>>691
648さんではないのですが、質問してもよろしいでしょうか?

>残りn種類から残りn−1種類までの平均回数はn/n回。
>残りn−1種類から残りn−2種類までの平均回数はn/(n−1)回。
>...
>残り1種類から残り0種類までの平均回数はn/1回。
>なので全て出るまでの平均回数は
>n/n+n/(n−1)+...+n/1回。

この前半までは納得できたのですが、後半が分かりません。
AがBになるまでの平均回数とBがCになるまでの平均回数の和は
AがCになるまでの平均回数になる、という事を使っていると思うんですが
これは証明しなくていいんでしょうか?それとも当たり前の事なのでしょうか?

数学オンチですので、なるべく分かりやすく教えていただけましたら幸いです。
785qazx:01/10/09 17:23
∫{1/(cosθ)^3}dθ が分かりません。誰か教えて下さい。
786なし:01/10/09 17:37
1/(cos(x))^2= 1/(1−(sin(x))^2)
と部分積分の公式を使えば、sin(x)の式になるはず。
そこで d/dx[cos(x)] = −sin(x) を使う。
間違っていたら済みません。
787名無し:01/10/09 17:42
>>785
sin θ=t とおいて,置換積分。
788132人目の素数さん:01/10/09 18:29
すいません、速報板からきたのですが
紙は6、7回以上おれない、というのを
数学的に証明することは可能でしょうか?
789132人目の素数さん:01/10/09 18:37
>788
物理とか化学じゃないの?
数学の紙は厚さが無くて体積を持たないから何度でも折れるんだけど
790132人目の素数さん:01/10/09 18:44
>>789
なるほど、一応物理学板にも今かいてみました
ティッシュでも厚紙でもそうなるようなので、厚さは関係ないのかな
とおもったのですがどうなんですかね
791132人目の素数さん:01/10/09 18:54
>790
ダンボールを6、7回折った人は見たことアリマセン
792名無し:01/10/09 18:55
>>788
新聞紙くらいの大きさなら,6回は折れるよ。
大切なのは,1回折ると面積が半分になるから,10回も折るのはかなり大変てことよ。
有名な話で,
『厚さ0.1mmの紙を100回折ると,厚さはどのくらい?』
っていうのがあって,130億光年くらいになるんだけど,実際には100回も
紙を折れるわけがないんだよね。そういう話では?
793132人目の素数さん:01/10/09 19:00
体育館くらいの紙でも6、7回が限度みたいです
不思議
794132人目の素数さん:01/10/09 19:49
>>793
なんで不思議なの?
折り目以外は関係ないのだからただ重なっているだけだったりするところは無意味なんだよ
795132人目の素数さん:01/10/09 20:01
どこから6,7回という数字がはじき出されるのでしょう
796132人目の素数さん:01/10/09 20:01
>>795
実験からでは
「探偵ナイトスクープ」で前にやってた。
体育館いっぱいの紙で
たしか9回か10回折れてたと思う。
798132人目の素数さん:01/10/09 21:31
他スレでも何度も書かれてる質問だけど・・・
1+1は何故2なの?1がふたつじゃどうしていけないの?
799名無し:01/10/09 21:43
>>798
じゃあ,そもそも1って何?
2って何?
800名無し:01/10/09 21:47
>>795
実際のところ,7回折ると面積は1/128になり,厚さは128倍なる。
つまり,面積は急激に小さくなるのに,厚さがどんどん大きくなる。
したがって,どんどん折りにくくなるんだね。
始めがよほど薄くてどでかい紙じゃないと,10回も折るのは大変。
ちょっとさば読んで,6〜7回ってことじゃないかな。
801>:01/10/09 22:24
その何回折れるか、と言う問題は、数学の問題にしても大して面白くないし
802798:01/10/09 22:33
>>799
1ってひとつだよね。これは理解できるんだけど2以降がわかんないんだよ。
ひとつとひとつっていう概念じゃなくてなんで2になるの?
803132人目の素数さん:01/10/09 22:51
>>802
「2になる」んじゃなくて、それを「2とする」んじゃないかな
804570:01/10/09 23:16
あの、どなたか>>570に対してヒントを頂けないでしょうか?

集合Aが有限集合だとして要素の数をnとして、
帰納法使えば何とかなりそうだったのに途中で挫折しました
ちょっとした手がかりでもとてもありがたいのでどなたか手助けを…
805132人目の素数さん:01/10/09 23:41
たぶん>>798は「2とする」という部分にひっかかってると思はれ。
どうして2を2としたのかの答えを知りたがってると思はれ。
これは禅問答みたいなもので永遠に答えは出ないと思はれ。
ttp://www.iit.edu/~maxiori/apps/Theseus.html
お願いです。誰かこれをクリアーして下さい。 数学とは関係ないん
ですが、ここにいるみなさんなら出来ると思いまして。ほんと、暇だ
ったらでいいんで、お願いします。
>>804
一変数にレベルダウンして、
2*f(m)=f(m+1)+f(m-1) なら簡単なんだよね。
f(m+1)-f(m)=f(m)-f(m-1)と変形すれば良いのだから。
これを、どう一般化するか・・・。

f(m,n)=am+bn+cなら、問題の方程式は満たすよ。
ただ、f(m,n)≧0 の条件をつけると、a=b=0になってしまうね。

あんまり役立たない答えでスマソ(俺も、よく分からんのだ、この問題)
808804:01/10/10 00:10
>>807
確かにその問題も難しいのですが
>・f(n,m)≡f(n+1,m)+f(n-1,m)+f(n,m+1)+f(n,m-1) (mod 2)
>・全てのn,mに対しf(n,m)は0か1
>(n,mは整数)
570は↑のように少し変えてあるんです。
んでこういうf(n,m)が

>(n,m)によって構成される集合Aを与えた時に
>とある整数k,lが存在し、Aの全ての要素(n,m)に対しf(n+k,m+l)=f(n,m)となっている。
これが証明できなくて困っております。
804でもう一度f(n,m)の条件を書くべきでした、すいません
809132人目の素数さん:01/10/10 00:11
>>693
遅れました、それも除いてください。
810132人目の素数さん:01/10/10 00:35
SO(2,R)はO(2,R)の指数2の部分群であり、O(2,R)/SO(2,R)
の代表系は、M=[[M[1,1],M[2,1]],[M[1,2],M[2,2]]] I=[1,0],[01]]and
I=[[1,0],[0,-1]]である、と本に書いてあったのですが、これを示す
方法を教えてください。よろしくお願いします。
811132人目の素数さん:01/10/10 03:20
   ベルトランの逆理

理解したと自分を納得させつつ、はや7年
>>570
「数学的帰納法などを使う」ために「問題を解く」ではなくて
「問題を解く」ために「数学的帰納法などを使う」なので
最初にどういう方法でと決めてしまうのはおかしいです。

問題を考えるためにはまずいろいろ試してみることから始めるべきです。
この問題だとfとしてどういうものがあるか
自分の手でいろいろ組み立ててみることです。
そうすれば(k,l)≠(0,0)ならば条件を満たすk,lは
存在するとは限らないということが分かるはずです。
813名無し:01/10/10 03:42
>>735

2だけ解答

[g^{(p-1)/2}-1]*[g^{(p-1)/2}+1]≡g^(p-1)-1≡0 (mod p)

pは素数より、g^{(p-1)/2}-1または、g^{(p-1)/2}+1がpで割り切れる。

g^{(p-1)/2}-1はpで割り切れると仮定する。

すると、g^{(p-1)/2}≡1 (mod p)となって、gがmod pでの原始根である事に反する。

ゆえに、g^{(p-1)/2}+1がpで割り切れる。つまり、g^{(p-1)/2}≡-1 (mod p)

[g^{(p-1)/4}]^2≡g^{(p-1)/2}≡-1 (mod p)

つまり題意が言えた。
814名無し:01/10/10 04:12
6だけ解答

mod nでの nの原始根をgとする。

0<r≦φ(n)なる自然数rに対して

g^rが、(g^r)^k≡1 (mod n)となる最小の自然数 kがk=φ(n)である ⇔
rとφ(n)は互いに素

を示せば良い。


もしそうでないと仮定する。

rとφ(n)の最大公約数をsとする。ただし、s>1

(g^r)^{φ(n)/s}≡(g^{φ(n)})^(r/s)≡1 (mod n)

これは、g^rが、(g^r)^k≡1 (mod n)となる最小の自然数 kが
φ(n)である事に反する。

逆に、rとφ(n)は互いに素とする。

g^rが、(g^r)^k≡1 (mod n)となる最小の自然数のkをとる。

g^(r*k)≡1 (mod n) よって、r*kはφ(n)で割り切れる。

ところが、rとφ(n)は互いに素だから、kがφ(n)で割り切れる。

よって、この条件を満たす最小の自然数はk=φ(n)である。

よって、題意は示された。


815名無し:01/10/10 04:30
すべての整数m,nで定義される実数値関数f(n,m):n^2→R
があり,任意のm,nに対して
4*f(n,m)=f(n+1,m)+f(n-1,m)+f(n,m+1)+f(n,m-1) および f(n,m)≧0
を満たしているとき,f(n,m)は定数関数であることを証明せよ。

不完全ですが,f(n,m)に最小値が存在するときは,次のように証明できるのかな?
f(n,m)の最小値をaとし,f(k,l)=aであるとする。また,関数g(n,m)=f(n,m)-a
とすると,すべてのn,mで
g(n,m)≧0 および4*g(n,m)=g(n+1,m)+g(n-1,m)+g(n,m+1)+g(n,m-1)
を満たす。ここで,4*g(k,l)=g(k+1,l)+g(k-1,l)+g(k,l+1)+g(k,l-1)
より,0=g(k+1,l)+g(k-1,l)+g(k,l+1)+g(k,l-1)
これが成り立つのはg(k+1,l)=g(k-1,l)=g(k,l+1)=g(k,l-1)=0
のときに限られる。以下,帰納的にすべてn,mについてg(n,m)=0がいえる。
すなわち f(n,m)=a
816815:01/10/10 04:32
1行目,間違えた。f(n,m):Z^2 → R だね。
818132人目の素数さん:01/10/10 05:11
すみません、詳しく教えてください。

f(x)を滑らかで単調増加な実数値関数とし、f(0)=0とする。
このとき、f(x)に対するNewton法による点列{x_n}が発散はしないが
零点x=0に収束しないような例を構成せよ。
819名無し:01/10/10 05:40
>>818
遅くなってすまん。
f(x)=-1.5x^4+8x^3-13.5x^2+8x として,初期点をx=1としてみそ。
面白いことになるよ。
820819:01/10/10 05:46
紛らわしいから,分数で書くね。
f(x)=(-3x^4+16x^3-27x^2+16x)/2 ってことだからね。
821819:01/10/10 05:49
やべえ,間違えた。単調増加じゃないといけないのか。
822819:01/10/10 06:04
訂正版 こっちなら大丈夫
f(x)=(2x^5-5x^3+7x)/2 で,初期点をx=1とすると,
次の点がx=-1,その次の点がx=1 となり,これを繰り返すはず。
ちなみに,f(x)は狭義単調増加。
823819:01/10/10 06:14
うわ,でも,発散しちゃいけないのか。振動するのも発散の一種か。
ってことは,0以外の点に収束するってことか?
狭義単調増加関数でそんなことがありえるのか?
824819:01/10/10 06:31
>>818 が書き込んですぐに気づけばよかったんだけど。
気づいたのが5時30分。 それからf(x)を設定して,>>819 を書いた。
そのあとのすったもんだは >>819-823 を見ての通り。
とっくに >>818 は愛想を尽かしているだろうに。
1人大騒ぎしている俺はなんてバカなんだ。打つだし脳・・・・
825804:01/10/10 17:39
>>812
実際にk,lが存在しないようなfと集合Aがあるんですか?

確かに方法を限定してましたね…もうちょっと頑張ってみます
826132人目の素数さん:01/10/10 19:03
a[1]=c
(2-a[n])*a[n+1]=1
の特性方程式を過程を含めて教えて下さい。
お願いします。
827132人目の素数さん:01/10/10 19:12
∀←これってどういう意味ですか
828なし:01/10/10 19:15
「任意の」「すべての」という意味で。記号論理が好みなもので。
829なし:01/10/10 19:22
A をひっくり返したもの。"for all"、「任意の…に対して」、「全ての…に対して」
全称量化子と呼ばれる論理記号。
830なし :01/10/10 19:31
for all。任意の...について。全ての...について。量化子と呼ばれる論理記号。Aをひっくり返したもの。
831なし:01/10/10 19:33
>>828-830

何か遅いと思ったら、3重カキコしてしまった。
832132人目の素数さん:01/10/10 19:52
>828
>記号論理が好みなもので。

どういう意味?
833なし:01/10/10 19:59
>>832 ある種の数学の形式化とか自動化に興味があるので。
834なし:01/10/10 20:22
>832
アルゴリズムの形式化とか、機械的自動化に興味深々。
835132人目の素数さん:01/10/10 20:48
>>833-834
そうじゃなくて>827の質問に対して
>「任意の」「すべての」という意味で。記号論理が好みなもので。

というのは、記号論理が好みだから何?ってこと
>827の質問は、君のレスのどれかへの質問だったってこと?
記号論理が好きかどうかに関わらず使われていると思うし
となると益々これ↓が何を言いたいのかわからない

>記号論理が好みなもので。
836 :01/10/10 21:03
1≠0.9999....
であることを
リアル厨房にわかりやすく説明してください。
>>836
いろんな意味で無理
838132人目の素数さん:01/10/10 22:22
f∈H^2
||f|| = |f(0)|^2+2∫[U]|f'(z)|^2*log|1/z|rdrdθ
φ:レンズ写像 ならば C(φ) compact
φ(z)=(1+z)/2 ならば C(φ) compactではない

以上3つの問題がどうしてもわかりません
ご教授願います
>>836
いろんな意味でガイシュツ
840132人目の素数さん:01/10/10 23:23
>>836
0=0である。
ここで
0=0*1、0=0*0.9999… と書けるから上式は
0*1=0*0.9999…
となり、この両辺を0で割ると
1=0.9999…

どう?わかった?
841827:01/10/10 23:36
分かりました
どうもありがとう(●´ー`●)
>>810なのですが、よろしくお願いします。
843132人目の素数さん:01/10/10 23:43
時間が無いので既出だったら大変スマソ&もう1回おしえて!!!
よくニュースや新聞で平均○○というけど例えば平均賃金サラリーマン
40万とかいいますよね、あれってどんな感じで計算してるの?俺的には
超高額&低額所得者の上下数百人はカットして計算すべきだと思うんですが
例  月給500万が1名  80万が3名  40万が2名
     30万が7名   20万が5名  15万が2名 
     合計20名の平均は58万だけど、どうみても上下カット
     すると35万くらいが平均と思うのは俺だけ?
844132人目の素数さん:01/10/10 23:43
>>840
それだと1=10000とかでも証明できそうな気が
845838:01/10/10 23:45
>>840
0で割るのがおかしい
846840:01/10/10 23:46
>>845
しぃ〜っ!
847132人目の素数さん:01/10/10 23:52
おいおい。
>>840を本気にしている奴がいるな。
>>847
本気も何も>>840は正解だよ。何言ってるの?
849132人目の素数さん:01/10/11 00:05
オレも840をしんじるよ!
850132人目の素数さん:01/10/11 00:06
    _____
   (〃___ノ  / ̄ ̄ ̄ ̄ ̄
  ◯(っ´∀`)◯ < >>840 君、
  》\ フVフ/》   \_____
  巛 |⌒I、│巛
   (_) ノ
      ∪



    | | | |
    _____
    (〃___ノ  / ̄ ̄ ̄ ̄ ̄
    (っ´∀`)っ < 素晴らしい証明をしたね。
   / フVフ\   \_____
  ○ |⌒I、│○
  Σ(_) ノ巛スタッ!
    _____
    (〃___ノ  / ̄ ̄ ̄ ̄ ̄
    (っ´∀`)っ < 正直、
   / フVフ\   \_____
  ○ |⌒I、│○
  巛(_) ノ巛
    _____
   (〃___ノ   / ̄ ̄ ̄ ̄ ̄
  ○(っ´∀`)○ < カッコイイと思うよ。
 / \ フVフ/ \ \_____
 |氏  |⌒I、│ね  |
 |___(_) ノ____|  
>>115
これ分かる人いますか。
851∈{Imai}
業務連絡ですわ
132人目のさくらちゃん、もしスレッドを立てる
元気がないのでしたら私にお任せください。

>>25を見る限りさくらちゃんが心配で心配で…どうしましょう
854132人目の素数さん:01/10/11 07:22
おしえちくり。

「pを素数として群Gの位数がp^nであるとき
Gの部分群の個数≡1+n (mod p)を示せ。」
855132人目の素数さん:01/10/11 07:51
置換を互換の積で書いたとき、互換の数の最小数はどう計算されるか。
851∈{z案}
>>852
>>856
それじゃあ答えられなくなると宇治蒸しと言う人と変わらないよ。
>>857

851=宇治蒸しという人だろ?
859132人目の素数さん:01/10/11 08:04
>857
うん。だから
851∈{Imai,z案}
なんでしょ?
860857:01/10/11 08:20
追加
質問に答えられないからって
>それじゃあ答えられなくなると宇治蒸しと言う人と変わらないよ。
861132人目の素数さん:01/10/11 08:21
Z案スレを読みきった俺としては>851は今井というよりZ案スレの1と同じニオイを感じる
862857:01/10/11 08:25
つまり
852≒宇治蒸しと言う人≒856
ということ。
863132人目の素数さん:01/10/11 08:26
今井v.s. z案か?こりゃ見ものだ(ワラ
Z案爆走中
>861
禿げ禿げ同
866856:01/10/11 09:16
スマソ。俺が間違ってた。猛省(w
851∈{野依}
>866
856は正しいと思うぞ
869132人目の素数さん:01/10/11 09:28
どう見ても

>851∈{z案}

だよ。
870852=866:01/10/11 09:35
866の856を852に訂正。更に猛省
871132人目の素数さん:01/10/11 10:07
教えてください。

M:メビウスバンドル
g=(M,p,s^1):vector bundle
C^∞(s^1):s^1上のC^∞ function全体のなすring
C^∞(s^1,M):C^∞ section全体のなすアーベル群
この時

(1)C^∞(s^1,M)はC^∞(s^1)-moduleである事を示せ。
(2)C^∞(s^1,M)はfree C^∞(s^1)-moduleでは無い事を示せ。

(1)は何とかなりますが、(2)はさっぱり分かりません。
>>843
単に平均と言ったら算術平均を意味することが多いけれど、
統計的な代表値(広義の平均)としては、
算術平均よりも中央値(メジアン)や最頻値(モード)の方が
適切であるという話もある。
873>854:01/10/11 14:07
これはなかなかいい問題やと思う.
(もしかして既出?もしくは有名問題?)
大雑把な考え方だけなので分からなかったら聞いて.
位数p^s(s=0,1,2,…,n)の部分群の数をr[s]とする.
r[n]=1は明らか
r[n-1]≡1(modp) を示す.それは次の手順で示す.
@Gは冪零である.
A位数p^(n-1)のGの部分群は正規である事を示す.
B1つはあることを示す.(これがけっこうめんどい)
C2つ以上あったら、その2つの共通部分も群で、それで割った群を考えると
 準同型定理からその共通部分は位数p^(n-2)でさらにG/(その共通部分)は
 Z/pZ×Z/pZ と同型であることが分かる.
 これよりこの共通部分を含む位数p^(n-1)の群は数えれてp+1個
 他にまだ異なる群があれば、それは今の共通部分とは異なる群を含み、同様に
 くりかえせば、r[n-1]≡1(modp)
 が示せる.
次にr[1]≡1(modp) を示す.次の手順で示す.
@巡回群である.
Aよって位数pの元はr[1]*(p-1)個
BGの中心に属する位数pの元の集合をX
Gの中心に属さない位数pの元の集合をYとする. #X+#Y=r[1]*(p-1) である.
 Xの元と単位元で部分群となるから、#X=-1(modp)
 Yの元を1つaとすると、gag^(-1)の位数もpで共役をとる作用について閉じている.
 これは#Y≡0(modp) を示していて、以上から分かる.

これらを総合すると、位数p^sの群を含むような位数p^(s+1)の群は1(modp)個
位数p^(s+1)の群に含まれるような位数p^sの群も1(modp)個.
これより、r[s]≡r[s+1]が分かる
以上より題意成立.

かなりはしょったな。。。読んでも分からんかも(笑)
わるい
874132人目の素数さん:01/10/11 14:21
>>843
>超高額&低額所得者の上下数百人はカットして計算すべきだと思うんですが

異常値はカットされています。
全体の何%とかによりますが人数的に数百人であれば数%も無いですから
カットされている範囲に入っていると思います。
875132人目の素数さん:01/10/11 16:02
∫(cosx)^5dx

を置換(t=sinx)で解いて下さい。
876タリバン先生:01/10/11 16:09
>875
dt/dx = cos x

∫(cosx)^5dx =∫cosx (1-sin^2 x)^2 dx=∫(1-t^2)^2 dt
∫(t^4-2t^2+1)dt= (t^5)/5 - 2 (t^3)/3 +t +c
877なし:01/10/11 17:09
>>835
>記号論理が好きかどうかに関わらず使われていると思うし

使われていても、知らない人がいることを配慮していなかった私どもの
不注意により、このようなご迷惑をおかけ致したことに付いては、責任者
共々遺憾に思っております。
878132人目の素数さん:01/10/11 17:18
タリバン先生三球。

ついでに、

∫1/(cos^2 x+3sin^2 x) dx

を置換(t=tanx)で解いて下さい。
>877
>知らない人がいることを配慮していなかった私どもの不注意により、

だからさ、あんた何番だよ?
私どもって他に誰かいるのかよ?
責任者って誰だよ?
そもそも>827は、あんたらのレスからコピってきて質問したのかよ?
880なし:01/10/11 17:45
>>879 ちょっと待った。今忙しいから後で答えるよ。
>>878 タリバン先生の代わりに答えます。
dt/dx = 1/cos^2 x = t^2 + 1.

dx/(cos^2 x+3sin^2 x) =(dt/(t^2+1)) / (3 - 2/(t^2 + 1))
=dt / (3(t^2 + 1) - 2).
881132人目の素数さん:01/10/11 17:52
便乗みたいで悪いけど、

∫arctan x/2 dx

もお願い。途中経過がよくわかんないんだ・・・
882:01/10/11 17:57
arctan(x/2) だよね?
883132人目の素数さん:01/10/11 17:58
>>882
そうです。
884タリバン先生:01/10/11 18:00
>880
えっと、解く気があるのでしょうか?
885132人目の素数さん:01/10/11 18:07
すいません、既に別スレ立てちゃったんですけど、誰か教えて下さい。
http://cheese.2ch.net/test/read.cgi/math/1002722455/l50

p, q を素数、M < pq (M は pq と互いに素とは限らない)で、
M = M^x mod pq
を満たす x は、k を非負の整数として、
x = k(p - 1)(q - 1) + 1
だと、あるホームページに書いてるんですが、これってどのよう
にして、この式が導かれるのでしょうか?
886132人目の素数さん:01/10/11 18:26
>>880
なしまた逃げたのか?
887タリバン先生:01/10/11 18:44
>878
どうやら>880に逃げられてしまったようなので私が解きます。

dt/dx = 1/(cos^2 x)

∫1/(cos^2 x+3sin^2 x) dx = ∫1/((1+3tan^2 x)(cos^2 x))dx
= ∫1/(1+3 t^2) dt

s= √3 t と置いて

= (1/√3)∫1/(1+s^2) ds

s = tan yと置いて ds/dy = 1/(cos^2 y)

= (1/√3)∫1/(1+s^2) ds = (1/√3)∫1 dy = (1/√3) y +C

y= arctan s = arctan (√3 t) = arctan (√3 tan x)
を代入して終わり
888タリバン先生:01/10/11 18:57
>>881
t=arctan(x/2)
x = 2 tan t

dx/dt = 2(1+tan^2 t)= 2 + (x^2)/2

dt/dx=2/(4+x^2)

∫arctan x/2 dx
= x arctan x/2 - ∫x (2/(4+x^2)) dx (部分積分)
=x arctan(x/2)-log(4+x^2)+c
889タリバン先生:01/10/11 19:22
>885
そのHPに書いてある通りフェルマーの小定理からだと思いますが?
互いに素でないときもMがpの倍数とでも思ってM=R p^αみたいに
pの部分を外してしまってp^(q-1) mod q ≡1等を使うんでは?
890885:01/10/11 19:50
>>889
んー、すいません、ちょっと意味が理解できないです...
891132人目の素数さん:01/10/11 20:00
ファルマーの最終定理ってまだだれも解けてないんだよね?
892:01/10/11 20:20
大歩危
893>891:01/10/11 20:21
この板の看板はだれ?
894ちむ教の信者:01/10/11 20:22
乗法定理がよくわかりません。
簡単に説明してください。お願いします。
895132人目の素数さん:01/10/11 21:04
>885
pqとMが互いに素なとき、フェルマーの小定理は成立しないし、
暗号も作れない.
最初から、p<q
として、M<p のとき間違いなく成立する.
暗号はこれで十分.
896895:01/10/12 01:19
↑ごめんなさい間違いです
897132人目の素数さん:01/10/12 03:20
880 名前:なし 投稿日:01/10/11 17:45
>>879 ちょっと待った。今忙しいから後で答えるよ。
>>878 タリバン先生の代わりに答えます。
dt/dx = 1/cos^2 x = t^2 + 1.

dx/(cos^2 x+3sin^2 x) =(dt/(t^2+1)) / (3 - 2/(t^2 + 1))
=dt / (3(t^2 + 1) - 2).

884 名前:タリバン先生 投稿日:01/10/11 18:00
>880
えっと、解く気があるのでしょうか?

886 名前:132人目の素数さん 投稿日:01/10/11 18:26
>>880
なしまた逃げたのか?
898 ◆pvySbQO2 :01/10/12 09:00
>>855
互換の最小個数を求めるには一致していない部分を
一致するように互換を使っていけば求まります。
例えば[1,2,3,4,5]を[3,4,1,5,2]にするには
[1,2,3,4,5]の1と[3,4,1,5,2]の3が
一致していないので1と3を入れ換えて[3,2,1,4,5]とする。
[3,2,1,4,5]の5と[3,4,1,5,2]の2が
一致していないので5と2を入れ換えて[3,5,1,4,2]とする。
[3,5,1,4,2]の5と[3,4,1,5,2]の4が
一致していないので5と4を入れ換えて[3,4,1,5,2]とする。
これで一致したので互換の最小個数は3個と分かります。

もうひとつの方法は対象となるものの数から環の数を引く方法です。
[1,2,3,4,5]を[3,4,1,5,2]にする場合は
1−>3−>1と2−>4−>5−>2の2つの環があるので
互換の最小個数は5−2=3個です。

>>861
z案の人は僕に似ているのですか。
>>898
>z案の人は僕に似ているのですか。

861じゃないけど、そっくりだと思う
彼は大学2年生くらいだから質問のレベルからするに同年くらいでは?
900132人目の素数さん:01/10/12 14:50
タリバン先生 わかりやすい回答サンキュ。

∫(2x+3)/(x^2+4)dx

∫1/(x^2+2x+2)dx

∫(2x-3)/(x^2-2x+4)dx

これらの問題の途中過程と結果もお願いします。

このような問題での基本変形の仕方で
行き詰まってます・・・ポイントがあれば教えて。
901132人目の素数さん:01/10/12 15:03
>>900
部分分数分解
902なし:01/10/12 15:34
1/(ax^2+bx+c)=1/((x-α)(x-β))
=(α-β)(1/(x-α) - 1/(x-β)).

dx/(ax^2+bx+c)=d(α-β)(x/(x-α) - x/(x-β)).
=d(α-β)((1 + α/(x-α)) - (1 + β/(x-β))).
=d(α-β)(α/(x-α) - β/(x-β)).
903なし:01/10/12 15:44
(px+q)/(ax^2+bx+c)
=p(α-β)(α/(x-α) - β/(x-β)) + q(α-β)(1/(x-α) - 1/(x-β))
=(α-β)(pα/(x-α) - pβ/(x-β) + q/(x-α) - q/(x-β))
904なし:01/10/12 16:10
部分分数分解で分子の次数が少なくするか、
平方完成して ∫dx/(1-x^2), ∫dx/(1+x^2), ...
の形に変形するか。問題集や公式集では、形で分類しているよね。
905132人目の素数さん:01/10/12 16:25
873の解答が理解できないのですが。。。
(通りすがりに見てただけですが)
誰か、もうちょっと分かりやすく説明してもらえませんか?
906なし:01/10/12 16:31
1/(x^2+2x+2)=1/((x+1)^2+1)
1/(x^2+4)=1/(4(x/2)^2+4)=4/((x/2)^2+1).
1/(x^2-2x+4)=1/((x-1)^2+3)=1/(3((x-1)/√3)^2+3)=3/((x-1)/√3)^2+1).

全部 1/(t^2±1) の形になる。これは定式。暗記すると楽。
xsinx^2の積分教えてください。
908132人目のともよちゃん:01/10/12 22:32
━━━━━━━━━━━━━━━━━━━━━━━━━━━━━━

            このスレッドは終了しましたわ
  質問がある人は、以下の新スレに行って頂けると嬉しいですわ

        ◆ わからない問題はここに書いてね 14 ◆
     http://cheese.2ch.net/test/read.cgi/math/1002893257/


━━━━━━━━━━━━━━━━━━━━━━━━━━━━━━
>>899
そうすると大学3年以上なら >>115 の問題に答えられるんですか。
910ちむ教の信者:01/10/13 19:19
8人の生徒がいる。次のようなわけ方はなん通りあるか。
(1)P,Q,R,Sの4つの部屋に2人ずつ入るように分ける。
(2)2人ずつの4つのグループに分ける。
(1)はわかるんですけど、(解2520通り)(2)の質問の意味及び
なぜ、これが(1)の解を利用して答えるのかがわかりません
式 X×4!=2520
  また、このXはほかの方法で求められないのでしょうか?
  それと、なぜ4!なんですか?これになる原因も教えてください。
  お願いします。
部屋が4つだからにキマッテルダロ
>>910
8人を a,b,c,d,X,Y,Z,W で表すことにして、
i) a,b,c,dが同じ組にならない場合… 4!
例 (aX)(bY)(cZ)(dW)
ii) a,b,c,dのうち二人が同じ組… 4C2 * 4P2
例 (ab)(cX)(dY)(ZW)
iii) a,b,c,dで二組 … 3 * 3
例 (ab)(cd)(XY)(ZW)
と、場合分けすることも出来るし、こういう考え方も必要。

だけど、X * 4! = 2520 という等式はもっと重要。
nCr * r! = nPr と同じ意味合い。
913なし:01/10/16 11:50
微分の結合則に対応するものは、積分の置換積分です。

t=g(x) とします。dt/dx = f(t) と表されるとき
∫dx/f(g(x))=∫f(t)dt/f(t)=∫dt となります。

この式を使えば、同種の積分の問題が無限に作れるでしょう。
914なし:01/10/16 12:06
x sin(x^2)を部分積分すれば、 cos(x^2) のみの積分になる。
後は置換積分を使うこと。
915132人目の素数さん:01/10/16 16:16
四点A(2,2,3)B(1,−3,1)C(1,2,−1)D(3,4,5)がある。AB,AC,ADを三辺とする平行六面体の頂点の座標を求めよ。
この問題の解答を詳しく教えてください。お願いします。
916名無し募集中。。。:01/10/16 17:18
次スレに移動してね。
ここに書かれた質問にはもう答えません。
917 :01/10/30 02:59
将棋の必勝法、連珠の必勝法、オセロの必勝法。
>>917
自分で頑張って下さいな
919132人目の素数さん:01/11/04 22:53
Mathematicaに関する質問なんですが・・・
自然数nを用いて表された量に関して,ある一定の範囲のnにおける
総和を求める計算は Sum を用いて計算できますが,
自然数n,mを用いて表された量に関して
nとmに一定の制約条件がある場合,その条件を満たす範囲での総和を
求めるにはどうしたらいいんですか?
ちなみに制約条件とは
 m+n=(偶数), m>0,n>0,m>n+p (p:定数)
です。
mとnを含む量が階乗で表された複雑な形になっているもので,
手計算では歯がたちません。
全くの初心者&数学の知識なんて全くないもので
すごく基本的な質問かも知れませんが
どなたか教えていただけると幸いです。
920初めまして。:01/11/04 22:54
悩んでますが解けません。ぜひ,皆様のお力を...
@1/cos(x)の不定積分を求めよ。
Ap>0,q<0の時,(p-q)^2/(p^2+q^2)の最大値を求めよ。

例の検定で出た問題です。
@はt=sin(x)と置くことで,置換積分で解けましたが,なぜか,手持ちの参考書の解答と一致しません。
計算がちがっているのか,その本が間違っているのか???
正しい答えをお教え下さい。

p+q=a,pq=b とでも置くのかな?と思ったんですが,置いても手が着かないです。。。
最小値なら出るようなんですが。。。
予想は『max=2』!当たってますか?
921文系:01/11/04 23:02
オマエラ、このスレに書きこむんじゃねーよ
クズどもめ
>>920
@あなたの計算、手持ちの参考書、
 微分して1/cos(x)になるほうが正解です。
A当たってます。
923高野:01/11/05 18:40
2(x−2y)2乗ー4x(x−2y)
━━━━━━━━━━━━━━━━━━━━━━━━━━━━━━

             新しいスレッドが出来ましたので
     新たに質問をする方はこちらでして頂けると嬉しいですわ

         ◆ わからない問題はここに書いてね 15 ◆
     http://cheese.2ch.net/test/read.cgi/math/1004171159/


━━━━━━━━━━━━━━━━━━━━━━━━━━━━━━
925132人目の素数さん:01/11/20 22:52
間違ってスレ12に書いてしまいました.すみません.

重なった部分が線対称の五角形となるように一辺がaの正方形を折る.
この時の五角形の面積Sの最大値を求めよ.がわかりません.,
━━━━━━━━━━━━━━━━━━━━━━━━━━━━━━

             新しいスレッドが出来ましたので
     新たに質問をする方はこちらでして頂けると嬉しいですわ

         ◆ わからない問題はここに書いてね 15 ◆
     http://cheese.2ch.net/test/read.cgi/math/1004171159/


━━━━━━━━━━━━━━━━━━━━━━━━━━━━━━
━━━━━━━━━━━━━━━━━━━━━━━━━━━━━━

             新しいスレッドが出来ましたので
     新たに質問をする方はこちらでして頂けると嬉しいですわ

         ◆ わからない問題はここに書いてね 16 ◆
     http://cheese.2ch.net/test/read.cgi/math/1005735838/l50

━━━━━━━━━━━━━━━━━━━━━━━━━━━━━━
928 ◆pvySbQO2 :01/12/16 02:00
(df(i+1)/dt)(t)=−f(i)(t)f(i+1)(t)
f(i)(0)=1
のとき
(d/dt)(f(i+1)(t)exp(∫_[0,t]f(i)(t)dt))=0。
f(i+1)(t)exp(∫_[0,t]f(i)(t)dt)=1。
f(i+1)(t)=exp(−∫_[0,t]f(i)(t)dt)。
逆にこのとき
(df(i+1)/dt)(t)=−f(i)(t)f(i+1)(t)
f(i)(0)=1
となる。

0≦f(1)(t)≦f(0)(t)から
f(2k+1)(t)≦f(2k+3)(t)≦f(2k+2)(t)≦f(2k)(t)
となるので
g(t)=lim_{k−>∞}f(2k+1)(t)
h(t)=lim_{k−>∞}f(2k)(t)
が存在する。

f(2k+1)(t)=exp(−∫_[0,t]f(2k)(t)dt)
f(2k+2)(t)=exp(−∫_[0,t]f(2k+1)(t)dt)
でk−>∞として
g(t)=exp(−∫_[0,t]h(t)dt)
h(t)=exp(−∫_[0,t]g(t)dt)
となる。
∫_[0,t]g(t)dt,∫_[0,t]h(t)dtはtの連続関数なので
h(t),g(t)はtの連続関数で
h(t),g(t)はtの連続関数なので
g(t),h(t)はtで微分可能となるのでtで微分して
(dg/dt)(t)=−g(t)h(t)
(dh/dt)(t)=−g(t)h(t)
d(g−h)/dt=0
g(0)−h(0)=1−1=0なのでg=hとなり
f(t)=lim_{i−>∞}f(i)(t)は存在し
(df/dt)(t)=−f(t)^2となる。
0<f(1)(t)=exp(−t)≦f(t)なのでf(t)≠0。
(d/dt)(1/f(t))=1。
1/f(t)=1+t。
f(t)=1/(1+t)。
lim_{i−>∞}f(i)(t)=1/(1+t)。
981超えた後3日はもつかと思ったけど1日もたない場合もあるみたいね
そこら辺は運次第…って事かな?
982132人目の素数さん:02/01/28 23:02
大学教員の授業に対して
最近、かなりきつくなって来ました。
今回大学の方から、来年の『ベクトル解析』の授業
の目的とねらいを[我々]が納得出来るようまとめて
提出するよう求められ、何回も書いては、捨ててる状態
が続いております。
なにか、良いお知恵御座いませんでしょうか
もし宜しければお教え頂ければ大変嬉しくおもいます。
984埋め立て屋:02/01/30 16:51
◆ わからない問題はここに書いてね 21 ◆
http://cheese.2ch.net/test/read.cgi/math/1011689052/
985埋め立て屋:02/01/30 16:51
 
986埋め立て屋:02/01/30 16:51

987埋め立て屋:02/01/30 16:51
 
988埋め立て屋:02/01/30 16:51
988
989埋め立て屋:02/01/30 16:51
989
990埋め立て屋:02/01/30 16:52
◆ わからない問題はここに書いてね 21 ◆
http://cheese.2ch.net/test/read.cgi/math/1011689052/
991埋め立て屋:02/01/30 16:52
◆ わからない問題はここに書いてね 21 ◆
http://cheese.2ch.net/test/read.cgi/math/1011689052/
992埋め立て屋:02/01/30 16:52
◆ わからない問題はここに書いてね 21 ◆
http://cheese.2ch.net/test/read.cgi/math/1011689052/
993埋め立て屋:02/01/30 16:52
◆ わからない問題はここに書いてね 21 ◆
http://cheese.2ch.net/test/read.cgi/math/1011689052/
994埋め立て屋:02/01/30 16:52
 
995埋め立て屋:02/01/30 16:52

996埋め立て屋:02/01/30 16:52
 
997埋め立て屋:02/01/30 16:53
◆ わからない問題はここに書いてね 21 ◆
http://cheese.2ch.net/test/read.cgi/math/1011689052/
998埋め立て屋:02/01/30 16:53
◆ わからない問題はここに書いてね 21 ◆
http://cheese.2ch.net/test/read.cgi/math/1011689052/
999埋め立て屋:02/01/30 16:53
◆ わからない問題はここに書いてね 21 ◆
http://cheese.2ch.net/test/read.cgi/math/1011689052/ 
1000です。。。
10011001
このスレッドは1000を超えました。
もう書けないので、新しいスレッドを立ててくださいです。。。